Sei sulla pagina 1di 270

ANANTARA

RESPI

1. Pulmonary disturbance pasient need Blood gases analysis to deternime the diagnose of disease.
What is the indication of blood gases analysis ?

A. To evaluate the level of bicarbonate in the lung


B. Titrate the appropriate CO2 oxygen flow rate
C. Qualify a patient for home monitor of bronchodilator therapy oxygenation use
D. To evaluate ventilatory and acid base disturbance and monitor effectiveness of therapy
E. To evaluate renal compensation for patient with pulmonary disturbance

2. Mr. Cody is flight in the altitude of 8000 feet. The average atmospheric pressure is about 564 mmHg and oxygen is 118
mmHg.
How much the PO2 pressure in this altitude?

A. 0.21
B. 21%
C. 118 mmHg
D. 446 mmHg
E. 44.6 mmHg

3. A 50-year-old man with history of tobacco use who complained of several months of cough and lower back pain and an
11.3-kg weight loss. He was treated for pneumonia after a chest radiograph showed total opacification of the right lung.
Computed tomography imaging subsequently revealed a superior right hilar mass and mediastinal lymphadenopathy.
Mediastinal biopsy showed poorly differentiated epithelioid tumour with desmoplastic stromal reaction, neutrophil
infiltration, and squamous differentiation. Tissue immunostaining confirmed a non-small-cell lung cancer.
Which nuclear medicine examination is the most suitable for the case management?

A. Bone scan
B. V/Q study
C. Tc-99m Sestamibi Whole Body Scan
D. Tc-99m Ethambutol scan
E. F-18 FDG PET Scan

4. Which of the following is NOT a function of the lungs?

A. Metabolism
B. It is a filter to protect the systemic vasulature
C. Serves as a reservoir of the blood for the left ventricle
D. Facilitate the exchange of O2 and CO2 between air and blood
E. All of the above are true

5. A patient has been suffering severe asthma attack for the last three hours. Do you think this patient can experience blood pH
disturbance?

A. No, because CO2 that can not be excreted through the lung will be excreted through the skin
B. No, because beside lung, kidney can also act as a blood pH stabilizer
C. Alkalosis will be developed because higher respiratory rate leads to high CO2 excretion
D. Acidosis will be developed because less blood O2 leads to anaerobic metabolism that produces a lot of lactic acid
E. Acidosis will be developed because airway obstruction leads to less CO2 excretion

6. Which of the following statement concerning Ipratropium bromide is correct?


A. It is cholinoceptor agonist
B. More potent than ß2-agonist agents in reversing asthmatic bronchospasm
C. Reduce the bronchodilation produced by nebulized albuterol
D. It is highly absorbed and readily enter the CNS
E. It is derivative of atropine and more selective than the parent compound

7. This structure forming the superior part of the nasal septum, descends from the cribriform plate and is continued superior to
this structure as the crista galli. Which of the following structure is correct based on above description?

A. Nasal part of the frontal bone


B. Frontal processes of the maxilla
C. Palatine process of maxilla
D. Horizontal plate of palatine bone
E. Perpendicular plate of the ethmoid bone

8. Anisa 9-month-old girl patient present to pediatric emergency room with the difficulty of breathing and suffered from fever
since the last 3 days which has proceeded by common cold sincw 4 days before.
After taking history, physical examination and preliminary investigation, she is diagnosed with atypical Pneumonia and
treated by Macrolide drugs.
Which of the following is the right statement?

A. Azithromycin inactive cytochrome P450 enzyme


B. Azithromycin's bioavailability is altered by aluminium
C. Azithromycin's bioavailability is altered by magnesium
D. Magnesium increases peak serum concentration
E. Magnesium delays absorption of Azithromycin

9. What is the predicted lung compensatory response to low blood PH?

A. Decrease PCO2 and decrease bicarbonate ion


B. Decrease PCO2 and increase bicarbonate ion
C. Increase PCO2 and decrease bicarbonate ion
D. Increase PCO2 and increase bicarbonate ion
E. Decrease bicarbonate ion only

10. A man was brought to the Emergency Unit. The doctor did the physical examination, and declared that he was dead. On
examination, a horizontal ligature mark was seen encircling the neck, presence of conjunctival petechial haemorrhages, and
congestion of the face. The body was then sent to Forensic Medicine Department to undergo autopsy. Autopsy findings
revealed Tardieu's spot, cerebral hypoxia, and fracture of the superior horn of the thyroid cartilage.
What is most likely cause of death?

A. Accident
B. Hanging
C. Homicide
D. Strangulation
E. Suicide

11. An 11-month-old baby boy without underlying congenital abnormality presented to the Emergency Room with a history of
fever for one day associated with runny nose, noisy breathing, barking cough, dyspnea, hoarseness of voice and irritability.
His respiratory rate was 60 breaths per minute and oxygen saturation of 99%. There were also presence of inspiratory
stridor, subcostal recession and nasal flaring. He was treated with nebulized adrenaline 0.5 mg, iv hydrocortisone 50 mg and
oxygen via nasal prong.
How many types does this agent has?

A. One
B. Two
C. Three
D. Four
E. Five

12. A 30-year-old male, developed severe pneumonia after 8 days of hospitalization due to fracture of his right femur.
What is the most likely antibiotics of choice for the above patient?

A. Aminoglycoside and ceftazidime


B. 2nd generation cephalosporin and macrolide
C. 1st generation cephalosporin and aminoglycoside
D. 2nd generation and macrolide
E. Aminoglycoside and macrolide

13. A 33-year-old man complaining of chest pain on the left side since 3 days ago accompanied by fever. The pain was
worsened by sneezing, coughing, and lying on the affected side. He had to sleep on his right side. The doctor diagnosed him
as having pleuritis TB.
Which of the following best describes the affected organ?

A. The two epithelial membranes are non-continuous and forms a sac


B. Between the visceral and parietal layer there is a potential space that contains air in normal individual
C. The visceral pleura covers the surface of the lung, except on its fissures
D. The diaphragm part of parietal pleura adheres to posterior surface of diaphragm
E. The mediastinal pleura reflects at the hilar and becomes the visceral pleura

14. A 33-year-old male presented in your clinic with productive cough for 4 weeks, low-grade fever, and weight loss. This
patient smokes 1 pack of cigarettes every day. X-ray revealed patchy lesions in both upper lung.
What is the easiest microbiology examination which should be performed for this patient?

A. CT scan of the chest


B. Bacterial culture
C. Smear microscopy
D. Observation
E. Bronchoscopy

15. Which number is pointed to the structure that supplies segments in lung?

A. 4
B. 5
C. 6
D. 7
E. 8

16. A premature male infant developed rapid, shallow respiration shortly after birth. A diagnosis of respiratory distress
syndrome (RDS) was made.
Which of the following statement is the most appropriate one in relation with this condition?

A. The surfactant increasing surface tension at the air-alveolar interface


B. Surfactant production begins by 18 weeks 20-22 w
C. Type I alveolar cells secretes pulmonary surfactant
D. By 24 to 26 weeks, the fetus has sufficient surfactant 26-28 w
E. The fetus has sufficient surfactant since terminal saccular period

17. A 12-year-old girl with a childhood history of asthma complained of cough, dyspnea, and wheezing. Her symptoms became
severe that her parent brought her to the emergency room. Physical examination revealed diaphoresis, dyspnea, tachycardia,
and tachypnea. Her respiratory rate was 42/min, pulse rate was 110 beats per minute, and blood pressure was 132/65 mmHg.
A drug contraindicated in this patient.

A. Inhaled cromolyn
B. Oral or IV methylprednisolone
C. Inhaled ipratropium
D. Inhaled albuterol
E. Intravenous propanolol

18. What is the irrationale management for Acute Rhinosinusitis?

A. Hydration dan good nutrition


B. Herbal treatment such as Pellargonium sidoides
C. Consider antibiotic if the symptoms acute bacterialis rhinosinusitis occur
D. No need antibiotic at all in the first ten day illness
E. Supportive treatment

19. Alternative drug for the penicillin-hypersensitive individual is :

A. Ceftriaxone
B. Ticarcillin
C. Amikacin
D. Erythromycin
E. Tetracycline

20. Patchy consolidation in post-primary pulmonary tuberculosis is usually appears in :

A. Anterior segments of the upper lobe apical and posterior


B. Apex of an upper or lower lobe
C. Lateral segment of middle lobe
D. Medial segment of middle lobe
E. Lingula segment of upper lobe

21. Patient admitted due to traffic accident with RR 36-40x/min, BP90/70mmHg jugular vein distension, asymmetrical chest
movement, trachea distortion. What's the diagnosis?
A. Cardiac tamponade
B. Massive Haematothorax
C. Vena Cave Superior Syndrome
D. Tension Pneumothorax
E. Simple Pneumothorax

22. PO2 can be calculated with the equation below....

A. PIO2-(VO2/VA)863 mmHg
B. PICO2-(VCO2/VA)863 mmHg
C. (VO2/VA)863 mmHg
D. (VCO2/VA)863 mmHg
E. None of the above

23. A 5-year-old girl is admitted to the hospital with an upper respiratory tract infection. During physical examination her sense
of hearing appears to be poor. Her right ear is painful, and upon otoscopic examination a golden brown fluid can be
observed through the tympanic membrane.
Which is the most likely direct route for the spread of an infection from the upper respiratory tract to the middle ear cavity?

A. Pharyngotympanic tube
B. Choanae
C. Nostrils
D. Facial canal
E. Internal acoustic meatus

24. A 6-year-old boy presented to the emergency ward with high fever, cough and air hunger/breathlessness progressing within
the last 12 hours as chief complaint and then he diagnosed as clinically suspected epiglottitis.
Describe the right statement of epiglottitis accordingly :

A. Beta-hemolytic streptococcus is usually isolated from blood cultures


B. A temperature above 38.5°C would cast doubt on the diagnosis
C. Children characteristically prefer to sit leaning forward
D. Virtually all cases will require tracheostomy
E. An intramuscular dose of an appropriate antibiotic should be given as early as possible before the reaches hospital

25. A 26-year-old female works as a teacher complained of productive cough for 10 days, accompanied with malaise, fever, and
chest pain. The patient did not feel any muscle ache or pain. An x-ray revealed patchy consolidation in the right lung and the
white blood count was slightly increased. Microscopic examination revealed Gram (+) cocci.
What is the most likely bacteria causing this disease?

A. Staphylococcus aureus
B. Chlamydia trachomatis
C. Streptococcus pneumoniae
D. Pseudomonas aeruginosa
E. Mycobacterium tuberculosis

26. Anti-tuberculosis medication side-effects constitute major factor for poor adherence to tuberculosis treatment. What is the
most likely anti-tuberculosis drugs which have hepatotoxic effect?
A. Rifampicin, isoniazide, ethambutol
B. Rifampicin, isoniazide, pyrazinamide
C. Pyrazinamide, ethambutol. Streptomycin
D. Streptomycin, rifampicin, ethambutol
E. Isoniazide, streptomycin, pyrazinamide

27. Twelve years old child with severe wheezing had laboratory result of IgE Level ; 597 IU/ml, the reference value of IgE
Level as shown below : 0-380 IU/mL
What is the method to measure the level of IgE?

A. Immunochemistry test by RIA


B. Immunodiagnostic tests by ELISA method
C. Immunoelectroassay method
D. Immunoelectroassay by ELISA method
E. Immunodiagnostic test by RIA

28. Select the strongest stimulator of respiratory centre in the brain.

A. Blood pH
B. Blood O2
C. Blood CO2
D. Blood lactate
E. Blood haemoglobin

29. A 33-year-old male presented in your clinic with productive cough for 4 weeks, low-grade fever, and weight loss. This
patient smokes 1 pack of cigarettes every day. X-ray revealed patchy lesions in both upper lung.
What is the property of this bacteria which allows it to be stained and identified microscopically?

A. DNA
B. RNA
C. Mycolic acid of the cell wall
D. Lipoarabinomannan
E. UreC gene

30. An 11-month-old baby boy without underlying congenital abnormality presented to the Emergency Room with a history of
fever for one day associated with runny nose, noisy breathing, barking cough, dyspnea, hoarseness of voice and irritability.
His respiratory rate was 60 breaths per minute and oxygen saturation of 99%. There were also presence of inspiratory
stridor, subcostal recession and nasal flaring. He was treated with nebulized adrenaline 0.5 mg, iv hydrocortisone 50 mg and
oxygen via nasal prong.
What is the method to determine the causative agent?

A. CT scan
B. Culture
C. Smear microscopy
D. DNA-ase test
E. Real time PCR

31. A 26-year-old female works as a teacher complained of productive cough for 10 days, accompanied with malaise, fever, and
chest pain. The patient did not feel any muscle ache or pain. An x-ray revealed patchy consolidation in the right lung and the
white blood count was slightly increased. Microscopic examination revealed Gram (+) cocci.
When is the best time to collect sputum for microbiology examination?

A. Before the patient going to bed in the night


B. In the afternoon
C. Anytime
D. In the morning right after the patient wakes up
E. After breakfast

32. Lobar consolidation with positive air bronchograms can be found in :

A. Pneumonia
B. Pleural effusion
C. Atelectasis
D. Bronchopnemonia
E. Schwarte

33. Which of the following reaction take place at the pulmonary capillary?

A. Hb+O2 → HbO2
B. CO2+H2O → H2CO3 → H++HCO3-
C. Hb+CO2 → HbCO2
D. HbH → Hb+H+
E. All are true

34. This upper part of the pharynx, is a broad single cavity overlying the soft palate. It is continuous anteriorly with the nasal
fossae and inferiorly with the oral part of the pharynx. The walls are supported by bone and skeletal muscle.
Which of the following epithelial tissue is most likely lined the mucous layer of this cavity?

A. Stratified squamous non keratinized


B. Stratified squamous keratinized
C. Respiratory epithelium
D. Stratified columnar
E. Stratified cuboidal

35. Choose one of the most appropriate option from the statement below:
The tonsil commonly called the adenoid when enlarged.

A. Lingual tonsil
B. Palatine tonsil
C. Pharyngeal tonsil
D. Tubal tonsil

36. Which structure is part of the conducting portion of the airway?


A. Bronchiole
B. Bronchus
C. Terminal bronchioles
D. Respiratory bronchioles
E. Alveolar duct

37. Which of the following is fact about Rhinosinusitis?

A. Rhinitis and sinusitis are not usually coexist and concurrent


B. The correct terminology of sinusitis causes by apical dental problems is sinusitis.
C. Impact significantly on patient quality of life, productivity and healthcare resource utilization
D. The newest pathophysiology mechanism is about osteomeatal complex
E. Antibiotic not always given

38. A 50-year-old woman is admitted to the hospital with a swollen right calf. She has history of surgical cholecystectomy 10
days ago. Physical and diagnostic findings reveal a deep vein thrombosis. On the next day, she suffered a mild-to-moderate
dyspnea. No history of dyspnea or respiratory before. Chest X-ray shows clear lungs without infiltrates.
Which one is the most suitable diagnostic finding for the working diagnosis?

A. Mismatch between ventilation and perfusion of the lung


B. Decrease blood flow of the blocked area in the calf
C. Pathological uptake in lungs apex
D. Hypermetabolic lesion in both lungs
E. Decrease FEV1

39. A 40-year-old male came to the outpatient clinic complaining right nasal blockage since 3 month ago. Rhinoscopy revealed
reddish fungating mass in right nasal cavity. Biopsy was performed and microscopic examination showed hyperplastic
squamous cell epithelium, more than 20 layers. Histopathologically this tumour is benign.
Which of the following condition is the most likely etiology for above tumour?

A. Allergy
B. EBV infection
C. HIV infection
D. HPV type 6 and 11 infection
E. HPV type 16 and 18 infection
40. A 65-year-old male, came to the clinic complaining of breathlessness on climbing stairs in his house and associated with
increase in severity with time. He also has productive thick whitish sputum. Spirometry examination revealed that
FEV1/FVC less than 70% and FEV1 47% than predicted.
What is the most likely diagnosis for this patient?

A. Chronic obstructive pulmonary disease stage 0


B. Chronic obstructive pulmonary disease stage I
C. Chronic obstructive pulmonary disease stage II
D. Chronic obstructive pulmonary disease stage III
E. Chronic obstructive pulmonary disease stage IV

41. Which of the following drug can cause decrease the ability visual acuity especially in children less than 5 years old ?

A. Etambutol
B. Isoniazid
C. Rifampin
D. Capreomycin
E. Pyrazinamid

42. The diagnosis features of asthma are the history of variable respiratory symptoms:

A. Wheeze, shortness of breath, chest tightness, cough


B. Wheeze, shortness of breath, chest tightness, rhinorhoe
C. Wheeze, shortness of breath, rhinorhoe, cough
D. Wheeze, rhinorrhoe, chest tightness, cough
E. Rhinorrhoe, cough, shortness of breath, chest tightness

43. The development of the nose started at the first trimester. Which of the following statement is the correct embryological
development?

A. Nasal bridge is developed from median nasal prominence


B. Alae is developed from lateral nasal prominence
C. Nasal tip is developed from lateral nasal prominence
D. Nasal crest is developed from lateral nasal prominence
E. Nasal placode is developed from mesoderm

44. In alveolar period the components of the mature blood-air barrier in the lung is established. Which of the following
embryonic pulmonary layers is derived component of the blood-air barrier in the lung?

A. Endoderm and ectoderm


B. Visceral mesoderm and ectoderm
C. Visceral mesoderm only
D. Visceral mesoderm and endoderm
E. Somatic mesoderm and Visceral mesoderm

45. A male newborn infant was choking and coughing suddenly after his first feeding. There was an excessive amount of mucus
secretion and saliva in the infant's mouth and the infant experienced considerable difficulty in breathing. The pediatrician
was unable to pass a catheter through the esophagus into the stomach.
Which of the following anomaly is often associated with this anomaly?
A. Achondroplasia
B. Amelia
C. Anencephaly
D. Cranial defects
E. Renal anomalies

46. Male patients, 25 years old, suffered shortness of breath since 5 days ago, two weeks before patients complained right chest
pain which increasing during deep inspiration. patients also complained of cough and low grade feversince 1 month ago.
Physical examination showed dull and decreases of breathing sound on intercostal space III - V right hemithorax.
What is the most appropriate next investigation to support diagnosis of above patient?

A. Microscopic FBA test


B. Chest x-ray
C. Rapid molecular test
D. Ultrasonography
E. Thorax CT Scan

47. A 50-year-old male came to Primary Health Care with dyspnea.He was frequently visiting PHC for the same problem in this
year. The doctor ask many question related to risk factor and the precipitation of his pulmonary problem.In the physical
examination, He found that the patient is thin, fever (39 degrees C), crackles and wheezing in both of lungs.
After assessment, we found that his body weight was 52 kg, while his body height was 170kg. We have to calculate his total
energy expenditure to increase his body weight. How many the basal energy expenditure of him?

A. 70 Cal per hour


B. 52 Cal per hour
C. 1,680 Cal per day
D. 1,248 Cal per day
E. 1,968 Cal per day

48. The increase pulmonary ventilation that occurs during exercise is called :

A. Tachypnea
B. Apnea
C. Hyperventilation
D. The Bohr effects
E. Hyperpnea

49. Which part of the respiratory tree is the structure that line with simple squamous epithelium?

A. Alveolar ducts
B. Alveoli
C. Alveolar sacs
D. Respiratory bronchioles
E. Bronchi

50. If the pO2 of the alveolar air is 104 mm Hg, what would the pO2 of the blood leaving the lungs be?

A. -9About the same as the pO2 of alveolar air because exchange of gas is a simple diffusion process
B. Lower than 104 mmHg because the high speed of blood flow through the lung makes not enough time to absorb all of
the alveolar O2
C. Lower than 104 mmHg because not all of the blood that enter the lung is oxygenated
D. Higher than 104 mmHg because can be absorbed using active transport mechanism
E. Can not be predicted because blood O2 level depends on many factors

51. A 17-month-old boy admitted to emergency ward with stridor. He has been playing with his sister while his mother cooked
in the kitchen.
What the most important question must be asked during taking a history?

A. Weight of the child


B. Antipyretic given
C. Foreign body aspiration
D. DPT vaccination
E. Nebulisation performed at home

52. A 39-year-old man is suffering from tuberculosis since 4 months ago. The doctor in duty gives combination of three
antituberculosis for him. Because of his ulkuspepticum, he took antacid to reduce his complaint.
Which of the following drug will undergo reduction in absorption if taken concurrently with antacid?

A. Isoniazid
B. Rifampin
C. Ethambutol
D. Capreomycin
E. Pyrazinamid

53. A 30-year-old male came to outpatient clinic complaining progressive shortness of breath and productive cough since 2 days
ago. This complains were accompanied by fever. After complete examination the doctor diagnose this case as pneumonia.
The xray showed consolidation of a large portion of left lower lung lobe.
Which of the following is most appropriate describing the stage of this disease?

A. Congestion, consolidation, red hepatization, grey hepatization


B. Congestion, red hepatization, grey hepatization, consolidation
C. Congestion, red hepatization, grey hepatization, resolution
D. Consolidation, congestion, red hepatization, grey hepatization
E. Consolidation, red hepatization, grey hepatization, resolution

54. An asthma sufferer finds she has to breathe at twice her normal rate.
How does that affected her dynamic compliance?

A. It stays the same


B. It decreases
C. It increases
D. Static compliance
E. None the answer above

55. A 10-year-old boy, 25 kg, came with runny nose with whitish discharge, cough and nasal blockage since 3 days before. The
doctor gave him pseudoephedrine, gliserilguaiakolat in combination with dextromethorphan.
What is the most appropriate reason that the combination of gliserilguaiakolat with dextromethorphan is not rationale?
A. Gliserilguaiakolat increases stiffness of mucus; dextromethorphanincreases airway mucus secretion
B. Gliserilguaiakolat increase the transportability of mucus; dextromethorphan inhibits airway mucus secretion
C. Gliserilguaiakolat stimulate surfactant secretion; dextromethorphan increases cough reflex
D. Gliserilguaiakolat increase the tenacity of secretions by increasing adhesively; dextromethorphan inhibits cough reflex
E. Gliserilguaiakolat increase airway mucus secretion by hydrating; dextromethorphan inhibits cough reflex

56. Which of the following is the most likely factor that stimulate the brain respiratory center?

A. High blood adrenalin level


B. High blood CO2 level
C. Low blood O2 level
D. Low blood pH
E. Low ATP level

57. Which of the following pathologic condition that most likely match with the above flow-volume loop?

A. Severe obstruction due to emphysema


B. Extrathoracic stenosis
C. Restrictive due to pulmonary fibrosis
D. Mild obstructive due to asthma
E. Intrathoracic stenosis

58. Twelve years old child with severe wheezing had laboratory result of IgELevel ; 597 IU/ml, the reference value of IgE Level
as shown below :

Which of the following mentions several etiology of Hyper-IgE?

A. The condition of pulmonary disease cause by thoracic trauma always with increasing the IgE level
B. The condition of respiratory alkalosis always combined with increasing the IgE Level
C. The condition of allergy, hay fever, asthma, anaphylaxisalways increasing the IgE Level
D. Respiratory disturbance cause of pneumonia always increasing the IgE Level
E. Respiratory disturbance cause primary by pulmonary embolism always increasing the IgE Level

59. Which of the following includes management of rhinosinusitis?

A. The usage of antibiotic should be rationally


B. Antibiotic management must be given to any rhinosinusitis generally empirical and based on result previous study
C. There is no indication to use intranasal steroid
D. Considered using oral steroid and oral antibiotic
E. Antihistamines should give in post viral acute rhinosinusitis

60. A 37-year-old male - heavy smoker - undergo spirometry examination for general check up. Sometimes he have shortness of
breath.
Which of the following spirometry result that most likely for this patient?

A. FEV1/FVC ratio normal, decrease FVC


B. FEV1/FVC ratio normal, normal FVC
C. FEV1/FVC ratio normal, increase FVC
D. FEV1/FVC ratio decrease, decrease FVC
E. FEV1/FVC ratio increase, increase FVC

61. A 6-month-old boy presents to emergency department with difficulty of breathing since yesterday. The symptom preceded
by fever and cough since 3 days before. The difficulty of breathing is not accompanied by wheezing or stridor. In physical
examination reveals fever, tachipnea, chest indrowing, and crackles at the auscultation of the lung. The chest X-ray revealed
infitrate at bilateral lung parenchymal.
What is the complication of the disease?

A. Pneumonia
B. Empyema
C. Chronic bronchitis
D. Lung cancer
E. COPD

62. Which of the options above points to C-shaped cartilage?

A. C
B. D
C. E
D. F
E. G

63. Which of the following best describes symptoms that accompanies obstructive pulmonary disease?
A. Fixation of the chest in a position larger than the normal end expiratory level
B. Loss of compliance of the lung
C. Stiffness costovertebral or sternocostal connection
D. Loss of muscle or loss of nerves that activate the muscle of respiration
E. Expanding the lung as the air distribution as the lung expands

64. A 40-year-old man, with dyspneu due to traffic accident one hour before with blunt chest trauma, Blood pressure 80/60
mmHg, Heart rate 120x/min and Respiratory rate 44x/min. At the thoracic region, asymmetric, hematom at the right side of
the sternum, vesicular Breath sound decreased and hypersonor on percussion with subconjunctiva bleeding on both of his
eyes.
Patient with tachypneu and cyanosis, what is the most appropriate action?

A. Chest tube thoracostomy insertion


B. Intubation
C. Blood gas analysis
D. Needle thoracostomy
E. Thoracostomy Exploration

65. A 65-year-old man came to ER with dyspnea and cough since 2 months ago. He had barrel chest and have smoking history
since teenager. Physical examination revealed increasing respiratory rate and expiratory wheezing.

Which of the following flow-volume loop spirogram that most likely for this patient?

A. A
B. B
C. C
D. D
E. E

66. A 25-year-old male perform spirometry examination and the result was like picture below :
Which of the following event that most likely caused above unacceptable trace?

A. Slow start
B. Stop early
C. Coughing
D. Extra breath
E. Poor effort

67. Select the most likely factor responsible for transferring O2 from a pregnant mother to her foetus.

A. Blood pH
B. Blood O2
C. Blood CO2
D. Blood 2,3-BPG
E. Blood Fe2+

68. This capacity is the sum result from expiratory reserve volume and residual volume :

A. FVC
B. IC
C. VC
D. FRC
E. EC
69. A 24-year-old man is admitted to the hospital after a street fight. Radiographic examination reveals an inferior (blow-out)
fracture of the orbit. Orbital structures would most likely be found inferiorly in which of the following spaces?

A. Ethmoidal sinus
B. Frontal sinus
C. Maxillary sinus
D. Nasal cavity
E. Sphenoidal sinus

70. A 50-year-old woman is admitted to the hospital with a swollen right calf. She has history of surgical cholecystectomy 10
days ago. Physical and diagnostic findings reveal a deep vein thrombosis. On the next day, she suffered a mild-to-moderate
dyspnea. No history of dyspnea or respiratory before. Chest X-ray shows clear lungs without infiltrates.
Which one is the principle of diagnostic tools for the working diagnosis?

A. Increase glucose uptake by malignant cells


B. Blood flow imaging of the vein and artery
C. Uptake of radiopharmaceutical by microorganism
D. Imaging of airway (inhalation) vs imaging of blood vessel (emboli)
E. Decrease air flow to the respiratory tract due to airway blockage

71. Which of the structure is one of the attachment site for vocal ligaments?

A. A
B. B
C. C
D. D
E. E

72. Which cell types are found in the respiratory mucosa?

A. Ciliated cells
B. Goblet cells
C. Basal cells
D. Brush cells
E. All of the above
73. Mention the characteristics of Haemophilus influenza from the following options:

A. Occurs in single, pairs, or chained Gram (+) coccus


B. Gram (-) pleomorphic rods
C. Only requires V factor to grow
D. Only requires X factor to grow
E. Has M-protein

74. Which of the following is the first branching of the bronchial tree that has gas exchanging capabilities?

A. Terminal bronchioles
B. Respiratory bronchioles
C. Alveoli
D. Alveolar duct
E. Segmental bronchi

75. Which of the structure is not a paired cartilage?

A. A
B. B
C. C
D. D
E. E

76. An 18-year-old man came to the doctor complaining poor performance in red-green color vision discrimination leading to
fail in getting drive license. On further history taking found he is taking a four-drug regimen for pulmonary tuberculosis.
What is the most likely drug he took?

A. Pyrazinamide
B. Ethambuthol
C. Isoniazid
D. Rifampin
E. Streptomycin
77. A 10-year-old girl admitted to pediatric clinic with a chief complaint of difficulty of breathing (dyspnea) since 7 days ago.
This complaint was accompanied by cough and high fever. In anteroposterior chest X-ray, there is a homogeneous lung
opacification with air bronchograms in the lateral segment of the lung.
The lateral segment of the lung is located in:

A. Upper lobe of the right lung


B. Upper lobe of the left lung
C. Middle lobe of the right lung
D. Lower lobe of the left lung
E. Lower lobe of the right lung

78. This capacity is the sum result from tidal volume, inspiratory reserve volume dan expiratory reserve volume :

A. FVC
B. IC
C. VC
D. FRC
E. EC

79. Which of the following changes is results of restrictive lung disorders?

A. Vital capacity
B. Forced expiratory volume
C. Forced inspiratory volume
D. Residual volume
E. Functional residual capacity

80. A 65-year-old male, came to the clinic complaining of breathlessness on climbing stairs in his house and associated with
increase in severity with time. He also has productive thick whitish sputum. Spirometry examination revealed that
FEV1/FVC less than 70% and FEV1 47% than predicted.
For the above patient, what is the most likely drug that will play a central role in the management?

A. Antibiotics
B. Corticosteroids
C. Bronchodilator
D. Cromones
E. Antihistamines
81. A 30-year-old man was diagnosed as having acute pharyngitis. Culture test showed that he was infected by Streptococcus
pneumonia. He had history of allergy to amoxicillin and cefadroxil.
What is the most appropriate antibiotic this patient and the rational explanation?

A. Ampicillin; the effective antibiotic against streptococci


B. Amoxicillin+clavulanate; It is a broad spectrum antibiotic that can prevents activity of bacterial beta-lactamase
C. Clarithromycin; it is effective against gram positive organism and has different immunologic characteristic with beta-
lactam antibiotic
D. Streptomycin; it is effective against both gram and can be given by parenteral
E. Metronidazol; it is effective against gram negative

82. Choose one of the most appropriate option from the statement below:
The collection of lymphoid tissue in the submucosa of the pharynx near the nasopharyngeal opening.

A. Lingual tonsil
B. Palatine tonsil
C. Pharyngeal tonsil
D. Tubal tonsil

83. What type of the tissue that forms the smallest airspace of the respiratory tree?

A. Stratified squamous epithelium


B. Simple columnar epithelium
C. Pseudostratified epithelium
D. Simple cuboidal epithelium
E. Simple squamous epithelium

84. The most likely membrane channel that is responsible in transporting CO2 :

A. Calcium channel
B. K+, Na+- ATPase channel
C. Anion exchange channel
D. Sodium-glucose channel
E. No membrane channel is needed

85. A woman, 25-year old, came to PUSKESMAS with hemaptoe. She has no history of tuberculosis. Her husband has been
treated for lung tuberculosis since 2 months ago. She has two children, who are a baby boy 3 months-old and a 4 year old girl.
Physical examination revealed:
BP: 130/80mmHg; T: 38oC; RR: 32x/mnt; HR: 105 bpm; lymphadenopathy of deep cervical; ronchi on apex chest, no other
organ abnormality.
Acid fast staining shows positive contains M.tb at 2 of 3 sputum samples.
If the patient had renal insufficiency and the doctor did not adjust the regimen dosage, which one is the most possible symptom
that could happen?

A. Nefropathy because of isoniazid


B. Hyperuricemia because of streptomysin
C. Hepatotoxicity because of rifampicin
D. Retrobulbar neuritis because of ethambutol
E. Peripheral neuropathy because of pyrazinamide accumulation

86. A 24-year old female with severe scoliosis undergo spirometry examination due to vertebra surgery preparation.
Which of the following spirometry result pattern that most likely for this patient?

A. Normal pattern
B. Restrictive pattern
C. Obstructive pattern
D. Mixed pattern
E. Normal pattern after given bronchodilator

87. A 30-year-old male came to outpatient clinic complaining progressive shortness of breath and productive cough since 2 days
ago. This complains were accompanied by fever. After complete examination the doctor diagnose this case as pneumonia. The
xray showed consolidation of a large portion of left lower lung lobe.
Which of the following is the most appropriate bacteria related to community acquired atypical pneumonia?

A. Haemophilus influenza
B. Legionella pneumophila
C. Moraxella catharralis
D. Mycoplasma pneumonia
E. Streptococcus pneumonia

88. A 54-year-old man is admitted to the hospital with severe pain in his nasal cavity. Radiographic examination reveals a
carcinoma in his nasal cavity. In which of the following locations would the carcinoma block the hiatus of the posterior
ethmoidal sinus?
A. Inferior meatus
B. Middle meatus
C. Superior meatus
D. Nasopharynx
E. Sphenoethmoidal recess

89. A man was brought to the Emergency Unit. The doctor did the physical examination, and declared that he was dead. On
examination, a horizontal ligature mark was seen encircling the neck, presence of conjunctival petechial haemorrhages, and
congestion of the face. The body was then sent to Forensic Medicine Department to undergo autopsy. Autopsy findings
revealed Tardieu's spot, cerebral hypoxia, and fracture of the superior horn of the thyroid cartilage.
Select the most likely autopsy finding which indicates carotid obstruction due to pressure on the neck.

A. Cerebral hypoxia
B. Congestion of the face
C. Conjunctival petechial haemorhages
D. Fracture of superior horn of the thyroid cartilage
E. Tardieu’s spot

90. Which of the following best describes Postural drainage?

A. Turning the patient from side to side hourly while the patient immobilized in bed
B. Teaching the patient to cough
C. Exercising the patient to make the trunk flexible
D. The positionthat can facilitate drainage the secrete from the lung by gravity
E. Chest percussion and vibration

91. Female patients, 35 years old, suffered cough with thick sputum since 1 month ago. Patients also complained of low grade
fever, night sweats and decreased appetite. Two times Acid Fast Bacilli (AFB) test shows: 2+/1+
Past history: 6 months ago the patient had received TB therapy for 3 weeks then did not continue the treatment.
What is the diagnosis of this patient now?

A. New case TB
B. Default case TB
C. Relaps case TB
D. Chronic TB
E. Resistant TB

92. Any lung disease that reduces the forced expiratory volume (FEV) but does not significantly affect the vital capacity is referred
as :

A. Restrictive lung disorder


B. Pulmonary fibrosis
C. Chronic pulmonary disease
D. Corpulmonale
E. Obstructive lung disorders

93. Which of the following biomolecules which is most likely unable to function normally in the case of abnormal blood pH that
can lead to death?

A. Lipid
B. Fibre
C. Protein
D. Nucleic acid
E. Carbohydrate

94. A 35-year-old woman came to your clinic with productive cough and occasionally bloody sputum as chief complains. She
also had night sweat and body weakness, lose of appetite and mild fever for the last 3 weeks. The doctor in duty prescribes
antimycobacterial for her.
Which of the following drug can results in an increased concentration of urate in the blood ?

A. Isoniazid
B. Rifampin
C. Etambutol
D. Ciprofloxacin
E. Streptomycin

95. Which of dental problems below most probably cause sinusitis?

A. Dental extraction complications such as oroanthral fistulae


B. Direct traumatic into sinus
C. Swimming or diving in contaminated pool
D. Iatrogenic problems such as using oral gastric tube
E. Dental abses from Molar 3 (M3)

96. What type of tissue makes up the epiglottis?

A. Compact bone
B. Spongy bone
C. Hyaline cartilage
D. Fibrocartilage
E. Elastic cartilage

97. A 1-year-old infant is admitted to the hospital with fever. Radiographic examination reveals a sinus infection. Which of the
following sinuses is present at this age?

A. Frontal sinus
B. Maxillary sinus
C. Sphenoid sinus
D. Middle ethmoidal air cells
E. Posterior ethmoidal air cells

98. The sphenoidal sinuses are located in the body of the sphenoid, they may extend into the wings of this bone. Because of this
extensive pneumatization (formation of air cells), the body of the sphenoid is fragile. Only thin plates of bone separate the
sinuses from which important structures?

A. Facial nerves
B. External carotid arteries
C. Parotid gland
D. Optic chiasm
E. Ethmoidal recess

99. A result of spirometry from 65 year old male with acute exarcebation of COPD came back with restrictive pattern.
Which of the following description that most appropriate with the patient's spirometry pattern?

A. Slow rise to reduce maximum volue


B. Fast rise to plateau at reduced maximum volume
C. Slow rise, reduce volume expired
D. Fast rise to reduce maximum volume
E. Fast rise, increase volume expired

100. A 6-month-old boy presents to emergency department with difficulty of breathing since yesterday. The symptom preceded by
fever and cough since 3 days before. The difficulty of breathing is not accompanied by wheezing or stridor. In physical
examination reveals fever, tachipnea, chest indrowing, and crackles at the auscultation of the lung.
What is the diagnosis of the patient?

A. Laryngotracheobronchitis
B. Bronchitis
C. Bronchiolitis
D. Pneumonia
E. Dipthteri

101. Which cell is found in large numbers in the terminal bronchioles?

A. Clara cell
B. Type I pneumocyte
C. Type II pneumocyte
D. Dust cell
E. Brush cell
102. A fourty five-old-year man, diagnosed as Pneumonia. Blood gases analysis was shown as below :

What is your interpretation of blood gases analysis from this patient ?

A. Respiratory acidosis with partial compesation, hyperoxemia and normal O2 saturation


B. Respiratory acidosis with complete compesation, hyperoxemia and normal O2 saturation
C. Respiratory acidosis with complete compesation, hypoxemia and normal O2 saturation
D. Respiratory alkalosis without compesation, hypoxemia and normal O2 saturation
E. Respiratory alkalosis with complete compesation, hyperoxemia and normal O2 saturation

103. A 12-year-old girl with a childhood history of asthma complained of cough, dyspnea, and wheezing. Her symptoms became
severe that her parent brought her to the emergency room. Physical examination revealed diaphoresis, dyspnea, tachycardia,
and tachypnea. Her respiratory rate was 42/min, pulse rate was 110 beats per minute, and blood pressure was 132/65 mmHg.
A drug likely to be ineffective in this patient :

A. Inhaled cromolyn
B. Oral or IV methylprednisolone
C. Inhaled ipratropium
D. Inhaled albuterol
E. Intravenous propranolol

104. A 55-year-old male came to outpatient clinic complaining epistaxis since 1 month ago. Nasopharyngoscopy revealed an
ulcerative fleshy mass in nasopharynx. Biopsy was performed and the histopathological examination revealed nasopharyngeal
cancer.
Which of the following is the most common histopathological type of nasopharyngeal cancer?

A. Adenocarcinoma
B. Keratinizing squamous cell carcinoma
C. Large cell carcinoma
D. Small cell carcinoma
E. Undifferentiated carcinoma

105. A 45-year-old man with a complaint of ear pain and difficulty hearing is diagnosed with tonsillitis. Otoscopic examination
reveals fluid in the middle ear cavity. Hypertrophy of which of the following structures would be most likely to compromise
the drainage of the auditory tube?
A. Lingual tonsil
B. Palatine tonsil
C. Tubal tonsil
D. Superior constrictor muscle
E. Uvula

106. The nasal conchae curve inferomedially, hanging like louvers or short curtains from the lateral wall. Which of the following
statement related to the superior nasal conchae?

A. Superior nasal conchae is the longest and broadest of the conchae


B. Superior nasal conchae is a narrow passage between the superior and the middle nasal conchae
C. Superior nasal conchae covered by a mucous membrane that contains large vascular spaces that can enlarge to control
the caliber of the nasal cavity
D. Superior nasal conchaeis medial processes of the ethmoid bone
E. The anterosuperior part of this conchae leads into a funnel-shaped opening, the ethmoidal infundibulum.

107. On a frontal chest film, there is the lung apex retracts towards the hilum, the sharp white line of the visceral pleura is visible,
separated from the chest wall by a radiolucent pleural space, which is devoid of lung markings:

A. Pneumothorax
B. Giant bulla
C. Pulmonary emphysema
D. Giant emphysema
E. Pleural effusion

108. Which of the following sign is the most specific feature for Pneumonia cases?

A. Lobar consolidation
B. Homogeneous consolidation
C. Lobular pneumonia
D. Patchy consolidation
E. A localized infection of terminal and respiratory bronchioles

109. The arterial supply of the medial and lateral walls of the nasal cavity is from five sources. Which of the following does NOT
supply the medial and lateral walls of the nasal cavity?

A. Sphenopalatine artery
B. Anterior and posterior ethmoidal arteries
C. Greater palatine artery
D. Infraorbital artery
E. Superior labial artery

110. What exercise is recommended in asthma?

A. Strengthening upper extremities


B. Strengthening intercostal muscle
C. Aerobic exercise
D. Lower extremities exercise
E. Chest expansion breathing exercise
111. The 56-year-old male came for general check up. His BMI was 38. Sometimes he felt exhausted and slightly difficulty of
breathing. He was undergospirometry examination.
Which of the following spirometry result that most likely for this patient?

A. Decrease FEV1 and FVC


B. Increase FEV1 and FVC
C. Increase FEV1, decrease FVC
D. Decrease FEV1, increase FVC
E. Increase FEV1/FVC ratio

112. To get better visualization and result in rhinosinusitis, which of the procedure we could use?

A. CT scan, especially axial position coronal


B. MRI
C. Radiologic special position, such as :occipitomental view
D. Nasal endoscopy
E. Transillumination

113. A 67-year-old male, came to theEmergency Unit complaining of productive cough, fever and shortness of breath for 5 days.
On physical examination, crackles were heard at the right middlehemithorax. Chest x ray revealed infiltrates in the right middle
lobe.
What is the most likely diagnosis for this patient?

A. Acute bronchitis
B. Pneumonia
C. COPD
D. Tuberculosis
E. Bronchiectasis

114. Which of the following concerning average lung volumes and capacities of a person at rest is TRUE?

A. TLC>VC>TV>FRC
B. TLC>FRC>VC>TV
C. TLC>VC>FRC>TV
D. TLC>FRC>TV>VC
E. FRC>TV>VC>TLC

115. Which of the structure points to laryngeal inlet?


A. C
B. D
C. E
D. F
E. G

116. What type of epithelium lines of below tissue section?

A. Simple squamous epithelium


B. Simple cuboidal epithelium
C. Simple columnar epithelium
D. Stratified squamous epithelium
E. Pseudostratified squamous epithelium

117. What would happen if the ostium of sinus blocked?


A. Mucosal will be thinner, creating further blockage
B. Cilia and epithelium could work properly
C. Secretion stagnate
D. Bacterial could not growth because lack of oxygen
E. Decreased tissue inflammation

118. A 30-year-old female patient diagnosed to have tuberculosis and was started on treatment for 2 months. Currently she
complained of numbness of both hand.
What is the most likely anti tuberculosis drug which cause the neuropathy?

A. Ethambutol
B. Pyrazinamide
C. Rifampicin
D. Isoniazide
E. Streptomycine

119. To see the causes of apical dental problem became sinusitis, which of the procedure we could use?

A. Plain sinus X-ray such as Schuller position


B. Plain X-ray : panoramic view
C. Plain X-ray :Schedel lateral
D. Bimanual palpation
E. Anterior rhinoscopy and nasal endoscopy

120. Which one of the following clinical condition is contraindicated of salbutamol ?

A. Epilepsy
B. Hypertension
C. Peptic ulcers
D. Diabetes mellitus
E. Diarrhea

121. The total minute volume of the lungs is obtained by multiplying the at rest by the number of breaths per minute is :

a. Vital capacity
b. Total lung capacity
c. Inspiratory capacity
d. Tidal volume
e. Dead space volume

122. Choose one of the most appropriate option from the statement below:
The tonsils are collections of lymphoid tissue on each side of the oropharynx.

a. Lingual tonsil
b. Palatine tonsil
c. Pharyngeal tonsil
d. Tubal tonsil

123. Anisa 9-month-old girl patient present to pediatric emergency room with the difficulty of breathing and suffered from fever
since the last 3 days which has proceeded by common cold sincw 4 days before.
After taking history, physical examination and preliminary investigation, she is diagnosed with atypical Pneumonia and
treated by Macrolide drugs.
Which of the following is the mechanism of action of macrolide drugs?

a. Inhibit protein synthesis by binding DNA 50 S


b. Inhibit protein synthesis by binding RNA 50 S
c. Inhibit protein synthesis by binding DNA 80 S
d. Inhibit protein synthesis by binding RNA 80 S
e. Inhibit protein synthesis by binding DNA & RNA 80 S

124. Which of the following is the proper breathing exercise for the obstructive pulmonary disorder?

a. Chest expansion and shouldergirdle exercise


b. Force expiration without external positive pressure
c. Strengthening inspiratory exercise and expansion of the chest
d. Force and deep breathing exercise
e. Pursed-lips breathing and diaphragmatic breathing exercise

125. Which numbers are structures that separated by the oblique fissure?

a. 1 and 2
b. 2 and 3
c. 6 and 7
d. 7 and 8
e. 8 and 10

126. Patient admitted after the traffic accident with RR 28-32x/min, BP 90/70mmHg, jugular vein distention, muffle heart sound,
trachea at the mid line of his neck.
What is the most appropriate diagnosis?

a. Tension pneumothorax
b. Cardiac tamponade
c. Heart contusion
d. Massive hemothorax
e. Lung Injury

127. A 6-month-old boy presents to emergency department with difficulty of breathing since yesterday. The symptom preceded
by fever and cough since 3 days before. The difficulty of breathing is not accompanied by wheezing or stridor. In physical
examination reveals fever, tachipnea, chest indrowing, and crackles at the auscultation of the lung.
What is definition of tachypnea according of the age of the patient?

a. Respiratory rate ≥ 70 x/minute


b. Respiratory rate ≥ 60 x/minute
c. Respiratory rate ≥ 50 x/minute
d. Respiratory rate ≥ 40 x/minute
e. Respiratory rate ≥ 30 x/minute

128. What are the symptoms of rhinosinusitis?

a. Epistaxis, rhinorrhea, hyposmia/anosmia


b. Dental pain, nasal blockage, gustational problem
c. Pain at the neck, dental pain, forehead pain
d. Hyposmia/anosmia, nasal blockage, facial pain
e. Hyposmia/anosmia, gustational problem, epistaxis

129. Patient with penetrating injury at the anterior right sided of his chest R 36-40x/min with 2-3cm the diameter of the open
wound.
BP 90/70mmHg. How to manage this patient?

a. Ask the chest X-ray examination due to tension pneumothorax


b. Close the open wound with sterile drape with 3 side tape due to sucking chest wound
c. Give the crystalloid fluid as soon as possible to maintain blood pressure
d. Intubation to manage his breathing
e. Chest tube thoracostomy

130. A 40-year-old male came to the outpatient clinic complaining right nasal blockage since 3 month ago. Rhinoscopy revealed
reddish fungating mass in right nasal cavity. Biopsy was performed and microscopic examination showed hyperplastic
squamous cell epithelium, more than 20 layers. Histopathologically this tumour is benign.
Which of the following tumor is most appropriate for this case?

a. Adenoma
b. Carcinoma
c. Papilloma
d. Polip
e. Rhinitis

131. A 23-year-old woman, presented with productive cough and fever for 7 days. Physical examination showed increase in
tactile fremitus, bronchial breath sound, whispering pectoriloquy and crackles on the right lower hemithorax.
What is the most likely clinical diagnosis for this patient?

a. Pleural effusion
b. Pneumonia
c. Pneumothorax
d. Pleural fibrosis
e. Lung emphysema

132. A 6-month-old boy presents to emergency department with difficulty of breathing since yesterday. The symptom preceded
by fever and cough since 3 days before. The difficulty of breathing is not accompanied by wheezing or stridor. In physical
examination reveals fever, tachipnea, chest indrowing, and crackles at the auscultation of the lung.
What is the intravena antibiotic choice for the patient?

a. Ampicillin 100-200 mg/BW/day


b. Gentamycin 7.5 mg/BW/day single dose
c. Chloramphenicol 50-100 mg/BW/day
d. Ampicillin 100-200 mg/BW/day and Gentamycin 7.5 mg/BW/day single dose
e. Ampicillin 100-200 mg/BW/day and Chloramphenicol 50-100 mg/BW/day

133. A 10-year-old boy, 25 kg, came with runny nose with whitish discharge, cough and nasal blockage since 3 days before. The
doctor gave him pseudoephedrine, gliseril guaiakolat in combination with dextromethorphan.
Which is the most appropriate role of pseudoephedrine in this patient?

a. Increasing nasal mucus production mediated by α1 receptors


b. Antagonist of histamine
c. Inhibit of nasal mucus production by interfering the parasympathetic system
d. Interfere the nasal mucus production mediated by α2 receptors
e. Decreasing nasal mucus production mediated by α1 receptors

134. Mention the general characteristics of Streptococcus pyogenes from the following options:

a. Gram (-) pleomorphic rods


b. Requires V and X factors
c. Non-capsulated
d. Has Ig-A protease
e. Occurs in single, pairs, or chained Gram (+) coccus
135. A 62-year-old man was being managed in the intensive care unit following a large anterior wall myocardial infarction. He
had been appropriately managed with oxygen, aspirin, nitrates, and β-adrenergic receptor blockers but developed recurrent
episodes of ventricular tachycardia. During these episodes he remained conscious but feeling dizzy. He was also diaphoretic
and hypotensive.
What is the most appropriate note that must be written in prescription for indicating "the prescription has to be priority"?

a. ITER
b. PRO INJ.
c. EMERGENCY
d. CITO
e. PRO RE NATA

136. A 13-month-old boy was brought to your clinic with noisy breathing for the first time which suddenly occurred last night
disturbing his sleep
What is the mostly working diagnosis for the patient?

a. Spasmodic croup
b. Viral laringotracheobronchitis
c. Epiglottitis
d. Bacterial tracheitis
e. Asthma bronchiale

137. A 55-year-old male came to outpatient clinic complaining epistaxis since 1 month ago. Nasopharyngoscopy revealed an
ulcerative fleshy mass in nasopharynx. Biopsy was performed and the histopathological examination revealed
nasopharyngeal cancer.
Which of the following viral gene expression that most related to pathogenesis of above tumour?

a. EBER
b. EBNA 1
c. P16
d. P21
e. P53

138. PO2 can be calculated with the equation below....

a. PIO2-(VO2/VA)863 mmHg
b. PICO2-(VCO2/VA)863 mmHg
c. (VO2/VA)863 mmHg
d. (VCO2/VA)863 mmHg
e. None of the above

139. A 20-year-old woman came to the doctor with dyspnea and fever. She has been had this condition for the last 2 days and
now its became worsen. She is noted as betalactam hypersensitive patient.
On physical examination found respiration rate 36X/minutes, temperature 38.6°C, epigastric and intercostal retractions;
vesicular breathing sound and crackles was found by auscultation .
Laboratory result showed WBC 18.000/mm. Blood culture : H. influenzae.
Which is the most effective antibiotic ?

a. Erythromycin
b. Clarithromicin
c. Azithromycin
d. Clindamycin
e. Spectinomycin

140. A sixty seven years old man presents to the Emergency Departement with cronic pulmonary obstruction. The Blood Gases
Analysis shown as below :
What is your interpretation of blood gases analysis from this patient?

a. Respiratory acidosis with partial compesation, hyperoxemia and normal O2 saturation


b. Respiratory acidosis with complete compesation, hyperoxemia and normal O2saturation
c. Respiratory acidosis with complete compesation, hypoxemia and normal O2saturation
d. Respiratory alkalosis with partial compesation, hyperoxemia and normal O2 saturation
e. Respiratory alkalosis with complete compesation, hyperoxemia and normal O2 saturation

141. What is the first portion of the respiratory tree where gas exchange occur?

a. Alveolar ducts
b. Alveoli
c. Alveolar sacs
d. Respiratory bronchioles
e. Bronchi

142. Mr. Cepi admitted to hospital due to shortness of breath.


Blood gas analysis result:
pH : 7.20,
PaCO2 : 60 mmHg,
PaO2 : 70 mmHg,
HCO3- :24 mEq/L.
What is your interpretation?

a. Respiratory alkalosis
b. Respiratory acidosis
c. Respiratory acidosis compensated
d. Metabolic acidosis
e. Metabolic alkalosis

143. Which of the structure points to cuneiform cartilageF?

a. A
b. B
c. C
d. D
e. E

144. A 50-year-old male came to Primary Health Care with dyspnea.He was frequently visiting PHC for the same problem in this
year. The doctor ask many question related to risk factor and the precipitation of his pulmonary problem.In the physical
examination, He found that the patient is thin, fever (39 degrees C), crackles and wheezing in both of lungs.
What considerations of nutrient will be very important for him in this condition?

a. Glucose, protein
b. Protein, carbohydrates, fat, vitamins, water
c. Energy from glucose, fatty acids, muscle glycogen, minerals
d. Oil from both saturated and unsaturated fat
e. Animal protein and fibers

145. What is considered as emergency in rhinosinusitis?

a. Serious orbital complication occur


b. Dental problem occur
c. Mild to moderate pain
d. Involving the cheek
e. Develop chronic cough especially in children.

146. In the prolonged exercise, lactic acid begins to accumulate in the muscle.
When the lactic acid starts accumulate in muscle?

a. Maximum oxygen uptake


b. Lactate treshold
c. Maximum respiratory rate
d. Vital capacity
e. Total lung capacity

147. What is the property of S. pneumoniae which enables it to avoid phagocytosis?


a. Endotoxins
b. Lipoarabinomannan
c. Mycolic acid of the cell wall
d. Capsule
e. UreC gene

148. Top lordotic projection is especially useful for demonstration of the :

a. Lung apices
b. Upper lung
c. Middle lung
d. Lower lung
e. Lingula of the left lung

149. If her vital capacity is 4000 ml and her inspirational reserve volume is 2650.
How much is her expirational reserve volume?

a. 350 ml
b. 650 ml 850?
c. 1000 ml
d. 1350 ml
e. 1650 ml

150. Which cell is an alveolar squamous pulmonary epithelial cell?

a. Cell “A”
b. Cell “B”
c. Cell “C”
d. Cell “D”
e. Cell “E”

151. What is the reason of term rhinosinusitis more appropriate than sinusitis?

a. Viral infection is the initial causes


b. Inflammation will be continue to nasopharyngeal area
c. Not so appropriate term if the causes are from dental problems
d. Appropriate with oncology term, sinonasal neoplasm
e. Not all disease will continuing from the nose to the sinus, for example in dental problems

152. What is the property of S. pyogenes which enables it to avoid phagocytosis?


a. M-protein
b. Hyaluronic acid capsule
c. Erythrogenic toxin
d. Hemolysins
e. Streptokinase

153. Which cell that has function to reduce surface tension and allows for easier gas exchange?

a. Cell “A”
b. Cell “B”
c. Cell “C”
d. Cell “D”
e. Cell “E”

154. A woman, 25-year old, came to PUSKESMAS with hemaptoe. She has no history of tuberculosis. Her husband has been
treated for lung tuberculosis since 2 months ago. She has two children, who are a baby boy 3 months-old and a 4 year old
girl.
Physical examination revealed:
BP: 130/80mmHg; T: 38oC; RR: 32x/mnt; HR: 105 bpm; lymphadenopathy of deep cervical; ronchi on apex chest, no other
organ abnormality.
Acid fast staining shows positive contains M.tb at 2 of 3 sputum samples.
What is the most appropriate planning for the baby?

a. Give BCG vaccination, stop breastfeeding and keep the baby away from the mother until 2-week TB treatment
b. Give BCG vaccination, continue breastfeeding and give preventive INH after TB is ruled out
c. Give BCG vaccination, continue breastfeeding
d. Give preventive INH for the baby after TB is ruled out and continue breastfeeding
e. Give full TB treatment and continue breastfeeding

155. What type of lung cancer fits this description?


"The tumour cells originate from neuroendocrince (Kulchitsky) cells. It is associated with smoking and is centrally located
along the bronchial airways. Tumours tend to be at a high stage at the time of diagnosis. It is treated with chemotherapy as it
is usually not amenable to surgery and spead easily through vessels".

a. Adenocarcinoma
b. Squamous cell carcinoma
c. Small cell carcinoma
d. Large cell carcinoma
e. Sarcomatioid carcinoma

156. A 25-year-old man came to outpatient clinic with nasal congestion since 1 month earlier. Physical examination revealed
bilateral gelatinous mass in nasal cavity. No ulceration were seen. Clinically, this mass is benign.
Which of the following condition is the most likely etiology for above tumour?
a. Allergy
b. EBV infection
c. HIV infection
d. HPV type 6 and 11 infection
e. HPV type 16 and 18 infection

157. Female patients, 35 years old, suffered cough with thick sputum since 1 month ago. Patients also complained of low grade
fever, night sweats and decreased appetite. Two times Acid Fast Bacilli (AFB) test shows: 2+/1+
Past history: 6 months ago the patient had received TB therapy for 3 weeks then did not continue the treatment.
What is the most appropriate TB treatment regimen for that patient?

a. 2RHZES/RHZE/3RHE
b. 2RHZES/5 RHZ
c. 2RHEZ/4RH
d. 2RHEZ/4RHE
e. 2RHEZS/RHZE/5RH

158. Select the most likely factor that is decreased and responsible to cause tachypnoea in a myocardial infarction patient :

a. Blood pH
b. Blood O2
c. Blood CO2
d. Blood 2,3-BPG
e. Blood Fe2+

159. A 50-year-old male came to Primary Health Care with dyspnea.He was frequently visiting PHC for the same problem in this
year. The doctor ask many question related to risk factor and the precipitation of his pulmonary problem.In the physical
examination, He found that the patient is thin, fever (39 degrees C), crackles and wheezing in both of lungs.
Nutrition will support the body in this lung capability restriction by decreasing the production of carbon dioxide, by way of:

a. Increasing fat, decreasing carbohydrate


b. Increasing protein, decreasing fat
c. Decreasing sugar, increasing protein
d. Decreasing cholesterol, increasing vitamins
e. Increasing carbohydrate, decreasing fat

160. Lung morphogenesis can be subdivided into distinct periods on the basis of the morphological characteristics of the tissue.
This period is occur from 26 to 36 weeks of gestation and characteristic by increased thinning of the respiratory epithelium
and pulmonary mesenchym, further growth of lung acini, and development of the distal capillary network.
Which of the following period is appropriate to be above?

a. Sacular period
b. Canalicular period
c. Alveolar period
d. Embryonic period
e. Pseudoglobular period

161. A 33-year-old male presented in your clinic with productive cough for 4 weeks, low-grade fever, and weight loss. This patient
smokes 1 pack of cigarettes every day. X-ray revealed patchy lesions in both upper lung.
This patient smokes every day. Mention another risk factor for this disease?

a. Eating greasy food


b. Congenital heart disease
c. Coronary arterial disease
d. Human Immunodeficiency Virus
e. Arrhythmia

162. A 50-year-old male came to Primary Health Care with dyspnea.He was frequently visiting PHC for the same problem in this
year. The doctor ask many question related to risk factor and the precipitation of his pulmonary problem.
In the physical examination, He found that the patient is thin, fever (39 oC), crackles and wheezing in both of lungs. What is
the metabolism may occurred and causing his nutritional status?

a. Because of the hypoxia, then the hypometabolism occurred


b. Nicotin, tar, and many poisonous agent in smoke build unhealthy fat in his body
c. He has difficulty to chew foods
d. This chronic lung disease increase oxidation of his muscle mass
e. His high temperature burn his fat, make him thin

163. Which of the following antituberculosis is associated with red-orange discoloration of saliva, tears and urine ?

a) Ethionamide
b) Para-aminosalicylic Acid
c) Isoniazid
d) Rifampicin
e) Capreomycin

164. A woman, 25-year old, came to PUSKESMAS with hemaptoe. She has no history of tuberculosis. Her husband has been
treated for lung tuberculosis since 2 months ago. She has two children, who are a baby boy 3 months-old and a 4 year old girl.
Physical examination revealed
BP: 130/80mmHg; T: 38oC; RR: 32x/mnt; HR: 105 bpm; lymphadenopathy of deep cervical; ronchi on apex chest, no other
organ abnormality.
Acid fast staining shows positive contains M.tb at 2 of 3 sputum samples.
What is the most appropriate planning for the mother?

a) Start new regimen: 2HRZES/HRZE/5H3R3E3


b) Start new regimen: 2HRZE/4H3R3E3
c) Start new regimen: 2HRZE/4H3R3
d) Stop breastfeeding and start new regimen: 2HRZES/HRZE/5H3R3E3
e) Stop breastfeeding and start new regimen: 2HRZES/4H3R3

165. Mr. Beni, 28 years old, is training for marathon. His alveolar ventilation rate is 3.0 Lt/min and he is consuming O2 at the rate
300 ml/lt. If RQ=1, and constant atmosphere pressure=863 mmHg.
What is the value of Mr. Beni PACO2?

a) 863 mmHg
b) 86.3 mmHg
c) 8.63 mmHg
d) 0.863 mmHg
e) None of the above

166. Male patients, 25 years old, suffered shortness of breath since 5 days ago, two weeks before patients complained right chest
pain which increasing during deep inspiration. patients also complained of cough and low grade feversince 1 month ago.
Physical examination showed dull and decreases of breathing sound on intercostal space III - V right hemithorax.
What is the most likely clinical diagnosis of this patient?

a) Pulmonary TB
b) Pneumothorax
c) Pneumonia
d) Pleural effusion
e) Lung cancer

167. Which of the following occurrence leads to consideration of rhinosinusitis surgery?

a) Removal small polyp especially causing by allergy


b) Mucociliary clearance seem working properly
c) The ostia seem opening widely
d) Blockage in osteomeatal complex
e) Mild to moderate pain at the cheek

168. A 64-year-old man came to a clinic for follow-up of his chronic congestive heart failure. He has a marked reduction in his
ejection fraction following a series of myocardial infarctions. His symptoms include dyspnea on exertion, paroxysmal
nocturnal dyspnea, orthopnea, and peripheral edema. He has normal renal function. He is on treatment with twice daily of
ACE inhibitor and once daily of furosemide. You decide to add digoxin to his regimen.
Which is the most appropriate action of furosemide in this patient?

a) It reduces ventricular preload


b) It reduces peripheral resistance
c) It reduces afterload
d) It increases cardiac contractility
e) It reduces heart rate

169. This capacity is the sum result from tidal volume and inspiratory reserve volume

a) FVC
b) IC
c) VC
d) FRC
e) EC

170. What is the most possible condition that could happen in patient with severe asthma?

a) Decrease bicarbonate ion


b) Increase bicarbonate ion
c) Increase blood PH
d) Increase white blood cells
e) Increase hemoglobin level

171. Which cell is a respiratory macrophage?


a) Cell “A”
b) Cell “B”
c) Cell “C”
d) Cell “D”
e) Cell “E”

172. The symptoms below IS NOT criteria for making the diagnosis of asthma:

a) Occur variably over time


b) Occur vary in intensity
c) Often worse at the day
d) Often triggered by allergens
e) Often appear or worsen with viral infections

173. If the plane depressurized, what would be the value of your PAO2?

a) 78.9 mmHg
b) 25.8 mmHg
c) 54 mmHg
d) 20.5 mmHg
e) None of the answer above

174. An 11-month-old baby boy without underlying congenital abnormality presented to the Emergency Room with a history of
fever for one day associated with runny nose, noisy breathing, barking cough, dyspnea, hoarseness of voice and irritability.
His respiratory rate was 60 breaths per minute and oxygen saturation of 99%. There were also presence of inspiratory stridor,
subcostal recession and nasal flaring. He was treated with nebulized adrenaline 0.5 mg, iv hydrocortisone 50 mg and oxygen
via nasal prong.
What is the most likely agent causing the disease?

a) Streptococcus pyogenes
b) Streptococcus viridans
c) Haemophilus influenza
d) Parainfluenza virus
e) Avian influenza virus

175. Which of the following does NOT happen during inspiration?


a) The ribs move upward
b) The diaphragm lifts up
c) The antero-posterior dimensions of the chest are increased
d) The tranverse dimensions of the thorax are increased
e) The scalene and sternocleidomastoid muscles can be recruited for inspiration

176. A 55-year-old male came to outpatient clinic complaining epistaxis since 1 month ago. Nasopharyngoscopy revealed an
ulcerative fleshy mass in nasopharynx. Biopsy was performed and the histopathological examination revealed nasopharyngeal
cancer.
Which of the following virus is most related to the above tumor?

a) EBV
b) CMV
c) HIV
d) HPV type 6 and 11
e) HPV type 16 and 18

177. A male newborn infant was choking and coughing suddenly after his first feeding. There was an excessive amount of mucus
secretion and saliva in the infant's mouth and the infant experienced considerable difficulty in breathing. The pediatrician was
unable to pass a catheter through the esophagus into the stomach.
Which of the following is the embryological basis that causes this anomaly?

a. Failure of recanalization of the larynx


b. Incomplete of recanalization of the larynx
c. Narrowing and obstruction of the trachea
d. Abnormal partitioning of the trachea and esophagus by the tracheoesophageal septum
e. Larynx and upper trachea fail to separate completely from the esophagus during the first trimester

178. Anisa 9-month-old girl patient present to pediatric emergency room with the difficulty of breathing and suffered from fever
since the last 3 days which has proceeded by common cold sincw 4 days before.
After taking history, physical examination and preliminary investigation, she is diagnosed with atypical Pneumonia and treated
by Macrolide drugs.
If the patient is also taken theophylline, which of the following is a drug of choice to avoid drug interaction between macrolides
and theophylline?

a) Erythromycin
b) Clarithromysin
c) Azithromycin
d) Levofloxacin
e) Clindamycin

179. Frontal sinus are air-filled extensions of the respiratory part of the nasal cavity into frontal bones. Which of the following
statement related to the frontal sinus?

a. Frontal sinuses are usually detectable in children by 7 years of age.


b. The right and left sinuses each drain through a frontonasal duct into the ethmoidal infundibulum, which opens into
the semilunar hiatus of the middle nasal meatus.
c. The frontal sinuses are innervated by branches of the supra-orbital nerves (CN V1).
d. The right and left frontal sinuses are requal size, and the septum between them is situated in the median plane.
e. Often a frontal sinus has two parts: a vertical part in the squamous part of the frontal bone, and a horizontal part in
the orbital part of the frontal bone.

180. State the blood pH disturbance which is most likely to occur in the case of severe asthmatic attack?

a) No blood pH disturbance because lung does not produce any acid or base
b) No blood pH disturbance because the kidney can keep blood pH in its normal value
c) Increase of blood pH because increase in respiratory rate will excrete more CO2
d) Decrease of blood pH because lack of O2 in turn will produce more lactic acid
e) Decrease of blood pH due to increase of H2CO3

181. A 42-year-old man came to the clinic complaining productive cough and occasionally bloody sputum. He also had night sweat
and body weakness, loss of appetite and mild fever for the last 4 weeks. He is also noted as schizophren patient. The doctor
plan to give a regimen.
What is drug should remove/change from the regimen?

a) INH
b) Rifampicin
c) Ethambutol
d) Streptomycin
e) Pyrazinamid

182. Submucosal venous plexus is an important part of the body's thermoregulatory system, exchanging heat and warming air
before it enters the lungs. Which of the following vein is correct based on above description?

a) Sphenopalatine veins
b) Facial veins
c) Ophthalmic veins
d) Angular
e) Medial nasal vein

183. Which cell is a type of neuron?

a) Basal cells
b) Olfactory cells
c) Sustentacular cells
d) Brush cell
e) Dust cells

184. A 30-year-old came to outpatient clinic complaining haemoptysis since 1 week ago. After complete examination the doctor
concluded that this patient had pulmonary tuberculosis.
Which of the following is the histopathological hallmark of tuberculosis infection?

a) Interalveolar septum destruction


b) Caseous granuloma
c) Non caseating granuloma
d) Cavity formation
e) Bleeding

185. Choose one of the most appropriate option from the statement below:
The tonsil is in the mucous membrane of the roof and posterior wall of the nasopharynx.
a) Lingual tonsil
b) Palatine tonsil
c) Pharyngeal tonsil
d) Tubal tonsil

186. A 25-year-old man came to outpatient clinic with nasal congestion since 1 month earlier. Physical examination revealed
bilateral gelatinous mass in nasal cavity. No ulceration were seen. Clinically, this mass is benign.
Which of the following condition is most appropriate for this mass?

a) Adenoma
b) Carcinoma
c) Papilloma
d) Polyp
e) Rhinitis

187. A 26-year-old female works as a teacher complained of productive cough for 10 days, accompanied with malaise, fever, and
chest pain. The patient did not feel any muscle ache or pain. An x-ray revealed patchy consolidation in the right lung and the
white blood count was slightly increased. Microscopic examination revealed Gram (+) cocci.
What is the most likely diagnosis for this patient?

a) Acute community acquired pneumonia


b) Lung tuberculosis
c) Pleural effusion
d) Miliary tuberculosis
e) Atypical pneumonia

188. Laryngotracheal tube initially is in open communication with the primitive foregut.
Which of the following embryonic strctures is responsible for separating these two structures?

a) Laryngotracheal diverticulum
b) Posterior oesophageal folds
c) Laryngotracheal groove
d) Bronchopulmonary segment
e) Tracheoesophageal septum

189. Which of the following describes the condition from dyspnea that happen in patient with asthma is because of stimulation in
the respiratory center?

a) Increase CO2 in the respiratory center


b) Decrease O2 in the respiratory center
c) Increase H+ in the respiratory center
d) Decrease blood PO2
e) Increase blood PCO2

190. Choose one of the most appropriate option from the statement below:
Lymph follicles covering the posterior 3rd of the tongue.

a) Lingual tonsil
b) Palatine tonsil
c) Pharyngeal tonsil
d) Tubal tonsil
191. Which of the following caused by the hyperventilation due to panic syndrome?

a) Respiratory acidosis
b) Metabolic acidosis
c) Respiratory alkalosis
d) Metabolic alkalosis
e) Normal blood PH

192. Which of the structure points to thyrohyoid membrane?

a) C
b) D
c) E
d) F
e) G

193. During the inspiration, how does alveolar pressure compare to atmospheric pressure?

a) Alveolar pressure is less than atmospheric


b) Alveolar pressure is greater than atmospheric
c) Alveolar pressure is the same as atmospheric
d) Alveolar pressure is independent with atmospheric pressure
e) None of the answer above

194. A 50-year-old woman is admitted to the hospital with a swollen right calf. She has history of surgical cholecystectomy 10
days ago. Physical and diagnostic findings reveal a deep vein thrombosis. On the next day, she suffered a mild-to-moderate
dyspnea. No history of dyspnea or respiratory before. Chest X-ray shows clear lungs without infiltrates.
What further less invasive work up suitable for the case?

a) F-18 FDG PET/CT Scan


b) Doppler Ultrasound
c) V/Q Study
d) Tc-99m Ethambutol Scan
e) Pulmonary Function Test

195. The nasolacrimal duct, which drains tears from the lacrimal sac, opens into the anterior part of this structure. Which of the
following structure is correct based on above description?

a) Superior meatus
b) Middle meatus
c) Inferior meatus
d) Nasopharynx
e) Sphenoidal sinus

196. A 26-year-old female works as a teacher complained of productive cough for 10 days, accompanied with malaise, fever, and
chest pain. The patient did not feel any muscle ache or pain. An x-ray revealed patchy consolidation in the right lung and the
white blood count was slightly increased. Microscopic examination revealed Gram (+) cocci.
What is the microbiological examination should be done after gram staining?

a) Blood smear
b) Ziehl Neelsen staining
c) Culture
d) Coagulase test
e) Blood culture

197. A 33-year-old male presented in your clinic with productive cough for 4 weeks, low-grade fever, and weight loss. This patient
smokes 1 pack of cigarettes every day. X-ray revealed patchy lesions in both upper lung.
What is the most likely bacteria that caused the disease?

a) Staphylococcus aureus
b) Mycobacterium tuberculosis
c) Streptococcus pyogenes
d) Streptococcus pneumoniae
e) Parainfluenza virus

198. A 6-month-old boy presents to emergency department with difficulty of breathing since yesterday. The symptom preceded by
fever and cough since 3 days before. The difficulty of breathing is not accompanied by wheezing or stridor. In physical
examination reveals fever, tachipnea, chest indrowing, and crackles at the auscultation of the lung.
What is the common etiology of the disease?

a) Streptococcus pneumoniae
b) Mycoplasma pneumoniae
c) Klebsiela pneumoniae
d) Pseudomonas aeroginosa
e) Bordetella pertusis

199. A 25-year-old woman is performed arterial blood specimen for blood gases analysis from femoral artery. She was very nervous
and discomfort because of this procedure, and her breathing become faster.
A condition of hyperventilation in this patient could result interference to the blood gases analysis such as:

a) Decreased pH levels, Decreased pCO2 levels, and increased pO2 levels


b) Increased pH levels, decreased pCO2 levels, and increased pO2 levels
c) Decreased pH levels, decreased pCO2 levels and decreased pO2 levels
d) Increased pH levels, increased pCO2 levels and increased pO2 levels
e) Decreased pH levels, Increased pCO2 levels and increased PO2 levels
200. An 11-month-old baby boy without underlying congenital abnormality presented to the Emergency Room with a history of
fever for one day associated with runny nose, noisy breathing, barking cough, dyspnea, hoarseness of voice and irritability.
His respiratory rate was 60 breaths per minute and oxygen saturation of 99%. There were also presence of inspiratory stridor,
subcostal recession and nasal flaring. He was treated with nebulized adrenaline 0.5 mg, iv hydrocortisone 50 mg and oxygen
via nasal prong.
Where can we find the antigen of microorganism causing this disease?

a) In cells from respiratory washing


b) Sputum
c) Gastric lavage
d) Oral swab
e) Outside the cells from respiratory washing

1. Which of these factors is the most responsible for regulation of repiratory center?
a. Blood Ph
b. Blood O2
c. Blood CO2
d. Blood Hb
e. Blood 2,3 BPG

2. Chlamydiahas an unusual three stage cycle of development. Which of the


following statements best describe the sequence of this cycle?
A. Development of an initial body, synthesis of elementary body progeny, penetra¬tion of
the host cell.
B. Penetration of the host cell, development of an initial body, synthesis of ele¬mentary
body progeny.
C. Penetration of the host cell, synthesis of elementary body progeny, development of an
initial body.
D. Synthesis of elementary body progeny, development of an initial body, penetra¬tion of
the host cell.
E. Synthesis of elementary body progeny, penetration of the host cell, development of an
initial body.

3. One of the most important conditions in all respiratory passageways is to keep them
open thus allowing easy passage of air to and from alveoli
Whay is the most likely mechanism to prevent bronchioles from collapsing?
A. Multiple cartilage rings
B. Rigidity of their walls
C. Transpulmonary pressure
D. Intra-alveolar pressure
E. Surfactant
4. As the bolus of food enters the posterior mouth and pharynx, it stimulates receptors
all around the opening of pharynx to initiate a series of automatic pharyngeal muscle
contraction. One of their purposes is to prevent the foods enter respiratory tract.
Which is the most likely mechanism is true?
A. Larynx moves downwardly to lift the glottis out of the main stream of blood flow, so the
food mainly passes on each side of the epiglottis, rather than over its surface
B. Larynx is pulled downward and anteriorly by the neck muscles
C. Vocal cords of the larynx are strongly approximated
D. Ligaments prevent downward movement of the epiglottis

the epiglottis is pulled down, and the arytenoid cartilages are pulled
forward, toward the epiglottis. Realize that two other mechanisms
which are independent of laryngeal movements per se also help to
keep food from entering the larynx during swallowing. First, the larynx
is raised, along with the pharynx, by the contraction of the longitudinal
muscles of the pharynx. Second, the epiglottis also acts partly as a
passive barrier to the movement of food, so that food tends to be
funneled to either side of the larynx, into the PIRIFORM RECESSES
and from there into the esophagus.
E. The epiglottis swings backward and move up over the opening of the larynx

5. A 64-year-old man came to a clinic for follow-up of his chronic congestive heart failure. He
has a marked reduction in his ejection fraction following a series of myocardial infarctions.
His symptoms include dyspnea on exertion, paroxysmal nocturnal dyspnea, orthopnea,
and peripheral edema. He has normal renal function. He is on treatment with twice daily
of ACE inhibitor and once daily of furosemide. You decide to add digoxin to his regimen.
Which is the most appropriate prescription for the furosemide?
A. R/ Furosemide 40 mg tab. No. XV
S 1/2-0-0 tab
B. R/ Furosemide 40 mg tab. No. XV
S 1/2x1 tab o.h
C. R/ Furosemide 40 mg tab. No. XV
S b.dd 1/2 tab o.m
D. R/ Furosemide 40 mg tab. No. XV
S q.dd tab 1/2 o.h
E. R/Furosemide 40 mg tab. No. XV
S s.dd tab ½ om

6. Due to poliomyelitis, a patient suffered paralysis of his exintspeirrantioan l intercostal muscles. For
this patient, which is the most likely parameter which would still remain normal?
A. Inspiratory reserve volume (IRV)
B. Expiratory reserve volume (ERV)
C. Total lung capacity (TLC)
D. Vital capacity (VC)
E. None of the above

7. Speech is composed of some mechanical functions, such as phonation, resonance,


and articulation. Select the most likely places where the phonation process occurs:
A. Lips, tongue, and soft palate
B. Mouth, tongue, and nose
C. Mouth, nose, and paranasal sinuses
D. Vocal cords
E. Laryngeal cartilage, pharynx
Speech is composed of two mechanical functions :
8. A 46-year-old male presented to the Emergency Department with complaint of dyspnoea
• phonation which is achieved by the larynx
The larynx, is especially adapted to act as a vibrator. The vibrating element is the vocal folds, commonly called the vocal
cord. During normal breathing, the cords are wide open to allow easy passage of air. During phonation, the cords move
several hours after a road trafic accidenttogether
. Onsoexamination, there
that passage of air between themwere
will causeseveral
vibration. Theinjuries on is determined mainly by the
pitch of the vibration
degree of stretch to one another and by the mass of their edges.

the right anterior chest and anterior•artiacubladtioon wmhicihnisaalchiew


vealyl.theCshtruectsurtes orfathde imoougthraphic examination
db

The resonators include the mouth (lips, tongue and soft palate), nose, associated nasal sinuses, the pharynx and even the

revealed fracture of the right 4th,,5th,andch6estthcarviityb. s at three different sites anteriorly . There were
left mediastinal shift , infiltration in the right lung , absence of pleural line , and opacification
with meniscus sign in both lower hemithorax.
Select the most likely features of hydropneumothorax.
A. Opacification in hemithorax with meniscus sign.
B. Air fluid level and pleural line.
hyperlucent => decrease density => lebih hitam => byk air trapping n decrease vasculari marking
C. Hyperlucency of lung with pleural line.
D. Hyperlucency of lung with mediastinal shift.
E. Hyperlucency of lung with meniscus sign.
‘meniscus’ sign that we normally see at the edge of a pleural effusion is generally absent when there is an
underlying pneumothorax –

9. Type of noisy breathing in above case -either inspiratory or expiratory-- is determined by the
level of obstruction. Select the most likely area where inspiratory stridor occurs:
A. Nasopharynx, oropharynx, supraglottis
B. Nasopharynx, glottis, tracheobronchial
C. Oropharynx, supraglottis, subglottis
Stridor is typically heard on inspiration, but can also be heard on expiration and may also be biphasic.
D. Supraglottis, glottis, subglottis Inspiratory stridor suggests an obstruction above the glottis, due to collapse of soft tissues with negative
pressure created during inspiration. Expiratory stridor indicates an obstruction in the lower trachea
E. Glottis, subglottis, tracheobronchial (intrathoracic trachea and lower bronchi). Biphasic stridor can indicate edema near the cricoid cartilage that
surrounds the trachea and is indicative of a fixed caliber of airway unaffected by changes in intrathoracic
pressure.

10. The structure of A secretes substance which reduces surface tension. Choose the best
epithelial characteristic that suit for structure A:
A. Ciliated columnar cells
B. Flattened simple squamous epithelium with prominent nuclei and vacuolated cytoplasma
C. Flattened simple squamous epithelium with nuclei that infrequently seen
D. Rounded shape with nuclei that infrequently seen and vacuolated cytoplasma
E. Rounded shape with prominent nucleus and vacuolated cytoplasma

11. A 62-year-old man was being managed in the intensive care unit following a large anterior
wall myocardial infarction. He had been appropriately managed with oxygen, aspirin, nitrates,
and β-adrenergic receptor blockers but developed recurrent episodes of ventricular
tachycardia. During these episodes he remained conscious but feeling dizzy. He was also
diaphoretic and hypotensive.
What is the most appropriate note that must be written in prescription for indicating "the
prescription has to be priority"?
A. ITER
B. PRO INJ.
C. EMERGENCY
D. CITO
E. PRO RE NATA

12. A 24 year old woman came to the clinic complaining fullness in face for more than a week.
She has a historyof allergy of dust and sneezing a lot in the morning. She complains of
continuous greenish nasal discharge for the past 2 weeks. From x-ray it was revealed there
was air-fluid level in the anterior part of the face.
Where does this sinus drain?
A. Semilunar hiatus in middle nasal meatus
post erior aspect, maxillary sinus
B. Frontonasal duct in superior nasal meatus
C. Sphenoethmoidal recess fro
pontal sinus, frontonasal duct in middle meatus
sphenoidal sinus sterior ethmoidal sinus ke superior nasal meatus
D. Inferior nasal meatus
E. Kiesselbach plexus

13. Select the most likely factor responsible for transferring O2 from a pregnant mother to her
foetus.
A. Blood O2
B. Blood CO2
C. Blood lactate
D. Blood haemoglobin
E. Blood 2,3-BPG

14. A 60-year-old man came to the emergency department because he had difficulty in breathing
which was worsening for the last 3 days. He also had cough with thick sputum. The patient
already had complained breathing difficulty since 5 years ago which slowly worsened. He had
been smoking for years.
Physical examination showed respiratory rate was 38x/minute, pursed-lip breathing,
decreased breath sounds through the stethoscope, rounded fingertips (clubbing), but no
cyanosis was found.
Chest radiographic findings: be
Th rar ti IPP makin negat if, k rn m eng im ban gi p ar u
e lungs were hyperexpa coarse bronchovascular
nygdhyepderexw
pan
istih.

markings. AP diameter and retrosternal airspace were increased, diaphragm was flattened.
Which of the following would be expected to be decreased in this patient?
A. Airway resistance
B. Lung compliance
C. Intrapleural pressure
In restrictive diseases, recoil of the lungs is increased. The only way to expand your lungs now would be to apply a greater (ie more negative) IPP.
D. FEV1
E. Options B, C, and D above

15. Which sinuses shares the same drainage with the affected sinus above?
maxilla : post aspect semilunar hiatus middle chonchae
A. The posterior ethmoidal cells rior nasal
supe
B. All the ethmoidal bullae
C. The middle and posterior bulla
D. The frontal sinus
E. The anterior ethmforoindtonaalsacl deucltlof semilun
ar middle chonchae

anterior semilunar hiatus middle chonchae

16. A 50-year-old man was a-known case of Chronic Obstructive Pulmonary Disease. He smoked
about 28 pack years of cigarettes. His body weight was 44 kg and his height was 165 cm.
The patient was advised regarding aspiration. What would be the most likely feeding
strategy that would prevent aspiration?
A. Resting after a big meal
B. Proper sitting posture during eating
C. Promoting parenteral route intake
D. Consuming highly nutritious food
E. Encourage intake of liquid meal

17. A 64-year-old man came to a clinic for follow-up of his chronic congestive heart failure. He
has a marked reduction in his ejection fraction following a series of myocardial infarctions.
His symptoms include dyspnea on exertion, paroxysmal nocturnal dyspnea, orthopnea, and
peripheral edema. He has normal renal function. He is on treatment with twice daily of ACE
inhibitor and once daily of furosemide. You decide to add digoxin to his regimen.
Which of the following statements regarding digoxin is the most appr opriate?
A. It has a very wide therapeutic index
B. It induces activity of myocardial Na+/Ca2+-exchange
C. It induces vagal tone
D. It inhibits cardiac conduction in ventricle
E. It decreases intracellular concentrations of calcium in myocytes

18. A 28-year-old man with severe kyphoscoliosis due to cerebral palsy was experiencing
worsening shortness of breath with exertion. On examination, he had a severe scoliosis to
the left and decreased air entry to that side. His right lung was clear, JVP was 3 cm, and heart
sounds were normal. Pulmonary function tests were performed.
Select the most likely abnormality to be seen on his pulmonary function tests:
A. Increased total lung capacity (TLC)
B. Increased functional residual capacity (FRC)
C. Decreased TLC
Restrictive lung disease manifested by a reduction in the total lung capacity (TLC) is characteristic of severe scoliosis. However, the measurement of
the TLC requires equipment (body plethysmography, helium dilution or nitrogen washout) that are not readily available in every clinical setting. In such
cases simple spirometry may provide a good estimate of the restrictive lung defect because the decrease in the FVC is proportional to the decrease in
TLC unless the patient has a mixed restrictive and obstructive defect.

Residual volume (RV) remains generally within the predicted values. Due to the relative decrease in TLC, RV/TLC ratio is increased. Similarly, the
functional residual capacity (FRC) is also normal or slightly diminished and the FRC/TLC ratio is increased.
D. Increased lung compliance
E. Decreased FRC

19. Bordetella pertussis is a Gram-negative rod that causes severe respiratory disease. Select
the statement which best characterizes this organism
A. It produces at least one protein toxin consisting of two subunits, A and B, that cause
severe spasmodic cough usually in children.
B. It produces toxin that blocks protein synthesis in an infected cell and carries a lytic
bacteriophage that produces the genetic information for toxin pro¬duction.
C. It requires cysteine for growth.
D. It secretes erythrogenic toxin thaBtrucceallau, Fserasncitshellea, Lcehgiaonreallac, Pteastriesurteillca signs of scarlet fever.
E. It secretes exotoxin which is called “Shiga-like toxin” following the attachment of the
organism to the mucosal membranes.

20. Many obstructive diseases of the lung result from narrowing of the lumen, due to excessive
contraction of the smooth muscle in its wall.
Which is the most frequent part of the airway that the excessive contraction of smooth muscle
occur?
A. Larynx - trachea
B. Trachea – large bronchi
C. Large bronchi – smaller bronchi
D. Smaller bronchi – larger bronchioles bronchi doesnt have smooth m
uscle
E. Larger bronchioles – smaller (respiratory) bronchioles

21. M. Tuberculosis can be found in the sputum of patients with tuberculosis. Select the best
medium for the isolation of this organism
A. Loffler’s medium
B. Lowenstein-Jensen medium
C. Sheep blood agar
D. Thayer-Martin agar
E. Thiosulfate citrate bile salts sucrose medium

22. An immunocompromised patient was suspected of having an infection with A. Fumigatus.


Which of the clinical condition is most likely to occur?
A. Thrombosis
B. Superficial rash
C. Thrush
D. Urinary tract infection
E. Wound infection
23. A 10-year-old boy came to the Pediatric clinic with chronic cough and decrease in body
weight. His father was on tuberculosis treatment for the past 2 weeks. The boy was treated
with anti-TB drugs.
Select the most appropriate TB treatment regiment!
A. 2RHZ-4RH
belum boleh ethambutol
B. 2RHZE-4RH
C. 2RHZ-7RH
D. 2RHZE-7RH
E. 2RHZE-10RH

24. There is lining epithelium changes from bronchi to alveoli. What is the epithelium type for
bronchi and alveoli?
A. Pseudostratified ciliated columnar cells and simple squamous epithelium
B. Simple squamous epithelium and pseudostratified ciliated columnar cells
C. Simple columnar epithelium and simple cuboidal epithelium
D. Simple cuboidal epithelium and simple columnar epithelium
E. Pseudostratified ciliated columnar cells and simple cuboidal epithelium

25. 50-years-old labourhospitalized because of exacerbation of Chronic Obstructive


Pulmonary Disease. At the admissionday he got shortness of breath (RR 40x/min) and
fever (Temp 38,9 0C). He is now on the 3rd day of hospitalization and he still feel shortness
of breath but the respiratory rate decreased to 26x/min. He has no fever. There are
minimal sputum retension. He can sit independently but still feel tired to walk. His
previous spirometry test reveals FEV1/FVC 60%
Select the most important benefit of Pulmonary Rehabilitation for COPD patient with
exacerbation!
A. Improvement in exercise tolerance
B. Improvement in vital capacity
C. Improvement in oxygen consumption
D. Reduction in dyspnoea and respiratory rate
E. Improvement in ambulation capacity

26. A 24 year old woman came to the clinic complaining fullness in face for more than a
week. She has a historyof allergy of dust and sneezing a lot in the morning. She
complains of continuous greenish nasal discharge for the past 2 weeks. From x-ray it
was revealed there was air-fluid level in the anterior part of the face
Which nerve conducted the pain in this patient?
A. Facial nerve
B. Trigeminal nerve
C. Ophthalmic nerve
D. Maxillary nerve
E. Mandibular nerve

28. The most likely membrane channel that is responsible in transportingCO2


A. Calcium channel The mucous
membranes
receive their
postganglionic
parasympathetic
nerve innervation
for mucous
secretion
originating from
the greater
petrosal nerve (a
branch of the
facial nerve). The
superior alveolar

+ + (anterior, middle, and posterior) nerves, branches of the maxillary nerve provide sensory innervation.

B. K , Na

C. Anion exchange channel


D. Sodium-glucose channel
E. No membrane channel is needed

29. A 4-years old boy brought to the outpatient clinic by his mother because for the past hour his
nose had been bleeding. She said that she had tried to stop the bleeding by pinching his nose.
Her son had constant fever for 3 days, and it doesn't subside with fever medicine she gave
him. There were red spots in his arms and legs since yesterday.
Which arteries most likely contributed to the bleeding?
A. The anterior and posterior ethmoidal arteries
B. The anterior and posterior palatine arteries
C. The greater and lesser palatine arteries
D. The superior and inferior ethmoidal arteries
E. The superior and inferior labial arteries
Woodruff area is located on the inferior aspect of the lateral nasal wall, posterior to the inferior turbinate.
It is formed from the anastomoses of the sphenopalatine and pharyngeal arteries. The posterior location
makes it a common source for severe, nontraumatic bleeds. Kiesselbach plexus is an anastomosis with
branches from both the internal and external carotid artery systems. The anterior ethmoid, greater
30. Treatment of asthma consist of controller and reliever. Select the most likely reliever drug in
palatine, sphenopalatine, and superior labial arteries all form a plexus of vessels in the anteroinferior

the management of asthma B. Cromones


A. Inhalation glucocorticoid C. Long acting Beta 2
nasal septum. Kiesselbach plexus is the
agonist source of the majority of nose bleeds.

D. Short acting beta 2 agonist


E. Leucotriene modifiers

31. Which numbers are structures that separated by the oblique fissure?
A. 1 and 2
B. 2 and 3
C. 6 and 7
D. 7 and 8
E. 8 and 10
32. A 29-year-old man came to the emergency department complaining of breathlessness for
the past 3 days. It was also associated with cough, chest pain, mild fever, night sweat, and
weight loss for the last 6 months.
Physical examination showed:
Respiratory rate : 32x/minute
Thoracic movement : asymmetrical user
Hemithoraxdextra : - vocal fremitus decreased 2017-12-03 09:28:56
: - dullness on percussion
--------------------------------------------
: - diminished breath sounds
Chest X-ray : Radio-opaque density extending from the base of hemithoraxdextra with a may play a role
The following mechanisms
meniscus-shaped upper surface. Costophrenic angle was in the formation
blunt. of pleural effusion:
Mediastinal
shift was negative. Altered permeability of the pleural m m
e
s
e
n
ra
b
Select the most possible physiologicaplleum
ral effusion
(eg, inflammation,
echanism as the cause of this patient's chief complain: malignancy,
A. Capillary obstruction pulmonary embolism)
B. Hydrostatic pressure decreased Reduction in intravascular oncotic presure(eg,
C. Osmotic pressure increased hypoalbuminemia due to nephrotic
decrease syndrome or cirrhosis)
D. Capillary permeability increased Increased capillary permeability or vascular
E. Lymphatic obstruction disruption (eg, trauma, malignancy,
bisa sih ini jg inflammation, infection, pulmonary
infarction, drug hypersensitivity,
uremia, pancreatitis)
33. Which of these factors is closely related to the chloride shift in maintaining its level?
Increased capillary hydrostatic presurein the
systemic and/or pulmonary circulation
(eg, congestive heart failure, superior
vena cava syndrome)
A. Blood pH
B. Blood O2 The underlying properties
creating the chloride shift are
C. Blood CO2 the presence of carbonic
anhydrase within the RBCs but
not the plasma, and the
permeability of the RBC
membrane to carbon dioxide
and bicarbonate ion but not to
hydrogen ion. Continuous
process of carbonic acid
dissociation and outflow of
bicarbonate ions would
eventually lead to a change of
intracellular electric potential
because of lasting H+

D. Blood Hemoglobin ions. Inflow of chloride ions maintains electrical neutrality of a cell. The net direction of bicarbonate-chloride exchange (bicarbonate out of
RBCs in the systemic capillaries, bicarbonate into RBCs at pulmonary capillaries) proceeds in the direction that decreases the sum of the
eleTchterocchhleom
ridiceasl hpiofttemnatiyalaslsfoorrtehgeuclahtleortihde afnfidnibtyicoafrbhoenmaoteglioobnisn bfoerinogxytrgaennspthorroteudgh the chloride ion acting as an allosteric effector
E. Blood 2,3 biphosphoglycerat (BPG, DPG)
34. A 23-year-old woman, presented with productive cough and fever for 7 days. Physical
examination showed pectoriloquy
increase in and
tactile fremitus, b
crackles o ronchial
breath sound,
whispering n
the right lower
hemithorax
Select the most likely clinical diagnosis for this patient.
A. Pleural effusion
B. Pneumonia
C. Pneumothorax
D. Pleural fibrosis
E. Lung emphysema

35. 50-years-old labourhospitalized because of exacerbation of Chronic Obstructive Pulmonary


Disease. At the admissionday he got shortness of breath (RR 40x/min) and fever (Temp 38,9
0 An increase in tactile fremitus indicates denser or
C). He is now on the 3rd inflamed lung tissue,

day of hospitalization and he still feel shortness


w of breath but the
h
i
c
h
c
a
n
b
e
c
a
u
s
e
d
b
y
d
i
s
e
a
s
e
s
s
u
c
h
a
s
p
n
e
u
m
o
n
ia. A decrease
suggests air or fluid in the pleural spaces or a
decrease in lung tissue

respiratory rate decreased to 26de


xns/im
ty, which can be caused by diseases such as chronic obstructive
pulmonary indisease
. Heorhasthma.
as no fever. There are minimal sputum retension.
He can sit independently but still feel tired to walk. His previous spirometry test reveals
FEV1/FVC 60%. Select the most beneficial rehabilitation technique in this patient to reduce
dyspnoea!
A. Muscle relaxation technique
B. Flexibility exercise
C. Controlled breathing exercise
purse lip + diaphragm breathing??
D. Sputum toileting/drainage
E. Aerobic exercise

36. A 43-year-old male with positive AFB is diagnosed to have pulmonary TB which has never
been treated before. Select the most likely antituberculosis drugs treatment
A. 2 R7H7E7Z7 / 4 R3H3
B. 2 R7H7E7Z7S7 / 1 R7H7E7Z7 / 5 R3H3E3
C. 2 R7H7Z7 / 4 R3H3
D. 2 S7H7E7 / 10 H3E3
E. 2 S7H7Z7 / 10 S7H7
37. An 18-year-old female singer came to her private doctor because of a change in her voice. She
suffered from common cold associated with stuffy and runny nose for the past two days. Select
the most likely resonance chamber that is affected:
A. Mouth and Soft Palate
tounge, soft palate, lips = articulators
B. Mouth and Nose
C. Nose and Larynx
D. Due t o its s mall size , t he larynx acts as a resonator only for high frequencies
Vocal cord and Paranasal Sinus
E. Soft Palate and Pharynx
The pharynx is the most important resonator by virtue of its position, size, and degree of adjustability. It is the first cavity of any size through
which the product of the laryngeal vibrator passes;

38. Acute bronchitis is a form of lower respiratory tract inflammation affecting the bronchi of
the lungs. It usually comes on suddenly and can last for 3 to 10 days. Select the most
common
etiology of acute bronchitis in adult.
A. Streptococcus pneumonia
B. Hemophilusinfluenza
C. Klebsiellapneumoniae.
D. Rhinovirus
E. Staphyllococuspneumoniae.
The most common viruses include influenza A and B, parainfluenza, respiratory syncytial virus, and coronavirus, although

39. A 50-year-old patient with normal anlung


etiologic agent is identified only in a minority of cases. [1]
volumes (total lung capacity = 5.0 liters, vital capacity
Acute bronchitis is usually caused by infections, such as those caused by Mycoplasma species, Chlamydia pneumoniae,
= 4.0 liters, expiratory reserve volume = 1.0 liters). What is the most likely range of the
Streptococcus pneumoniae, Moraxella catarrhalis, and Haemophilus influenzae, and by viruses, such as influenza,
parainfluenza, adenovirus, rhinovirus, and respiratory syncytial virus. Exposure to irritants, such as pollution, chemicals, and

FEV1 (one-second forced expired tob


v oaclcuo sm
mo ke, may a l so cau se a cu te b ronchial irritation.
e) in this patient?
A. 1.0-2.0 liters
B. 2.0-3.2 liters
C. 3.2-4.0 liters
D. 4.0-5.0 liters
E. 5.0-6.0 liters

40. A 7-year-old boy was brought by his mother to the primary health care for difficulty in
swallowing for the past 2 days. He could only consume semisolid food and fluid. His condition
was associated with fever, sore throat, and snoring while sleeping. For the last one year, he
has been having recurrent sore throat. There were no associated ear discharge, earache,
runny, obstructed nose, and hoarseness. What is the effect of the pharyngeal stage of
swallowing on respiration?
A. Pharyngeal movements obstruct the airway temporarily
B. Pharyngeal contractions push the upper airway to the front
C. The respiratory center inhibits the swallowing center, halting swallowing to allow
respiration to proceed
D. The swallowing center inhibits the respiratory center, halting respiration to allow
swallowing to proceed
E. There is no direct relationship between swallowing and the respiration process

41. A 50-year-old man with history of tobacco use who complained of several months of
cough and lower back pain and an 11.3-kg weight loss. He was treated for pneumonia
after a chest radiograph showed total opacification of the right lung. Computed
tomography imaging subsequently revealed a superior right hilar mass and mediastinal
lymphadenopathy.
Mediastinal biopsy showed poorly differentiated epithelioid tumour with desmoplastic
stromal reaction, neutrophil infiltration, and squamous differentiation. Tissue
immunostaining confirmed a non-small-cell lung cancer.
Nuclear medicine procedure reveal high uptake of radiopharmaceutical on a superior right
hilar mass and mediastinal lymphadenopathy. Another high radiopharmaceutical uptake also
found on lumbar bones and on right adrenal
What most likely causes the uptake on bones and adrenal?
A. Inflamation process
B. Chronic infection
C. Metastatic process
D. Healing process
E. Complication of pneumonia treatment

42. A 67-year-old male, came to theEmergency Unit complaining of productive cough, fever and
shortness of breath for 5 days. On physical examination, crackles were heard at the right
middlehemithorax. Chest x ray revealed infiltrates in the right middle lobe.
Select the most likely diagnosis for this patient
A. Acute bronchitis
B. Pneumonia
C. COPD
D. Tuberculosis
E. Bronchiectasis

43. A 55-year-old man underthe treatment for adenocarcinoma of the lung presentedwith fever,
chest pain, and dry cough. Gram stain of the sputum was unremarkable butthe culture
revealed many small Gram-negative rods on charcoal yeast extract agar. Select the most
likelymicroorganism described
A. Chlamydia trachomatis
B. Klebsiellapneumoniae
C. Legionella pneumophila
D. Mycoplasma pneumonia
E. Staphylococcusaureus

44. Select the most likely factor which will increase in a severe vomiting patient:
A. Blood pH
B. Blood haemoglobin
C. Blood 2,3-BPG
D. Blood Fe2+
E. Blood CO2
45. Gas exchange occurs across the respiratory membrane. Enumerate the most likely layers
which constitute this membrane:
A. Type 2 alveolar epithelium, fused basal laminae endothelial cells Type 2 alveolar
epithelium, fused basal laminae, type 1 alveolar epithelium
B. Type 1 alveolar epithelium, fused basal laminae, endothelial cells Type 1 alveolar
epithelium, fused basal laminae, endothelial cells in the type 2 alveolar epithelium
C. Type 2 alveolar epithelium, fused basal laminae, squamous endothelial cells in the
bronchiolus capillaries
46. Asthma is a chronic inflammatory disease with characteristic tracheobronchial tree hyper-
responsiveness to multiple stimuli which resulting in episodes of reversible airway narrowing.
Some physiological changes occurred in this disease are related to nervous and local control
of the organ affected. Select the most likely statement which is true about parasympathetic
stimulation of the bronchiole:
symphatetic b2 => bronchodilation
A. Their fibres are derived from the accessory nerve which penetrate the lung parenchyma
B. Parasympathetic nerves secrete cAMP in order to produce effects
Ach --> Na/K channel
C. Parasympathetic stimulation causes bronchiolar constriction

D. Irritation of the epithelial membrane of the respiratory tracts can activate these nerves
E. Their effects in bronchiolar smooth muscles are resulted from adenylate cyclase activation

47. Which of these factors is the most responsible for converting T-form hemoglobin to R-form
hemoglobin? taut

A. Blood pH relax

B. Blood O2
C. Blood CO2 T
a
u
D. Blood Hemoglobin. t
-
E. Blood 2,3 biphosphoglycerat (BPG, DPG) -
>
d
e
o
x
y
g
e
n
a
t
e
d
-
-
>
l
e
s
s
a
f
f
i
n
i
t
y
r
e
l
a
x
-
-
>
o
x
y
g
e
n
ated --> more affinity

48. An 18-year-old male came to the outpatient clinic complaining of pain in his throat. The pain
occurred after 3 hours ago he accidentally swallow small fishbone. He had tried to remove
them with no avail. Where would it most likely stuck?
A. In the pharyngotympanic fossa
B. In the piriform fossa
C. In the esophagus The most common site of impaction is usually at the
level of the tonsils, although the impacted bone
may be found at the base of the tongue, the
D. In the esophagus vallecula or the pyriform fossa.

E. In the posterior arch of the pharynx

49. A 21-year-old college student complained of malaise, low-grade fever, and a harsh cough
for a week. There was no muscle-ache and pain. An x-ray revealed diffuse interstitial
pneumonia in the left lung. The WBC count was normal. Based on the information given,
what is the most likely bacteria causing this disease?
A. Influenza
B. Legionella
C. Mycoplasma pneumonia
D. Pneumococcal pneumonia
E. Staphylococcal pneumonia

1. epiglottis
50. Which structure is the rima glottis? 2. vestibule of larynx
3. hyoid bone
A. Number 2 4. vestibular fold
5. thyroid cartilage
6. vocal fold
B. Number 10 7. vocal ligament
8. vocalis muscle
C. Between either side of number 6 9. cricoid cartilage
10. subglottic
D. Between either side of number 4 rima
11. trachea
vestibulli

E. Posterior to the number 1


51. Pulmonary Disease. At the admissionday he got shortness of breath (RR 40x/min) and
fever (Temp 38,9 0C). He is now on the 3rd day of hospitalization and he still feel shortness
of breath but the respiratory rate decreased to 26x/min. He has no fever. There are
minimal sputum retension. He can sit independently but still feel tired to walk. His
previous spirometry test reveals FEV1/FVC 60%. Select the least likely aspect to be
included as the goalfor pulmonary rehabilitation process in this patient!
a. Improvement in cardiopulmonary capacity
b. Prevention and treatment of complications
c. Improve the recoil ability of the lung
d. Increased patient responsibility for self-care
e. Improvement in level of activity and quality of life

52. Paramyxoviruses are spherical or pleomorphic shaped viruses with three kinds of
glycoprotein which are HN, NP and F.Select the most likelyparamyxoviruswhich causes croup
syndrome.
a. Influenza virus
b. Measles virus
c. Mumps virus
d. Parainfluenza virus
e. Respiratory syncytial virus
53. A 42 year old man came to the outpatient department complained that his throat feels
sore since 2 days. Since 1 day ago, his voice became hoarse. He had mild fever and general
malaise. After checking his vital signs and performed head examination, the doctor examined
his throat. He noticed the prominent Adam's apple, as he palpate them, he asked the patient
to swallow What structure did he palpate?
a. The epiglotic cartilage
b. The thyroid cartilage
c. The hyoid bone
d. The cricoid cartilage
e. The arytenoids cartilage

54. Which is the most likely type of pressure that distends the lungs?
a. Alveolar pressure
b. Airway opening pressure
c. Transthoracic pressure
d. Transpulmonary pressure
perbedaan intrapleural - intraalveolar.
e. Intraalveolar pressure

55. A 42 year old man came to the outpatient department complained that his throat feels
sore since 2 days. Since 1 day ago, his voice became hoarse. He had mild fever and general
malaise. After checking his vital signs and performed head examination, the doctor examined
his throat. He noticed the prominent Adam's apple, as he palpate them, he asked the patient
to swallow Which groups of muscle controls the maneuver the patient was making?
a. The superior laryngeal muscles
The intrinsic laryngeal muscles are responsible for controlling sound production.

b. The thyroid muscles


c. The extrinsic laryngeal muscles The extrinsic laryngeal muscles support and position the larynx within the trachea (termasuk infrahyoid, suprahyoid).

d. The intrinsic pharyngeal muscles


e. The sternocleidomastoid

56. A seven-year-old girl is brought by her mother for severe sore throat,hoarseness, barking
and cough.This virus infects mammalian cells in culture andcauses haemolysis of red
blood cells.Select the most likely described virus.
a. Adenovirus
b. Group B Coxsackievirus
c. Parainfluenza virus
d. Rhinovirus
e. Rotavirus

57. A 44-year-old woman noticed a tightness or ache in her chest after playing tennis. The
tightness disappeared after a few minutes rest. Her LDL cholesterol was 300 mg/dL.
She was diagnosed with angina and prescribed isosorbidedinitrate (ISDN) tablets, which were to
be placed under the tongue, not swallowed. In addition, she was given aspirin and simvastatin.
What is the common side effect of ISDN?
a. Tension headache
b. Orthostatic hypertension
hipotension
c. Tachycardia
d. Gastric ulcer
e. Tolerance

58. A 30-year-old male, developed severe pneumonia after 8 days of hospitalization due
to fracture of his right femur.
According to ATS guideline, the most likely pathogen that caused his pneumonia is
a. Streptococcus pneumoniae
hospital acquired

b. Chlamydia pneumoniae
c. Mycobacterium tuberculosis
d. Mycoplasma pneumoniae
e. Pseudomonas aeruginosa

59. A 23-year old woman comes to the clinic complaining prolonged nose blocking. She had the
complaint since 2 weeks. She said that sometimes she felt something running down her throat,
and her phlegm smelled. She has history of frequent sneezing and runny nose particularly when
the weather was cold. The doctor advised her to do x-ray examination.
From water's view, it revealed cloudy view of both right and left large hollow structure inside a
bone. It has triangular shape, with the point directed toward the upper teeth, located in each side
of the nasal cavity
Where do the sinus above drains its content?
maxillary sinus??
a. Semilunar hiatus
b. The inferior nasal meatus
c. The superior nasal meatus
d. The sphenoethmoidal recess
e. The anterior nasal meatus

60. Streptococcus pyogenes is a toxigenic bacterium causing a variety of diseases. Select the
statement which best characterizes this organism.
a. It has capsules of polyglutamic acid, which is toxic when injected into rabbits
b. It produces at least one protein toxin consisting of two subunits, A and B, that cause
severe spasmodic cough usually in children
c. It produces toxin that blocks protein synthesis in an infected cell and carries a lytic
bacteriophage that produces the genetic information for toxin production.
d. It secretes erythrogenic toxin that causes the characteristic signs of Scarlet fever.
e. It secretes exotoxin which is called “Shiga-like toxin” following the attachment of the
organism to the mucosal membranes.

61. A 34-week-gestation baby boy presented with tachypnea, tachycardia, nasal flaring,
chest wall retraction, expiratory grunting, and cyanosis. The Downs score is more than 4.
The condition of this baby most probably because of structural immaturity of the lungs and
developmental insufficiency of a substance production in the lung secreted by type II
alveolar epithelial cells.
What is the most likely function of the substance secreted by type II alveolar epithelial cells?
a. Increase pulmonary surface tension
b. Increase pulmonary compliance
c. Decrease airway resistance
d. Increase pulmonary blood flow
e. Decrease transpulmonary pressure gradient

62. A 50-year-old man was a-known case of Chronic Obstructive Pulmonary Disease. He
smoked about 28 pack years of cigarettes. His body weight was 44 kg and his height was
165 cm.
Which of the following is the most appropriate nutritional recommendation for this patient?
a. Adding folic acid above the Recommended Dietary Allowance (RDA) values.
b. Adding vitamin B12 above the RDA values
c. Avoid overfeeding, taking small frequent meals
d. Decreasing intake of fiber
e. Decreasing intake of omega-3 fatty acids below RDA values

63. Which of these are part of the structure that made the nose vary in size and shape?
a. The nasal bone
b. Frontal process of maxilla
c. The alar cartilage
d. Crista Galli The projecting nose on the face of homo-sapiens is a later
development in the phylogenetic sequence [1]. It is the
e. Ethmoidal bulla development of cartilaginous superstructure of the nose
i.e. cartilaginous septum, upper alar cartilage, lower alar
cartilage which makes the nose a prominent midline
structure on the face and distinguishes us in the animal
kingdom
64. What is the bifurcation at the end of structure number 4 called?
a. Main bronchus
b. Bronchial bifurcation
c. Carina
d. Terminal Trachea
e. Tracheobronchial branch

65. Increase of this factor will decrease the affinity of Hemoglobin for O2
a. Blood pH
shift to the left
b. Blood O2

c. Blood CO2
d. Blood Hemoglobin
e. Blood 2,3 biphosphoglycerat (BPG, DPG)
66. H. Capsulatum,a dimorphic fungus found in soil which is heavily contaminated with bird
droppings. This organism causes pulmonary disease in human and can be detected by
tissue biopsy.
Select thestatement best describes the presence of the organism in tissue biopsy.
a. Arthrospores found within the hyphae.
b. Oval budding yeasts inside macrophages
c. Single-cell yeasts with pseudohyphae
d. Spherules containing endospores
e. Yeasts with broad-based bud

67. An infant, seen in the Emergency Department, presented with fever and persistent
cough. Physical examination and chest x-ray was suggestive of pneumonia
Select the most likely cause of this infection
a. Adenovirus
b. Coxsackievirus
c. Respiratory syncytial virus
d. Rhinovirus
e. Rotavirus
68. COPD characterized by persistent airflow limitation that is usually progressive and
associated with an enhanced chronic inflammatory response in the airways and the lung
to noxious particles or gases.
Select the most likely characteristic of chronic inflammatory process in COPD.
a. Lymphocyte CD 4+ dominant
b. Eosinophil dominant
asthma
c. Neutrophil dominant
d. Immunoglobulin E dominant
e. Basophil dominant

69. A 62-year-old man was being managed in the intensive care unit following a large anterior
wall myocardial infarction. He had been appropriately managed with oxygen, aspirin, nitrates,
and β-adrenergic receptor blockers but developed recurrent episodes of ventricular
tachycardia. During these episodes he remained conscious but feeling dizzy. He was also
diaphoretic and hypotensive.
Which of the following is the most likely action of verapamil?
a. It blocks L-type calcium channels
b. It increases heart rate
c. It induces contraction of coronary artery smooth muscle
d. It increases cardiac contractility
e. It inhibits the convertion angiotensinogen to angiotensin II

70. A 5-year old boy brought to the ER by his parents. They say he was playing with beads, and
one of them stuck in his nose.
When you try to take the bead out, blood coming out from his nose. From which vessel it most
likely bleed?
kieesel bach plexus : LEGS --> sup labial, ant ethmoidal, greater palatine --> sphenopalatine

a. Anterior palatine artery


b. Facial artery
-- > su p erior lab ia l
c. Posterior ethmoidal artery
d. Greater sphenoidal artery
e. Sphenoethmoidal artery

71. Chlamydia pneumoniahas recently been implicated in respiratory disease primarily


in children
Which of the following identification test is the best for this fastidious bacterium?
a. Cold agglutinin test
pa ke EL IS A
b. Culture of respiratory secretions in HeLa cells.
c. Culture of respiratory secretions on monkey kidney cells.
d. Detection of specific antigen in urine
e. Electron microscopy of sputum

72. A 7-year-old girl was brought by her parents to the outpatient clinic complained of having
fever for 2 days. She also complained of pain during swallowing. Her parents said that this was
the second time in 4 months she had the same complains. On physical examination, her
temperature was 38.4ºC, pulse rate 100 bpm. Her other vital signs was within normal limit.
When the doctor performed oral and pharyngeal examination, he found two enlarged bumps
on either side on of her throat. The enlargements were occupying half of her fauces, they were
red. Afterward, the doctor asked her parents for consent to do posterior rhinoscopy using a
mirror that introduced into the back of her throat.
What is the arterial supply for the first two enlargements?
a. tonsilar branch of facial artery
b. ascending pharyngeal artery
c. ascending palatine artery
d. descending palatineartery
e. superior laryngeal artery

73. In a person standing upright, which region of the lungs has the highest ventilation rate
and which region has the highest circulatory perfusion rate?
a. Highest ventilation: Apex; highest perfusion: Apex
b. Highest ventilation: Apex; highest perfusion: Base
c. Highest ventilation: Base; highest perfusion: Apex
d. Highest ventilation: Base; highest perfusion: Base
e. There is no "highest" region as the apex and base have equal ventilation and
perfusion rates

74. A 22-year-old male, admitted to emergency room in a remote hospital with a chief
complain breathing difficulty. His respiratory rate was 30x/min, and after several examinations
and chest x-ray, you diagnose this patient as massive right pleural effusion
What is the most appropriate definitive treatment for this patient?
a. Needle chest decompression
b. Chest tube insertion
c. Endotracheal intubation
d. Needle Aspiration
e. Oxygenation

75. A 9-month-old boy admitted to the Pediatric ER with difficulty in breathing. He had the
history of fever and cough. The parents noticed that he had noisy breathing prior to
admission. On physical examination, there was chest in-drawing with inspiratory stridor
What is the most likely diagnosis for this patient?
a. Asthma
b. Bronchiolitis
c. Croup
d. Pneumonia
e. Upper respiratory tract infection

76. A 30-year-old male, developed severe pneumonia after 8 days of hospitalization due
to fracture of his right femur.
Select the most likely antibiotics of choice for the above patient.
A. Aminoglycoside and ceftazidime
B. 2nd generation cephalosporin and minai HcArPolide
bisa 2 nd or 3rd generation ini CAP
C. 1st generation cephalosporin and aminoglycoside
D. 2nd generation and macrolide
E. Aminoglycoside and macrolide

77. A 70-year-old male was presented with low mentation and severe shortness of breath.
His condition was associated with hoarseness, dysphagia, and stridor. He had been diagnosed
with nasopharyngeal carcinoma for the last 6 months, confirmed by contrast CT scan. On
physical examination, he was semiconscious with GCS E2M4MT, cachexic, dehydrated,
dyspnea, with inspiratory stridor. In view of the patient's clinical condition, an emergency
tracheostomy under local anesthesia was performed followed by direct endoscopic
examination which revealed irregular mass involving the false and true vocal cords, anterior
commissure, laryngeal surface of the epiglottis, with subglottis extension.
What is the most likely physiological mechanism can still occur in this patient who
breathes air through a tracheostomy?
A. Air warming process
B. Air humidifying function
C. Partially air filtration process
D. Mucous gland secretion
E. Articulation function

78. A 50-year-old woman is admitted to the hospital with a swollen right calf. She has
history of surgical cholecystectomy 10 days ago. Physical and diagnostic findings reveal a deep
vein thrombosis. On the next day, she suffered a mild-to-moderate dyspnea. No history of
dyspnea or respiratory before. No further pathological clinical finding. Chest X-ray shows clear
lungs without infiltrates.
What is most probable working diagnosis of the problems?
A. COPD
B. Pulmonary embolism
C. Tuberculosis infection
D. Pneumonia
E. Lung cancer

79. What structure serves as the floor of the affected sinus above?
A. Toward the zygomatic bone maxillary sinus
apex

B. The alveolar part of the maxilla


C. The floor of the orbit
roof

D. The lateral wall of the nasal cavity


base
E. The superior part of the orbit

80. A 30-year-old female presented to the Emergency Departement with complaints of


high fever, cough and dyspnoea since 3 days ago. Her mother was on medication for
tuberculosis. On examination, chest percusion is dull, with ronchi heard on auscultation. The
laboratory examination revealed leucocytosis.
Chest radiographic examination showed opacification in the right lower lung zone , air
bronchogram +, meniscus sign + with no enlargement of the heart.
What is the most likely diagnosis of this patient?
A. Tuberculosis
B. Pneumonia
C. Pleuropneumonia
D. Pleural effusion
E. Brochopneumonia

81. A 44-year-old woman preAisign


sr bebecause
nntcehodgraitw
ro m dietsh
cribiensc
garsewaits
hie
nda brs
ohnco
hurstth
naet iss ssurorofundberdebyaatlhve,olicfiloleduwgithh f,luiad,npuds osr pothuetr umam
terial. It is a very useful
is highly sensitive and specific for the presence of consolidation rather than collapse.
pathophysiology
production. She had asthma sincegas-filled
chilbronchus
dhoodinside and used her
a consolidated lung medications as directed. On the
p us/fl uid/c ells wi th in the a lveo lar spaces
examination, she was in moderattheebronchi
respofteni raremain
toryaerateddistress, respiration rate was 26x/min. There were
proves pulmonary consolidation

bilateral wheezes, and oxygen saradiographic


utsu
efr
ulawthieo
nndiffew
renatis
features
atin9
g0fro%
m coo
llan room air. On her blood gas, the pCO2 was
pse

50 mm Hg. chest x-ray

b
l
a
c
k
g
a
s
-
f
i
l
l
e
d
b
r
o
n
c
h
u
s
r
u
n
n
i
n
g
t
h
r
ough opacified lung air bronchograms
help to differentiate:

Select the most likely mechanismairsapascetohpeacifcicaatiuons(econosoflidtahtioins) patient's carbon dioxide retention:


atelectasis (collapse) doesn't have air bronchograms

A. Impaired diffusion syndroplem


urae s doesn't have air bronchograms
l fluid

B. Right-to-left shunt
C. Hyperventilation
D. Ventilation-perfusion ratio inequality
E. Decreased lung compliance

82. Pulmonary TB patient with negative AFB (acid fast bacilli) was treated with
antituberculosis drugs for less than 1 month previously.
Currently the most likely choice of anti-tuberculosis regiment is
A. 2 R7H7E7Z7 / 4 R3H3.
pasien baru

B. 2 R7H7E7Z7S7 / 1 R7H7E7Z7 / 5 R3H3E3. relaps, gagal, default

C. 2 R7H7Z7 / 4 R3H3.
D. 2 S7H7E7 / 10 H3E3.
E. 2 S7H7Z7 / 10 S7H7.

83. Which of these sentences are correct about the nasal septum?
A. It divides the nasal cavity into four chambers
B. Only made up of bony part
C. Completely made up of cartilaginous part
D. It is usually deviated by lateral blow
The internal nasal septum separates the
E. It has minimal vascular supply nasal cavity into two nostrils. The bones
that contribute to the nasal septum can
be divided into:

P
a
i
r
e
d
b
o
n
e
s
:
N
a
s
a
l
,
m
a
x
i
l
l
a
r
y
a
n
d
p
a
l
a
t
i
n
e
b
o
n
e
s
U
n
p
a
i
r
e
d
b
o
n
e
s
:
E
t
h
m
o
i
d
a
n
d
v
o
m
e
r
b
o
n
e
s
.

In addition to the bones of the nose,


the septal and greater alar cartilages
also constitute part of the nasal
septum.
84. 50-years-old labour hospitalized because of exacerbation of Chronic Obstructive
Pulmonary Disease. At the admissionday he got shortness of breath (RR 40x/min) and fever
(Temp 38,9 0C). He is now on the 3rd day of hospitalization and he still feel shortness of breath
but the respiratory rate decreased to 26x/min. He has no fever. There are minimal sputum
retension. He can sit independently but still feel tired to walk. His previous spirometry test
reveals FEV1/FVC 60%
Which of the lung capacity that most likely to increased in this patient?
A. Tidal volume
B. Residual volume
C. Inspiratory reserve volume Obstruktir : RV, FRC, TLC naik semua, ratio turun

D. Inspiratory capacity restriktif : RV, FRC, TLC turun semua, ration normal or
naik

E. Expiratory reserve volume

85. Bacteria or their components may be characterized by unique con¬stituents


or structures.Streptococcus pneumoniaeisa bacteria which is most likely
characterized by
A. hyaluronic acid.
B. outer-membrane proteins.
C. repeating polysaccharide capsule of glucose and glucuronic acid.
D. sialic acid polymers.
E. Υ-Glutamyl polypeptide.

86. Select the most likely reaction which is catalyzed by the carbonic Streptococcus
anhydrase enzyme as
pneumoniae is characterized by a polysaccharide

capsule that completely encloses the cell, and


blood passes through systemic capillaries : plays a key role in its virulence. The cell wall of
S. pneumoniae is composed of peptidoglycan,
A. Conversion of dissolved carbon dioxide to carbonic acid with teichoic acid attached to every third

B. Conversion of carbonic acid to bicarbonate ion N-acetylmuramic acid, and is about 6 layers thick.
Lipoteichoic acid is attached to the membrane via a
lipid moiety, and both teichoic and lipoteichoic acid
contain phosphorylcholine. Two choline residues
may exist on each carbohydrate repeat, which is
important to S. pneumniae

C. Conversion of gaseous carbon dioxide to dissolved carbon dioxidehuman


because the choline
cells. ( adheres to choline-binding receptors located on

D. Binding of carbon dioxide to haemoglobin, thus displacing oxygen


E. All of the above
87. A 55-year-old man was brought to the emergency department because he had difficulty
in breathing which was worsening for the past 2 days. He also had cough with thick and
greenish sputum. The patient had already suffered dyspnea for the last 4 years which occurs
intermittently, and been several times hospitalized.
Physical examination showed that his respiratory rate was 36x/minute, prolonged expiratory, and
wheezing was heard all over his chest in auscultation. Crackles were not detected.
As initial treatment, the patient would be given oxygen therapy. However, its concentration and
his response should be evaluated carefully, since inappropriate oxygen therapy could result in
respiratory depression in the patient with chronic dyspnea.
What is the most powerful respiratory stimulus for breathing in this patient compared to a
healthy person?

in normal person
Increase of carbon diolevxelsidtheat

In individuals with chronic obstructive


pulmonary disease who receive
supplemental oxygen, carbon dioxide
accumulation may occur through three
main mechanisms:[5]
88. A 30-year-old female patient diagnosed to have tuberculosis and was started on treatment
Ventilation/perfusion matching: under-
for 2A.months. Currently she complained of numbness of both hand.
Ethambutol ventilated lung usually has a low oxygen
content which leads to localised
vasoconstriction limiting blood flow to that
B. Pyrazinamide lung tissue. Supplemental oxygen
abolishes this constriction, leading to poor
C. Rifampicin ventilation/perfusion matching. This

D. Isoniazide
Select the most likely anti tuberculosis drug which cause the neuropathy.
redistribution of blood to areas of the lung
with poor ventilation reduces the amount of
carbon dioxide eliminated from the system.

The Haldane effect: most carbon dioxide is


carried by the blood as bicarbonate, and
deoxygenated hemoglobin promotes the
production o bicarbonate. Increasing the
amount of oxygen in the blood by
administering supplemental oxygen reduces
the amount of deoxygenated hemoglobin,
and thus reduces the capacity of blood to
carry carbon dioxide.

Respiratory homeostasis: in healthy


individuals, a rise in carbon dioxide
causes an increase in the drive to
breathe. However, in some patients with
chronic obstructive pulmonary disease,
this response has been blunted, leaving
low oxygen levels as the main stimulus of
respiration (hypoxic drive). Hence, giving
supplemental oxygen reduces their
stimulus to breathe, causing respiration to
slow (hypoventilation), and allowing
carbon dioxide to accumulate in the body.

makanya harus di kombinasi dg vit B6/ piridoksin

E. Streptomycine

89. A 5-year-old boy was brought by his parents to the emergency room with the chief
complain of noisy breathing for the last 2 days. His condition was associated with barking
cough, mild fever, and difficulty to breath. However there was no difficulty in swallowing and
no history of allergy.
What is the most likely cause of the patient's chief complain?
A. Free liquid in the airway lumen
B. Narrowing of the upper airway
C. Narrowing of the lower airway
Movement of the airway to open-craounpd= scublgolosttiec, biantasorersdpieatrast-boawaahcgcloottim
s?
D. modate air
movement during inspiratory and expiratory phases
E. Hypersecretion of the airway mucous gland

90. Select the most likely factor that is decreased and responsible in a
tachypnoea
to cause myocardial infarction patient.
A. Blood pH
B. Blood lactate
C. Blood CO2
D. Blood Fe2+
E. Blood lactate
91. A 30-year-old female presented to the Emergency Departement with complaints of
high fever, cough and dyspnoea since 3 days ago. Her mother was on medication for
tuberculosis. On examination, chest percusion is dull, with ronchi heard on auscultation. The
laboratory examination revealed leucocytosis.
Chest radiographic examination showed opacification in the right lower lung zone , air
bronchogram +, meniscus sign + with no enlargement of the heart.

Select the most likely second choice of radiograph ic examination


A. Chest fluoroscopy for pleural
B. Chest utrasonography effusion.
C. Chest CT scan
D. Chest MRI
Depending on the clinical context, ultrasonography or
computed tomography (CT) scanning can be used to confirm
a pleural effusion, especially in cases of loculated pleural
effusion, complete opacification of hemithorax, or associated
E. Chest colour doppler ultrasonography lung parenchymal abnormalities. Ultrasonography and

CT scanning are more accurate than chest radiography in identifying the underlying etiology. Ultrasonography or CT scanning can be used to
guide thoracocentesis or catheter drainage of effusions. [18] Thoracocentesis is primarily performed under ultrasonographic rather than CT scan
guidance.

92. In patient with dyspnea, blood gas analysis is often performed to measure the acidity
and the arterial levels of oxygen and carbon dioxide. This examination checks how well the
lungs are able to move oxygen into the blood and remove carbon dioxide from the blood. Thus
the result of blood gas analysis is related to the transport mechanism in the blood.
Select the most likely statement which is true about carbon dioxide transport in blood :
A. Its concentration in the blood is increased by hyperventilation
hipoventilasi
B. Its accumulation in the blood is associated with an increase in pH
decrease
C. More carbon dioxide dissolves in the blood plasma than is
carried in the RBCs kebalik. yg trbanyak HCO3-, carbamino, dissolved in
plasma.

D. More carbon dioxide carried in RBCs, binds to hemoglobin,


forming

carbaminohemoglobin than is converted to carbonic acid by carbonic anhydrase


E. HCO3- moves out of RBC in exchange for Cl- (chloride shift)

93. A positive-smear acid fast bacilli mother household


living with who he E. Immunization
seems look healthy. history
What is the most essential factor in determining the
A. Direct contact
B. Nutritional status
C. Socio-economic status
D. Past medical history
her 32months old son in the same

in this patient? risk of transmission


94. Which of these factors is the most responsible for transfering O2 from a pregnant
woman to her foetus?
A. Blood pH
B. Blood O2
C. Blood CO2
D. Blood Hemoglobin.
E. Blood 2,3 biphosphoglycerat (BPG, DPG)

95. A 5-year old boy brought to the ER by his parents. They say he was playing with beads,
and one of them stuck in his nose.
Where would it lodged most probably?
A. Between the concha inferior bone and nasal septum
B. Between the superior and middle conchae
C. Between the ethmoid sinus and sphenoid sinbuetween
s middle and inferior chonchae
D. Between the sphenoethmoidal recess and the cribiform plate
E. Between the alar cartilage and sphenoethmoidal recess

96. A 50-year-old woman is admitted to the hospital with a swollen right calf. She has
history of surgical cholecystectomy 10 days ago. Physical and diagnostic findings reveal a deep
vein thrombosis. On the next day, she suffered a mild-to-moderate dyspnea. No history of
dyspnea or respiratory before. Chest X-ray shows clear lungs without infiltrates.
Which one is the most suitable diagnostic finding for the working diagnosis?
A. Increase glucose uptake by malignant cells
B. Uptake of ethambutol by mycobacterium
C. Matching defect
D. Mismatch defect
E. Decrease glucose uptake by malignant cells

97. Coronaviruses are recognized by club-shaped surface projections that are 20 nm long
and resemble solar coronas. These viruses are best characterized by their ability to
A. agglutinate human red blood cells.
B. cause common cold.
C. grow profusely at 50°C.
D. grow well in the usual cultured cell lines.
E. infect infants more frequently than adults.

98 Which of the following is a normal occurrence with increasing age?


. A. Vital capacity of the lung increased
B. Residual volume decreased Vital
capacity
declines
progressiv
ely with
age. There
is a linear
loss of 5%
to 20% of
functional
ability per
decade.
From age
20, vital
capacity decreases progressively

C. Functional residual capacity decreased


The increased outward pull of the stiffer chest wall combined with the reduced ability of the lung to pull

inward results in a small increase in functional residual capacity (the volume at which the lung comes to rest at the
end of a quiet expiration) and residual volume (the volume that remains in the lung after a maximal expiration)
user

2017-12-05 06:55:09

--------------------------------------------

D. Inspiratory capacity increased Anatomical


E. Expiratory reserve volume decreased Air space size Increased

Compliance
99. This virus is a double-stranded respiratory
DNA infections and virus. It is
Select the most likely virusacute
described Chest wall compliance D e
ra
csd
diarrhoe responsible for
A. Adenovirus approximately
Lung compliance Increased to u
d
e
g
a
h
c
n
B. Influenza virus 15%complianceof a in
Total respiratory system
C. Measles virus Decreased chil¬dren.
D. Parainfluenza virus
Muscle strength
E. Respiratory syncytial virus
Maximal inspiratory pressure (M
IP
)
Decreased

100. This virus is a member of the family Picornaviridaeand the genus Enterovirus
Trans diaphragmatic pressure (Pdi) , single
Decreased
stranded-RNA (ssRNA) viruses of approximately 7,200 bp and usually cause common cold.
Select the virus described. Maximum voluntary ventilation (M V)
A. Cytomegalovirus Decreased
B. Paramyxovirus Lung function
C. Lyssa virus
FEV1 Decreased
D. Respiratory syncytial virus
E. Rhinovirus FVC Decreased

TLC Unchanged
101. A 50-year-old man with a chronic respiratory problem was admitted
Vital capacity to the hospital because
Decreased
of increasing shortness of breath and sputum production. He was started on antibiotics,
Functional residual capacity Inc
reas
ed
inhaled bronchodilators, and anticholinergic agents. The next day he was found confused
Residual
and sleepy in his room. A pCO2 examination revealed Volume
severe Increased
hypercarbia (pCO2 75 mmHg).
DLCO/VA Decreased
Select the most likely explanation which causes his elevated pCO2 :
Exercise capacity

A. Occurs only with CO2 inhalation VO2 max Decreased

B. Can result in respiratory alkalosis Dead space ventilation Inc


reas
ed
C. Normally, may lead to hypoventilation compensatory mechanism
Immunology
D. May worsen with oxygen administration
Bronchial fluid
E. Occurs with chronic hypocapnia
Neutrophils % Increased
102. Select the strongest stimulator of respiratory centre in the brain:
Ratio of CD4+/CD8+ cells Inc
reas
ed
A. Blood pH
B. Blood O2
C. Blood CO2
D. Blood lactate
E. Blood haemoglobin
103. A 7-year-old boy was brought by his mother to the primary health care for difficulty in
swallowing for the past 2 days. He could only consume s H
condition was associated with fever, sore throat, and snoring while sleepin i
s
emisolid food and fluid.
g. For the last one
year, he has been having recurrent sore throat. There were no associated ear discharge,
earache, runny, obstructed nose, and hoarseness.

Select the most likely swallowing stage that is affected:

A. Oral stage
B. Pharyngeal stage
C. Oropharyngeal stage
D. Esophageal stage
E. Voluntary stage

104.

What is the characteristic of the structure pointed by number 17?


sphenoid sinus

A. It has drainage leading to the semilunar hiatus


B. It is evenly divided by thick bony septum
C. It is located inside the maxilla, and comes as a pair
D. It is located in the body of sphenoid, and may extend to its wings
E. It is the largest of all paranasal sinuses and occupy the ethmoid bone

105. A 4-years old boy brought to the outpatient clinic by his mother because for the past hour
his nose had been bleeding. She said that she had tried to stop the bleeding by pinching his
nose. Her son had constant fever for 3 days, and it doesn't subside with fever medicine she
gave him. There were red spots in his arms and legs since yesterday.

Which structure(s) were bent when the mother did the maneuver to stop the bleeding?

A. The septal cartilage


B. The alar cartilages
C. The nasal bones
D. The collumella
E. The dorsum

106.
A 6-week-old boy admitted to Pediatric ER with difficulty of breathing, fever and cough for
the past 2 days. The diagnosis of pneumonia was made.
Select the appropriate treatment for this patient:

A. IV Ampicillin
B. IV Ampicillin + IV Gentamicin
C. IV Cefotaxim
D. IV Chloramphenicol
E. IV Gentamycin
In children younger than five years of age, initial treatment of pneumonia includes IV ampicillin or nafcillin
plus gentamicin or cefotaxime (for neonates). Ceftriaxone or cefotaxime can be administered as a single
agent (for >28 d to 5 y). An alternative regimen includes a penicillinase-resistant penicillin plus an
antipseudomonal aminoglycoside.
107. Some of our body mechanism to prevent the entry of foreign body are by coughing and
sneezing. The cough reflex is very much like the sneeze reflex.

What is the most likely similarity between cough and sneeze reflex?

A. Their center are in the pons


medulla
B. Their initiating stimuli irritate receptors in the lower respiratory passages
zneeze di upper
C. The afferent impulses pass through the vagus and glossopharyngeal nerves
ini cough, kalo sn
D. A series of reaction involved, consists of epiglottis and vocal cord movement eeze trigeminal

mechanism, and the respiratory muscles involved


E. Both of the reflexes involve uvula depression

sneeze aja

108. A 25-year-old woman came to ER with fever and dyspnoea. She has a positive history of
tuberculosis. On examination she was found to have pleural effusion.

What is the best clinical sign elicited in this patient?

A. Crackles on auscultation
B. Dullness on percussion
C. Hyper-resonance on percussion
D. Increase in vesicular breathing sound
E. Ronchi on auscultation
109. A 50-year-old man was a-known case of Chronic Obstructive Pulmonary Disease. He
smoked about 28 pack years of cigarettes. His body weight was 44 kg and his height was
165 cm.

Which of the following is the most likely consequence for this patient if thediet is not
adjusted accordingly?

A. Obesity
B. Edema
C. Muscle wasting
D. Under nourished
E. Anorexia

110. A man was brought to the Emergency Unit. The doctor did the physical examination, and
declared that he was dead. On examination, a horizontal ligature mark was seen encircling
the neck, presence of conjunctival petechial haemorrhages, and congestion of the face. The
body was then sent to Forensic Medicine Department to undergo autopsy. Autopsy findings
revealed Tardieu's spot, cerebral hypoxia, and fracture of the superior horn of the thyroid
cartilage.

What is the most likely factor related to pressure on the neck that may be of legal relevance
considering the defendant's actions?

A. Body weight
B. Demarcation of congestion
C. Length of time
D. Location of the ligature mark
E. Profound bradycardia

111. A 65-year-old male, came to the clinic complaining of breathlessness on climbing stairs in
his house and associated with increase in severity with time. He also has productive thick
whitish sputum. Spirometry examination revealed that FEV1/FVC less than 70% and FEV1
47% than predicted.

For the above patient, select the most likely drug that will play a central role in the
management.

A. Antibiotics
B. Corticosteroids
C. Bronchodilator
D. Cromones
E. Antihistamines

112. Select the main blood transport system of CO2 and its explanation:

A. Dissolved CO2 because CO2 will dissolve easily in the plasma


B. Dissolved CO2 because CO2 will be secreted easily by the lung
C. HCO3- ion because the red blood cell contains a lot of carbonic anhydrase
D. HCO3- ion because its increased level will not change the blood pH dramatically
E. Carbamino haemoglobin because haemoglobin is found abundantly in the blood
system

113.

The structure of B lining the alveoli. Choose the best characteristic that suit for structure B:

A. Ciliated columnar cells; takes part in gas exhange


B. Flattened simple squamous epithelium with prominent nuclei; produces surfactan
C. Flattened simple squamous epithelium with nuclei that infrequently seen; takes
part in gas exchange
D. Rounded shape with nuclei that infrequently seen and vacuolated cytoplasma;
produces surfactan
E. Rounded shape with prominent nucleus and vacuolated cytoplasma; takes part in
gas exhange

114. A 50-year-old woman is admitted to the hospital with a swollen right calf. She has
history of surgical cholecystectomy 10 days ago. Physical and diagnostic findings reveal a
deep vein thrombosis. On the next day, she suffered a mild-to-moderate dyspnea. No
history of dyspnea or respiratory before. Chest X-ray shows clear lungs without
infiltrates.

What further less invasive work up suitable for the case?

A. F-18 FDG PET/CT Scan


B. Doppler Ultrasound untuk cancer

C. V/Q Study
D. Tc-99m Ethambutol Scan
E. Pulmonary Function Test

115. Generally airflow limitation in COPD is irreversible, however there are some factors that
can improve airflow limitation if adequately treated.

Select the most likely factor.

A. Fibrosis and narrowing of airway


B. Loss of elastic recoil
C. Alveolar destruction
D. Peripheral and central airway smooth muscle contraction
E. Destruction of greater airway

116. A 64-year-old man came to a clinic for follow-up of his chronic congestive heart failure. He
has a marked reduction in his ejection fraction following a series of myocardial infarctions.
His symptoms include dyspnea on exertion, paroxysmal nocturnal dyspnea, orthopnea, and
peripheral edema. He has normal renal function. He is on treatment with twice daily of ACE
inhibitor and once daily of furosemide. You decide to add digoxin to his regimen.

Which is the most appropriate action of furosemide in this patient?

A. It reduces ventricular preload


B. It reduces peripheral resistance
C. It reduces afterload
D. It increases cardiac contractility
E. It reduces heart rate

117. An 8-month-old baby boy admitted to Pediatric ER with the history of fever and cough for
the past 4 days. His symptoms worsen with difficulty in breathing for the past 2 days. On
physical examination, the baby was alert, respiratory rate: 60 beats/minute, temperature:
38,7oC. There was chest in-drawing with crackles heard on auscultation. The leukocytes
count was 20,500/mm3 and chest x-ray showed lung consolidation at the upper right lobe.

What is the most likely diagnosis for this patient?

A. Asthma
B. Bronchiolitis
C. Croup
D. Pleural effusion
E. Pneumonia

118. Treatmentforpneumonia involves curing the infection and preventing complications. Select
the most likely factor which is not a criteria for admission for pneumonia patients.

A. Age > 65 years


B. Patients with co morbid factors
C. Immunosupresive treatment
D. Respiratory rate > 30/m
E. Blood pressure 120/80 mmHg 1. Skor PORT lebih dari 70
2. Bila skor PORT kurang < 70 maka penderita tetap perlu dirawat inap bila dijumpai s
alah satu
dari kriteria dibawah ini.

• Frekuensi napas > 30/menit


• Pa02/FiO2 kurang dari 250 mmHg
119. • Foto toraks paru menunjukkan kelainan bilateral
• Foto toraks paru melibatkan > 2 lobus
Tekanan sistolik < 90 mmHg
Tekanan diastolik < 60 mmHg
3. Pneumonia pada pengguna NAPZA

Which number is pointed to the structure that supsepglmieentsal sbreongchm


i??ents in lung?

A. 4
B. 5
C. 6
D. 7
E. 8

120. A 62-year-old man was being managed in the intensive care unit
following a large anterior wall myocardial infarction. He had been
appropriately managed with oxygen, aspirin, nitrates, and
but developed β- episodes of ventricular
recurrent
adrenergic receptor blockers
tachycardia. During these episodes he remained conscious but He was also
feeling dizzy.

diaphoretic and hypotensive.

Which is the most appropriate drug for this patient?


A. Verapamil ada
svt, ca channel blocker nanti makin BP naik
B. Digoxin
C. Lidocaine
D. Dopamine
E. Dobutamine
obat yg menyebabkan hipotensif

121. Which of the following biomolecules which is most likely unable to function normally in
the case of abnormal blood pH that can lead to death?

A. Lipid
B. Fibre
C. Protein
enzyme??
D. Nucleic acid
E. Carbohydrate

122. Inhaled corticosteroids are glucocorticoids that bind to the glucocorticoid receptors in the
airways and cause a reduction in lung inflammation. The role of inhaled corticosteroids in
COPD is controversial.

Select the most likely situation in COPD that glucocorticoid inhalaygtipoaknai ICisS ngruoptCDinwdhicihciac stteagde.3-4

A. Stage II COPD patients.


B. FEV1 < 30% predicted.
C. FEV1/FVC < 70%.
D. Recurrent exacerbations which need bronchodilator treatment.
E. Recurrent exacerbations which need glucocorticoid treatment.
122. Inhaled corticosteroids are glucocorticoids that bind to the glucocorticoid receptors in the
airways and cause a reduction in lung inflammation. The role of inhaled corticosteroids in
COPD is controversial.

Select the most likely situation in COPD that glucocorticoid inhalation is not indicated.

A. Stage II COPD patients.


B. FEV1 < 30% predicted.
C. FEV1/FVC < 70%.
D. Recurrent exacerbations which need bronchodilator treatment.
E. Recurrent exacerbations which need glucocorticoid treatment.
123. A 7 year-old girl was brought to the clinic complaining of high fever for 2 days. Her
parents said that she has pain when swallowing and also snores when sleeping
larynx

What nerve contribute to the nerve plexus supplying the area?

A. CN VII and IX
B. CN V3and IX
C. CN V3and X
D. CN IX and X
E. CN V2 and V3

124. A 42 year old man came to the outpatient department complained that his throat feels sore
since 2 days. Since 1 day ago, his voice became hoarse. He had mild fever and general
malaise. After checking his vital signs and performed head examination, the doctor
examined his throat. He noticed the prominent Adam's apple, as he palpate them, he asked
the patient to swallow

Which nerve controls the maneuver that the patient had done?

A. The vagus nerve


B. The recurrent laryngeal nerve
C. The inferior laryngeal nerve larynx

D. Mixture supplies from CN V, VII, XII


E. The external laryngeal nerve

125. The lung is an elastic structure that can inflate and deflate to expel its air through the trachea.
The air can move in or out of the lung because of pressure difference, as the result of thoracic
cavity volume adjustment by contracting or relaxing respiratory muscles.

Select the most likely statement which is true about properties of the lung:

A. Direction of elastic recoil of lungs is outward


B. Chest wall has elastic recoil which pulls the thoracic cage inwardly
C. Intrapleural pressure is negative relatively as the result of the interaction of lungs
and thorax elastic recoil
D. Normally, lungs do not have tendency to collapse
E. Alveolar pressure is less than atmospheric pressure when the glottis is open and
no air is flowing into or out of the lungs

126. A 10-year-old boy came to the Pediatric clinic with chronic cough and decrease in
body weight. His father was on tuberculosis treatment for the past 2 weeks. What is the
most appropriate examination for this patient?
a. Tuberculin skin test
b. Tuberculin skin test and chest x-ray
c. Tuberculin skin test, chest x-ray, and sputum examination
d. Tuberculin skin test, chest x-ray, sputum examination, and erythrocyte
sedimentation rate
e. Tuberculin skin test, chest x-ray, sputum examination, erythrocyte sedimentation rata,
and PCR

127.
A 46-year-old male presented to the Emergency Department with complaint of dyspnoea
several hours after a road trafic accident . On examination, there were several injuries on
the right anterior chest and anterior abdominal wall. Chest radiographic examination
revealed fracture of the right 4th,,5th,and 6th ribs at three different sites anteriorly . There
were le ft mediastinal shift , infiltration in the right lung , absence of pleural line , and

meniscus = ada efusi opacification with meniscus sign in both lower hemithorax

128.
Which structure is a bony
prominence comprised of an individual bone?
a. Pointed by number 7
b. Pointed by number 5
c. Pointed by number 9
d. Pointed by number 17
e. Pointed by number 8

129.
A man was brought to the Emergency Unit. The doctor did the physical examination, and
declared that he was dead. On examination, a horizontal ligature mark was seen encircling
the neck, presence of conjunctival petechial haemorrhages, and congestion of the face. The
body was then sent to Forensic Medicine Department to undergo autopsy. Autopsy findings
revealed Tardieu's spot, cerebral hypoxia, and fracture of the superior horn of the thyroid
cartilage.
select the most likely autopsy finding which indicates carotid obstruction due to pressure
on the neck.
a. Cerebral hypoxia
“Tardieu spots” are the result of intense lividity,
b. Congestion of the face leading to postmortem rupture of dependently
engorged blood vessels, entirely
c. Conjunctival petechial haemorhages unrelated to asphyxia or any other mechanism of death.
d. Fracture of superior horn of the thyroid cartilage

e. Tardieu’s spot

130.
Which is the most likely factor responsible for converting T-form haemoglobin to R-form
haemoglobin?
a. Blood pH
b. Blood O2
c. Blood CO2
d. Blood 2,3-BPG
e. Blood Fe2+

131.
A 7 year-old girl was brought to the clinic complaining of high fever for 2 days. Her
parents said that she has pain when swallowing and also snores when sleeping
Which of the following is most commonly affected?
a. Lingual tonsil
b. Tubal tonsil
c. Arytenoid tonsils
d. Uvular tonsil
e. Palatine tonsil

132.
7-year-old boy was brought by his mother to the primary health care for difficulty in
swallowing for the past 2 days. He could only consume semisolid food and fluid. His
condition was associated with fever, sore throat, and snoring while sleeping. For the last
one year, he has been having recurrent sore throat. There were no associated ear discharge,
earache, runny, obstructed nose, and hoarseness.

Pharyngeal stage of swallowing is initiated by stimulation of epithelial swallowing


receptor area in the pharyngeal opening, especially in the tonsillar pillars. After that, the
impulse is transmitted into the swallowing center in the lower medulla and pons through:

a. CN V and CN IX
b. CN V and CN X
c. CN IX and CN X
d. CN X and CN XI
e. CN IX and CN XI

133.
A 62-year-old man was being managed in the intensive care unit following a large anterior
wall myocardial infarction. He had been appropriately managed with oxygen, aspirin,
nitrates, and β-adrenergic receptor blockers but developed recurrent episodes of ventricular
tachycardia. During these episodes he remained conscious but feeling dizzy. He was also
diaphoretic and hypotensive.

Which of the following is/are the adverse effect(s) of lidocaine?


a. Agranulocytosis, leucopenia
b. Extrapyramidal disorders
c. Hypotension, paresthesias, convulsions
d. Bronchospasm , dyspepsia
e. Headache

134.
Human is able to produce sounds by physiological process of vocal apparatus. Select the
most likely vibrating vocal apparatus which produces sounds:
a. False vocal cords
b. Vestibular folds
c. True vocal cords
d. Intrinsic laryngeal muscles
e. Laryngeal ligament

135.
Unlike inspiration, expiration is a passive movement because no muscular contractions
are involved. Expiration, however, depends on two factors.
Which of the choices below lists those two factors?
a. The recoil of elastic fibers that were stretched during expiration and the outward
pull of surface tension due to the film of alveolar fluid
b. The expansion of respiratory muscles that were contracted during inspiration and
the lack of surface tension on the alveolar wall
c. The negative feedback of expansion fibers used during inspiration and the
outward pull of surface tension due to surfactant
d. The recoil of elastic fibers that were stretched during inspiration and the inward pull
of surface tension due to the film of alveolar fluid
e. Combined amount of CO2 in the blood and air in the alveoli, inhibit the respiratory
drive resulting in expiration

136.
A 50-year-old man was a-known case of Chronic Obstructive Pulmonary Disease. He
smoked about 28 pack years of cigarettes. His body weight was 44 kg and his height was
165 cm.
Which of the following is the best nutritional composition for this patient?
a. High carbohydrate and high protein.
b. High carbohydrate and low fat.
c. Low carbohydrate and high fat.
d. Low carbohydrate and high protein.
e. Low carbohydrate and low fat.
137.
60- years-old labourhospitalized because of exacerbation of Chronic Obstructive
Pulmonary Disease. At the admissionday he got shortness of breath (RR 40x/min) and
fever (Temp 38,9 0C). He is now on the 3rd day of hospitalization and he still feel
shortness of breath but the respiratory rate decreased to 26x/min. He has no fever.
There are minimal sputum retension. He can sit independently but still feel tired to
walk. His previous spirometry test reveals FEV1/FVC 60%

Which muscle should be the most active in his effort to overcome the shortness of
breath?

a. Diaphragm
i nspirati o n
b. External intercostal
inspiration
c. Internal intercostal
d. Sternocleidomastoid
e. Pectoralis Major inspiration

138.
A 50-year-old-year man was hospitalized following two months' history of haemoptysis
and intermittent fever. Histopathological examination confirmed a diagnosis of
granulomatous fungal infection in the lungs.
Select the most likely group of fungi causingthe gran¬ulomas.
a. Aspergillus, Coccidioides, Cryptococcus
b. Cladosporium, Aspergillus, Microsporum
c. Coccidioides, Blastomyces, Histoplasma

139.

In normal condition, elastic recoil of chest wall can be controlled by adjusting the tension
of respiratory muscles, such as primary muscles of inspiration, accessory muscles of
inspiration, and accessory muscles of expiration.
Select the most likely respiratory-associated muscles which would contract if you blow
up a balloon.
ekspirasi paksa

a. External intercostal, scalene, and pectoralis minor


b. External intercostal and abdominal muscles
c. Abdominal muscles, scalene, pectoralis minor, and diaphragm
d. Internal intercostal and abdominal muscles
e. Internal intercostal, scalene, and pectoralis minor
140.
A 45-year-old male, a known case of TB, was admitted to ER with severe dyspnoea, which
suddenly occurred about half an hour prior to explosive coughs. Physical examination
revealed his blood pressure was 80/50mmHg with elevated jugular venous pressure. On
percussion showed hypersonor on the right chest.
What is the appropriate initial treatment for this patient?
a. Insertion of chest tube
b. Needle aspiration drainage
c. Needle chest decompression
d. Endotracheal intubation
e. Thoracotomy decompression

141.
A 7 year-old girl was brought to the clinic complaining of high fever for 2 days. Her
parents said that she has pain when swallowing and also snores when sleeping
Upon physical examination, which lymph node would you find enlarged?
a. Tonsilar lymph node
b. Superficial cervical lymph node
c. Submandibular lymph node
d. Submental lymph node
e. Deep cervical lymph node

142.

Which structure is lined by the skin and serves as entrée to the nasal cavity proper?
a. Pointed by number 13
b. Pointed by number 7
c. Pointed by number 20
d. Pointed by number 9
e. Pointed by number 17

143.
A man was brought to the Emergency Unit. The doctor did the physical examination, and
declared that he was dead. On examination, a horizontal ligature mark was seen encircling
the neck, presence of conjunctival petechial haemorrhages, and congestion of the face. The
body was then sent to Forensic Medicine Department to undergo autopsy. Autopsy findings
revealed Tardieu's spot, cerebral hypoxia, and fracture of the superior horn of the thyroid
cartilage.
What is most likely cause of death?
a. Accident
b. Hanging
c. Homicide
d. Strangulation
e. Suicide

144.
Select the most likely time in the respiratory cycle which the intrapleural pressure is most
negative :
a. Just after the beginning of inhalation (inspiration)
b. Just before the end of inhalation
maximum inspiration , dari -5 mmH2O jadi -8mmH2O
c. Just after the beginning of exhalation (expiration)
d. Just before the end of exhalation
e. Any of the above, since the intrapleural pressure is constant during the
normal respiratory cycle

145.
A 35-year-old man has a vital capacity of 5 L, a tidal volume of 0.5 L, an inspiratory
capacity of 3.5 L, and a functional residual capacity of 2.5 L.
What is the most likely his expiratory reserve volume?
a. 4.5 L
b. 2.5 L
c. 2.0 L
t
d. 1.5 L o
t
a
e. 1.0L l
l
u
n
g
c
a
p
a
c
i
t
y
:
i
n
s
p
iratory capacity + functiona residu V
TLC = 3,5 + 2.5 =6 L =
5
-
V 3
C .
= 5
I =
C 1
+ .
E 5
R L
V
E
R

146.
COPD patients requires special adjustment in terms of carbohydrate, fat and protein
compositions in their diet.
Which of the following Respiratory Quotient (RQ) is the most correct?
a. Carbohydrate 0,8, protein 0,7 and fat 1
b. Carbohydrate 0,7, protein 1 and fat 0,7
c. Carbohydrate 0,7, protein 1 and fat 0,8
d. Carbohydrate 0,7, protein 0,8 and fat 1
e. Carbohydrate 1, protein 0,8 and fat 0,7

147.
A 60-year-old woman had a 6-month history of progressive shortness of breath on
exertion. She did not s moke.
Vital capacity 60 % - Pulmon
ary
function
findings
were
shown
below:
normal
predicte
d
- Forced expiratory volume in 1 second (FEV1) 70 % normal predicted
- Diffusing capacity for carbon monoxide 50 % normal predicted

- Maximum voluntary ventilation 60 % normal predicted


Select the most likely cause for her limited ability to increase ventilation :
a. Airway obstruction
b. Decreased activation of pulmonary juxtacapillary (J) receptors
c. Decreased lung compliance
d. Depression of central chemoreceptors
e. Depression of peripheral chemoreceptors

148.
State the blood pH disturbance which is most likely to occur in the case of severe
asthmatic attack?
a. No blood pH disturbance because lung does not produce any acid or base
b. No blood pH disturbance because the kidney can keep blood pH in its normal value
c. Increase of blood pH because increase in respiratory rate will excrete more CO2
d. Decrease of blood pH because lack of O2 in turn will produce more lactic acid
e. Decrease of blood pH due to increase of H2CO3

149.
A 46-year-old male presented to the Emergency Department with complaint of dyspnoea
several hours after a road trafic accident . On examination, there were several injuries on
the right anterior chest and anterior abdominal wall.
Chest radiographic examination revealed fracture of the right 4th,,5th,and 6th ribs at
three different sites anteriorly . There were left mediastinal shift , infiltration in the right
lung , absence of pleural line , and opacification with meniscus sign in both lower
hemithorax.

Select the most likely features of flail chest.


a. Lung contusion with pleural line.
b. Lung contusion with mediastinal shift.
c. Lung contusion with multiple ribs fractures. Flail chest is a life-threatening medical condition that
occurs when a segment of the rib cage breaks due to
trauma and becomes detached from the rest of the
chest wall. Two of the

d. sy mpt oms of flail che st are chest pain and shortness of breath.[1]
Hyperlucency of lung with pleural line and pleural effusion.
It occurs when multiple adjacent ribs are broken in multiple places, separating a segment, so a
e. Hyperlucency of lung with pleural line and mediastinal shift.
part of the chest wall moves independently. The number of ribs that must be broken varies by
differing definitions: some sources say at least two adjacent ribs are broken in at least two
places,[2] some require three or more ribs in two or more places.[3] The flail segment moves in
the opposite direction to the rest of the chest wall: because of the ambient pressure in
comparison to the pressure inside the lungs, it goes in while the rest of the chest is moving out,
and vice versa. This so-called "paradoxical breathing"[4] is painful and increases the work
involved in breathing.

150.
A 32-year-old woman came to primary health care with 10 days history of nasal discharge
and cough. The discharge was yellow-greenish and thick. Her condition was associated
with decreased of smelling sensation, sometime with nasal blockage, fever, headache, and
facial pain. Physical findings showed respiratory rate was 28x/minute and temperature was
38oC. On anterior rhinoscopy examination, both nasal cavities were hyperemic with
mucopurulent secretion and nasal turbinates were hypertophy. Pain was felt at right and
left maxilla region during palpation.
What are the most likely physiological changes happened in this patient?
a. Vasodilatation, ciliary beat increased, viscosity of mucus decreased
b. Vasoconstriction, ciliary beat increased, viscosity of mucus increased
c. Vasodilatation, ciliary beat increased, viscosity of mucus increased
d. Vasodilatation, ciliary beat decreased, viscosity of mucus increased
e. Ostia obstruction, vasoconstriction, ciliary beat decreased, viscosity of
mucus decreased

151. A 10-year-old boy came to the Pediatric clinic with chronic cough and decrease in
body weight. His father was on tuberculosis treatment for the past 2 weeks. His tuberculin
skin test was found to be positive.
What is the most likely cut of point of this test?
A. Induration diameter > 5 mm
B. Induration diameter < 5 mm
C. Induration diameter 5-10 mm
D. Induration diameter ≥ 10 mm
E. Induration diameter > 15 mm

152. Bronchi is composed of several type of cells. Which of the following cell type of the
bronchial epithelium that give contribution in the mechanism of coughing?
A. Ciliated cells
B. Mast Cells
C. Brush Cells
D. Kulchitsky cells
E. Mucous cells

153. A sputum sample was brought to the laboratory for analysis. Gram stain revealed rare
epithelial cells, 8 to 10 polymorphonuclear leukocytes per high-power field, and pleomorphic
Gram-negative rods.
As the laboratory consultant, which of the following interpretations should you make?
A. The appearance of the sputum is suggestive of Haemophilus pneumonia.
B. The patient has Pneumococcal pneumonia.
C. The patient has Vincent’s disease.
D. The sputum specimen is too contaminated by saliva to be useful.
E. There is no evidence of an inflammatory response.

154. Which part closes during swallowing?


A. Number 2
B. Number 4
C. Number 6
D. On either side of number 7
E. Posterior to number 11

155. A 44-year-old woman presented with increased shortness of breath, cough, and
sputum production. She had asthma since childhood and used her medications as directed. On
the examination, she was in moderate respiratory distress, respiration rate was 26x/min.
There were bilateral wheezes, and oxygen saturation was 90% on room air. On her blood gas,
the pCO2 was 50 mm Hg.
In order to relieve the symptoms above, what is the drug chosen so that normal physiologic
condition in this patient airway can be returned?
A. Beta-1-agonist
B. Beta-1-antagonist
C. Beta-2-agonist
D. Beta-2-antagonist
E. Alpha-1-antagonist

156. A 4-years old boy brought to the outpatient clinic by his mother because for the past
hour his nose had been bleeding. She said that she had tried to stop the bleeding by pinching
his nose. Her son had constant fever for 3 days, and it doesn't subside with fever medicine she
gave him. There were red spots in his arms and legs since yesterday.
If the boy complained his nose was hurting when his mother did the maneuver, which nerve
carries his pain impulses?
A. The olfactory nerve
B. The labial nerve
The understanding of nasal innervation can be
C. The ophthalmic nerve simplified by dividing it into the internal (mucosal)
and external (skin) aspects of the nose. [1] The
D. The facial nerve external nose is innervated by the ophthalmic (V1)
and maxillary (V2) nerves, which are the first 2
divisions of the trigeminal nerve (cranial nerve V).
E. The superior palatine nerve The superior aspect of the nose, including the tip,
is supplied by the infratrochlear nerve (V1), the
supratrochlear nerve (V1), and the external nasal
branch of the anterior ethmoid nerve (V1).

The infraorbital nerve (V2) supplies the inferior and


lateral aspects of the nose, extending to the lower
eyelids

157. Asthma is a chronic inflammatory disease with characteristic tracheobronchial tree


hyper- responsiveness to multiple stimuli which resulting in episodes of reversible airway
narrowing. Some physiological changes occurred in this disease are related to nervous and local
control of the organ affected.
Select the most likely statement which is true about nervous and local control of the bronchiolar
musculature.
A. Direct control of the bronchioles by sympathetic nerve fibre is relatively strong
B. Many of sympathetic fibres penetrate into the central portions of the lung
C. Bronchial tree is very much exposed to norephinephrine and epinephrine released into
the blood by adrenal gland
D. Sympathetic stimulation cause cdoilantiosntriction of the bronchial tree
E. Norepinephrine has greater stimulation of beta adrenergic receptors than epinephrine

158. Select the most likely affinity of haemoglobin for oxygen and the Hb-O2 dissociation
curve as blood passes through systemic capillaries :
A. Hb affinity for O2 increased and the dissociation curve shifts to the left
B. Hb affinity for O2 increased and the dissociation curve shifts to the right
C. Hb affinity for O2 decreased and the dissociation curve shifts to the left
D. Hb affinity for O2 decreased and the dissociation curve shifts to the right
E. Neither Hb affinity for O2 nor the Hb-O2 dissociation curve change

159. A patient without ketonaemia has increased respiratory rate. Which is the most
likely factor responsible for this increase?
A. Blood pH
B. Blood O2
C. Blood CO2
D. Blood 2,3-BPG
E. Blood glucose

160. A 50-year-old man with history of tobacco use who complained of several months of
cough and lower back pain and an 11.3-kg weight loss. He was treated for pneumonia after a
chest radiograph showed total opacification of the right lung. Computed tomography imaging
subsequently revealed a superior right hilar mass and mediastinal lymphadenopathy. Mediastinal
biopsy showed poorly differentiated epithelioid tumour with desmoplastic stromal reaction,
neutrophil infiltration, and squamous differentiation. Tissue immunostaining confirmed a non-
small-cell lung cancer.
Which nuclear medicine examination is the most suitable for the case management?
A. Bone scan
B. V/Q study
C. Tc-99m Sestamibi Whole Body Scan
D. Tc-99m Ethambutol scan
E. F-18 FDG PET Scan

161. A 63-year-old man has the FEV1/ FVC ratio less than 70% and FEV1 30 % of the
predicted value after bronchodilator treatment.
Select the most likely physical findings that is not affected by the bronchodilator
A. Pleural friction rub treatment.
B. Bronchial breath sound
C. Bronchophony
D. Narrowing of intercostal space
E. Hwyidpeneinrgs? onor in percussion

162. Hospital acquired pneumonia is a type of nosocomial infection. Select the


statement which best defines hospital acquired pneumonia.
A. Pneumonia which developed minimally 24 hours after admission.
B. Pneumonia which developed minimally 36 hours after admission.
C. Pneumonia which developed minimally 48 hours after admission.
D. Pneumonia which developed minimally 96 hours after admission.
E. Pneumonia which developed minimally 120 hours after admission.

164. A 67 year old man underwent surgery removing the lump in his voice box that had
made him slowly losing his ability to speak. The doctors inserted small tube in his neck to help
him breathe by making a small hole in the anterior part of his neck, under the laryngeal
prominence. How did the tube insertion possible?
A. Because the anterior part of the neck only consist of muscles
B. Because the c-shaped cartilage in the trachea is deficient in the posterior part
C. Because the c-shaped tracheal cartilage is only located at the superior most and
inferior most of trachea
D. Because the cartilage in the trachea can be pierced by the plastic tube
E. Because the c-shaped cartilage of cartilage is stacked up vertically and has gaps
between them.
insertion between 3rd and 4rd cartilage
165. Which of the following concerning average lung volumes and capacities of a person
at rest is true?
A. TLC > VC > TV > FRC
B. TLC > FRC > VC > IC
C. TLC > VC > FRC > TV
D. TLC > IC > VC > RV
E. TLC > FRC > TV > VC

166. This wall of the nasal cavity are irregular owing to three bony the nasal conchae,
plates, which project inferiorly, somewhat like louvers
What is the wall that is described in the above sentence?
A. Medial wall
B. Lateral wall
C. The roof
D. The floor
E. Nasal septum

168. A 62-year-old man was being managed in the intensive care unit following a large
anterior wall myocardial infarction. He had been appropriately managed with oxygen,
aspirin, nitrates, and β-adrenergic receptor blockers but developed recurrent episodes of
ventricular tachycardia. During these episodes he remained conscious but feeling dizzy. He
was also diaphoretic and hypotensive.
Amiodarone is commonly used to prevent further recurrence of ventricular tachycardia. If it is
given with warfarin, select the most likely reason why the dose of warfarin should be adjusted:
A. Amiodarone inhibits cytochrome P450 enzymes
B. Amiodarone induces excretion of warfarin
C. Amiodarone inhibits absobtion of warfarin
D. Amiodarone replaces protein binding site of warfarin
E. Amiodarone inhibits interaction of warfarion with antithrombin III

169. A 30-year-old female presented to the Emergency Departement with complaints of


high fever, cough and dyspnoea since 3 days ago. Her mother was on medication for
tuberculosis. On examination, chest percusion is dull, with ronchi heard on auscultation. The
laboratory examination revealed leucocytosis.
Chest radiographic examination showed opacification in the right lower lung zone , air
bronchogram +, meniscus sign + with no enlargement of the heart.
Select the best chest radiograpic examination position for pleural effusion.
A. Postero-anterior position
B. Right lateral oblique position
C. Right lateral position
D. Right lateral decubitus position
E. Antero-posterior position

170. Neonatal Respiratory Distress Syndrome (RDS) caused by insufficient secretory product
of certain cell type in the:
A. Bronchiole
B. Terminal bronchiole
C. Alveolar duct
D. Alveolus
E. Capillary

171. A 7-year-old girl was brought by her parents to the outpatient clinic complained of
having fever for 2 days. She also complained of pain during swallowing. Her parents said that
this was the second time in 4 months she had the same complains. On physical examination,
her temperature was 38.4ºC, pulse rate 100 bpm. Her other vital signs was within normal limit.
When the doctor performed oral and pharyngeal examination, he found two enlarged bumps
on either side on of her throat. The enlargements were occupying half of her fauces, they were
red. Afterward, the doctor asked her parents for consent to do posterior rhinoscopy using a
mirror that introduced into the back of her throat.
What is the doctor trying to confirm by doing the procedure?
A. Enlargement of lingual tonsil
B. Blockage of the choanae
C. Enlargement of the palatine tonsil
D. Blockage of the pharyngotympanic tube
E. Enlargement of adenoid

172. A 64-year-old man came to a clinic for follow-up of his chronic congestive heart failure.
He has a marked reduction in his ejection fraction following a series of myocardial infarctions.
His symptoms include dyspnea on exertion, paroxysmal nocturnal dyspnea, orthopnea, and
peripheral edema. He has normal renal function. He is on treatment with twice daily of ACE
inhibitor and once daily of furosemide. You decide to add digoxin to his regimen.
Which is the most appropriate action of ACE inhibitor in this patient?
A. It improves left ventricular ejection fraction
B. It increases peripheral resistance
C. It reduces ventricular preload
D. It increases cardiac contractility
E. It reduces heart rate
173. Inhalation of fungal spores can cause primary lung infections. Select the organism
which is most likely associated with this mode of transmission.
A. C. Albicans
B. C. Immitis
Endemic fungal pathogens (eg, Histoplasma capsulatum,
C. C. Tropicalis Coccidioides immitis,

D. S. Schenckii Blastomyces dermatitidis, Paracoccidioides brasiliensis,


Sporothrix schenckii, Cryptococcus neoformans) .
E. common route of infection is the introduction of spores to the body through a cut or puncture wound in the skin.
T. Tonsurans

174. A 44-year-old woman noticed a tightness or ache in her


chest after playing tennis. The tightness disappeared after a few
minutes rest. Her LDL cholesterol was 300 mg/dL.
She was diagnosed with angina and prescribed isosorbidedinitrate
(ISDN) tablets, which were to be placed under the tongue, not
swallowed. In addition, she was given aspirin and simvastatin.
What is the most appropriate action of aspirin in this patient?
A. It enhances the analgesic effect of ISDN
B. It minimizes the risk of blood clotting
C. It lyses the clot
D. At high dose, it inhibits platelet activation and aggregation
E. It vasodilates the coronary artery

175. There are some risk factors in the development of COPD,


but the most important risk factor is
A. tobacco smoking.
B. genetic factors.
C. alpha 1 antitrypsin deficiency.
D. socioeconomic status.
E. airway hyperresponsiveness.

Final respi 2009 9 - Januari 2012

1. For questions number 1 to 2, refer to scenario below:

A 17 year old man was biught to the emergency department due to sports injury. He was guarding the
goalpost when suddenly the ball was kicked right in his face’s direction and he couldn’t avoid it. His
nose was bleeding profusely, and bent. After the doctor performed maneuver to his nose, the shape
returned to normal.

Which of the following structure most likely cause the difference in his nose shape?

a. The septal cartilage


b. The alar cartilages
c. The cribiform plate
d. The vomer bone
e. The nasal bone

2. Which of the following artery most likely cause the bleeding?


a. Ophthalmic artery
b. Internal carotid artery
c. Anterior ethmoidal artery

d. Sphenoethmoidal artery
e. Facial artery

3. What structure anterior tampon filled to stop bleeding?


a. Middle meatus
b. Common meatus
c. Inferior meatus
d. Posterior choanae
e. Sphenoethmoid recess

4. Wanita 45 th datang dengan keluhan sneezing dan runny nose sejak pagi. Struktur manakah
yang bertanggungjawab untuk sensory innervasi pada pasien ini?

a. Olfactory nerve
b. Anterior dan posterior ethmoidal nerve
c. Vestibulocochlear nerve
d. Infratochlear nerve
e. Sphenoethmoidal nerve

5. 37 old brought to ER because of rapid and breathing difficulty. His family brought him to the
hospital after he fell from 5m pole in independent day. X-ray : collapsed right lung

From the following which sentence is the most appropriate ?

a. Right lung has 3 lobes and 10 segment


b. Right lung only have oblique fissure
c. Right lung have cardiac notch

d. Right lung only have horizontal fissure


e. Right lung only have 2 lobes and 10 segment

6. A middle aged man found in the road form traffic accident. His right clavicle was fractured. Which part
of the lung should be concerned?

a. Lower lobe of left lung


b. Upper lobe of left lung

c. Upper lobe of right lung


d. Middle lobe of right lung
e. Lower lobe of the right lung

7. On cadaver dissection, you found intersection muscle in the anterior part of thorax. The muscle
attached from lower end of one ribs to upper part of ribs below it. Muscle apakah itu?

a. Innermost intercostals muscle


b. Internal intercostals muscle
c. Anterior intercostals muscle
d. External intercostals muscle (inspirasi)
e. Serratus posterior
8. A 49 yo man complained of having constant hiccups for 2 day. He tried everything to stop them, but it
wouldn’t stop. What a innervations of the responsible structure for the case above?

a. Vagus nerve

b. Left and right recurrent laryngeal nerve


c. Intercostals nerve
d. Splanchnic nerve
e. Phrenic nerve

9. Bagian roof dari nasopharynx?


a. Body of sphenoid

b. Base of frontal
c. Mastoid part of temporal
d. Vomer
e. Perpendicular plate of ethmoid

10. A 42 years old woman came to emergency because of chocking after swallowing a big ball of rice. She
was eating fried fish when she felt a prickly sensation in her throat. Where would fish bone most
likely stuck?

a. The pharyngoepiglotic fold


b. The interarytenoid notch
c. The retropharyngeal space
d. The laryngeal inlet
e. The piriform recess
Ans : E. the piriform recess

11. Regarding the skeleton of larynx, WOTF statement is most suitable?

A. Thyroid cartillage lies superior to cricoid cartilage. Thyroid cartilage has laryngeal prominence
at lateral right below its greater horn.

B. Cricoid cartilage connected with epiglotis cartilage through thyrohyoid membrane


C. Epiglotic cartilage forms the laryngeal inlet, lies superoposteriorly from thyroid cartilage
D. Paired arytenoid cartilago lies lateral and inferior to the thyroid cartilage

E. Hyoid bone lies superior to all the laryngeal cartilage right at upper part of cricoid cartilage

12. Structure below form laryngeal inlet:


A. Cricoid cartilage

B. Triticeal cartilage
C. Thyroid cartilage
D. Corniculate cartilage
E. Hyoid bone

13. A 26 y.o man came to the clinic with chief complaint hoarse voice. This man is a vocalist of a rock band
that had performance in two consecutive days. In what mechanism which sound likely produced?

A. a wide-opened rima glottidis passed through when expiration


B. Apposition of vocal cord when expiration

C. Apposition of vocal cord when inspiration


D. Extension of vestibular folds during expiration
E. The widening of laryngeal vestibule during inspiration

14. . A 25 yo woman complaining of losing her voice after she had surgery that removed her thyroid
gland. What stucture is must likely affected in the surgery and cause the above symptom

A. Recurent laryngeal nerve


B. Vagus nerve
C. Inferior vagal ganglion

D. Superior laryngeal nerve


E. Glossopharyngeal nerve

15. Wotf sentence regarding the windpip (trachea)?


A. Extends from the base of pharyx into the posterior mediastinum

B. Has ring cartilage to keep its lumen open.


C. Innervated by the branch of CN xi
D. Started at level C6 to Tg IV disk
E. Lies posterior to esophagus

16. 7 years old come to the outpatient clinic due to earache and decrease hearing since 2 days before. His
parents says he had prior to earcharce, greenish nasal discharge for 2 weeks and was unable to
relieve symptoms by over the counter medicines. First change ...

A. long limb of incus


B. body of stapes

D. lenticular process of incus


E. handle of malleus

17. 30 y.o female had complaints of uncomfortable feeling in the ear and decrease hearing after her
plane landed.

What structure is responsible to relieve the complaint above?

A. The tensor veli palatini muscle that pulls the opening of pharyngotympanic tube in middle ear
B. The levator veli palatini muscle that pulls the opening of pharyngotympanic tube in middle ear
C. The tensor veli palatini muscle that pulls the opening of pharyngotympanic tube in nasopharynx

D. The levator veli palatini muscle that pulls the opening of pharyngotympanic tube in nasopharynx

E. The tensor tympanic muscle that pulls the opening of pharyngotympanic tube in middle ear

18. 24 years old medical student diagnosed having sinusitis and ask his physician why he experienced
a nasal drainage during the night but not the day. What's your explaination?

A. Disruption drainage due to mastication


B. Location ostia within nasal passage
C. Diurnal mucus production increases at night

D. Location ostia within eustachian tube


E. Sinus most likely affected is maxillary sinus

19. 22 yo woman, possibly sinusitis. Terdapat purulent discharge di middle nasal meatus. Sinus likely to be
affected?

A. posterior sphenoid sinus


B. lacrimal duct

C. frontal duct
D. sphenoid sinus
E. posterior ethmoid sinus

20. 42 y.o woman had accident trauma to orbital structure (extraocular muscle)maybe push inferior into
sinus:

A. Frontal
B. Maxilla
C. Sphenoid
D. Ethmoid
E. Sphenoethmoid

21. 18 y.o boy having sinusitis. Translucent (+). Sinus mana yang bisa diperiksa dengan teknik
transillumination?

A. Maxillary & ethmoidal


B. Anterior ethmoidal & frontal
C. Frontal & sphenoidal
D. Maxillary & frontal
E. Sphenoidal & maxillary
22. Post-tonsillectomy. Lost of taste sensation at posterior one-third of tongue. Inervasinya?
A. CN IX

B. Pterygopalatine nerve
C. Great-palatine nerve
D. CN V
E. CN X

23. During a procedure to remove a palatine tonsil, the operating field was suddenly filled with
bright red blood.

Which artery was inadvertently damaged?


A. Tonsillar branch of facial artery

B. Ascending pharyngeal artery


C. Ascending palatine artery

D. Descending palatine artery


E. Superior laryngeal artery

24. Cowo, menghirup kacang. Dilakukan bronchoscopy, objek ada di?


A. Left lower lobar bronchus
B. Left main bronchus
C. Left superior segmental bronchus

D. Right lober lobar bronchus


E. Right superior segmental bronchus

25. 55 y.o man smoker has an acute onset of shortness of breath and severe chest pain with breathing.
Physical exam shows COPD with barrel chest, decreased breathing sound, hyperresonance on
percussion at left lung. What is the most likely diagnosis?
A. AMI

B. Angina pectoris

C. Pneumothorax
D. Empysema
E. Pleural effusion

26. Pkoknya tadi ada pasien ibu-ibu 65 tahun, di lobectomy terus hoarseness, yang kena nerve
apa. Pilihannya lupa =p

27. Thoracosentesis, you decide to insert the aspiration needle over the top of the rib into n intercostal
space inferior to the lower border of the lung in the midaxillary line at the end of normal expiration.
What of the following is the higest level at which procedure might safely done?
th
A. 4 ICS
th
B. 5 ICS
C. 6th ICS
th
D. 7 ICS
th
E. 8 ICS

28. Orang 26 tahuun melakukan exercise stress test. Apa yang terjadi di skeletal musclenya?
A. Decrease capillary hidrostatic pressure

B. Decrease metabolite concentration


C. Increase arteriolar diameter
D. Increase oxygen concentration
E. Vascular resistance

29. Nausea, Vomit gastroentritis. 3 hari kemudian light-blindness ketika duduk atau berdiri. Hasil BGA nya
? metabolik alkalosis
30.
In the diagram shown, point X indicates the acid base status of healthy person. Which of the following is the most
likely cause of the condition indicated by point Y?

a. Adaptation to high altitude


b. Chronic obstructive pulmonary disease (COPD)
c. Diarrhea

d. Ingestion of strong acid


e. Severe prolonged vomiting

31. The unlabeled solid curve in the graph shown represents the loading behavior of normal
human hemoglobin at a pH of 7.35 as a function of oxygen concentration.

Which of the following labeled curves best represents the most likely change after addition of
carbon dioxide? B

32. Laki-laki 60 tahun, datang dengan keluhan shortness of breath. Riwayat hipertensi selama 25 tahun,
diobati dengan diuretic.

RR 19x/menit, HCO3 turun. Abnormalitas yang terjadi:

A. Respiratory alkalosis
B. Respiratory acidosis

C. Metabolic acidosis
D. Metabolic alkalosis
E. No abnormalities

33. After release O2 to tissue, Hb will transport …


A. CO2 + proton into lung

B. O2 into lung
C. CO2 + proton into tissue
D. O2 into tissue
E. O2 + proton into lung
34. Kurva disosiasi
oksihemoglobin, kalo increase 2,3 biphosphoglycerate kurvanya bergeser ke mana?

a. Displacement of the oxyhemoglobin dissociation curve to the left

b. No change in oxyhemoglobin dissociation curve


c. Displacement of the oxyhemoglobin dissociation curve to the right

d. Dissociation curve is not valid for 2,3 biphospoglycerate


e. Increase affinity for oxygen

35. 75 yo man ada peningkatan SOB selama 2 minggu terakhir, ada 25 tahun sejarah hipertensi terkontrol
dengan diuretic, 2 months ago serum urea nitrogen and creatinin were within references ranges. PR 98,
RR 19, BP 180/100. Lung dull percussion at base and crackles 1/3 way up bilaterally. JVP increase, S3
gallop, no murmur, and +3 pitting edema lower limb. Lab : Na 126, K 5,4, Cl 108, HCO3 16, urea
nitrogen 75, creatinin 3, Hb 12. Correct statement main fisiological buffer in case above?

A. Hb
B. Creatinin
C. Urea bitrogen
D. Bicarbonate
E. Chloride

36. Large quantities of carboxyhemoglobin hinder oxygen delivery substance related?

A. CO2
B. CO
-
C. HCO3

D. HCN
-
E. Cl

37. Buffering agent di asam basa tubuh adalah extracellular bicarbonate. Kalo kekurangan zat itu apa
yang terjadi?

A. Metabolic acidosis
B. Respiratory alkalosis
C. Metabolic alkalosis
D. Respiratory acidosis

38. The abnormality of blood pH is dangerous because mostly they will change the function of which
biomolecules

A. Lipid
B. Protein
C. Amino acid
D. Nucleic acid
E. Carbohydrate

39. Can severe asthma attack change blood pH ?


a. No, because asthma attack would not influence the function as a pH regulator organ

b. No, because the decrease of O2 blood level will decrease the oxidation processes so less CO2 are
produced
c. No, because in asthma the alveoli are still intact so gas exchanges are not disturbed

d. Yes, because the decrease of O2 blood level will lead to anaerobic metabolism causing higher
level of blood lactic acid

e. Yes, because less CO2 could be excreted by the lung causing higher level of blood carbonic acid

40. Long time hyperventilation akibat panic attack menyebabkan perubahan pH apa?
A. Respiratory acidosis
B. Respiratory alkalosis
C. Metabolic acidosis
D. Metabolic alkalosis

E. Tidak berubah

41. The ability of lung to act as pH regulator organ is because it can:

a. Produce CO2
b. Excrete CO2
c. Absorp O2
-
d. Produce H
-
e. Excrete H

42. 2 y.o boy didiagnosis pneumonia. Kultur: + Streptococcus pneumoniae. Alergi terhadap Penicillin dan
Cephalosporin. Antibiotik yang tepat?

a. Azithromycin; the most effective antibiotic against streptococci that can penetrate into most tissue

b. Amoxicillin + clavulanate; it is a broad spectrum antibiotic that can prevents activity of bacterial
betalactamase

c. Clarithromycin; it is effective against gram positive organism and has different immunologic
characteristic with beta-lactam antibiotic

d. Streptomycin; it is effective against both gram + dan gram – can be given by parenteral
e. Metronidazole; it is effective against both gram + and –

43. 45 y.o rhinosinusitis, alergi penicillin, renal insufficiency. Agennya resisten gentamycin tapi susceptible
terhadap penicillin dan macrolide. Resepnya eritromycin dan pseudoephedrine untuk 3 hari

44. kalau dokter ngasih erythromycin, info apa yang dikasih ke pasien?
A. Drug should be taken with food
B. Kemungkinan progresi renal dysfunction
C. Kemungkinan alergi karena riwayat alergi penisilin
D. Kemungkinan liver toxicity
E. Resiko toksisitas karena renal insufficiency

45. If the doctor gave him (penicillin allergic patient) clarythomycin, which issued should be considered
when the doctor prescribe it?

A. The ototoxic of clarythomycin


B. Toxicity potency related to his renal failure
C. Possibility of allergic reaction related to history of penicillin allergic
D. Potency of treatment failure related to sensitivity test result
E. Other drug used that may enhanced secretion by renal
Soal untuk 46 dan 47

a 10 year old boy; 25 kg came with runny nose with whitish discharge, cough, and nasal blockage since 3 days
before. The doctor gave him pseudoephedrine, gliserilgualakolatin combination with dextramethorphan
46. What is the most appropriate reason that the combinationof dextromethorphan as cough reflex
suppression is not rationale?

a. Gliserilgualakolat reduce the viscosity of mucus

b. Gliserilgualakolat increase the transportability of mucus


c. Gliserilgualakolat stimulate surfactant secretion
d. Gliserilgualakolat increase the tenacity of secretions by increasing adhesivity
e. Gliserilgualakolat increase airway mucus secretion by hidrating

47. Role of pseudoephedrine in rhinitis?


A. Menurunkan volume mucus lewat reseptor alfa-1

B. Mengubah permeabilitas vaskuler lewat histamine


C. Menurunkan produksi mucus lewat parasimpatetik
D. Intervensi produksi mucus lewat reseptor alfa-2
E. Lewat simpatetik membuat bronkokonstriksi

48. Seorang ibu yang baru melahirkan bayinya. Ibunya diobati OAT tapi menghentukan pengobatan karena
hamil. Appropriate planning for mother?

A. Lanjutkan pengobatan, stop menyusui

B. Lanjutkan menyusui dan beri pengobatan baru (kategori 2)


C. Stop menyusui dan beri pengobatan baru
D. Lanjutkan menyusui dan beri 5HRE
E. Stop menyusui, beri 5HRE

49. Woman, 25 yo, diagnosed lung TB few months before her pregnancy. She had 1 month of continuation
phase therapy using R & H before she stopped her treatment by herself, because afraid it may harm her
fetus. 1 week before she delivered a helath baby, BP, T, RR and HR increased. Lymphadenopathy of deep
cervical, ronchi on apex chest. What most appropriate planning for mother?

A. Start new regiment: 2HRZES/ HRZE/ 5 (HRE)3


B. Continue her last regiment until full 6 months
C. Give rest continual phase, then change it with category II regiment
D. Start new regiment: 2 HRZES/ HRZE/ 5(HRE)3

E. Start with 5 (HRE)3

50. Ibu hamil, dulunya suspect TB, berhenti pengobatannya karena takut mengganggu kondisi bayi. Jika
pasien mengalami renal insufficiency dan tidak mengubah dosis regimen, symptom apa yang bisa terjadi?

A. Isoniazid jadi nephropathy


B. Streptomycin jadi hyperuricemia
C. Rifampin jadi hepatotoxic
D. Ethambutol jadi retrobulbar neuritis
E. Akumulasi pyrazinamide jadi peripheral neuropathy

51. 53 yo extra pulmo TB, taking kategori II regimen anti TB with B6, normal liver, but starting visual
disturbance. WOTF of the appropiate statement related to his visual disturbances?

A. Pyrazinamide and consult to opthalmologist


B. Pyrazinamide with suplement B6
C. Check AST/ALT
D. Reduce streptomycin dose
E. Stop ethambutol

52. Which is the most correct prescription for this patient?


53. What is the role of vitamin B6 in this patient?
A. Prevent the gouty attack
B. Prevent the peripheral neuropathy
C. Prevent the drug induces neuropathy
D. Reduce abdominal upset because of INH dan R
E. Protect eyes from toxicity

54. 45 yo admnistrated ART, recently diagnosed TB.


A. 2HRZE/4HR, STOP ART
B. 2HRZE/4H3R3, STOP ART
C. 2HRZES/HRZE/4H3R3, CONTINUE ART
D. 2HRZE/4HR, CONTINUE ART
E. 2HRZE/4H3R3, CONTINUE ART

For questions number 55 to 58, refer to scenario and options below:

A 64 year old man came to the clinic complained prolong cough, night sweat, body weakness, loss of apetite and
mild fever since three month ago. After taking history, physical examination and preliminary investigation, he
was diagnosed with pulmonary and cervical lymph node TB. He also had history of after default therapy of
tuberculosis. He got fivedrug regiment consisting INH, rifampisin, pyrazinamide, acid ethambutol, and
streptomycin.

a. INH
b. Rifampisin
c. Pyrazinamide
d. Ethambutol
e. Streptomycin

55. inhaled Obat TB yang pemakaiannya dibarengi dengan pemberian pyrodoxine? A. Isoniazide
56. Obat TB apa yang menghambat sintesis yang mengikat subunit RNA polimerase? B. Rimfampisin
57. The drug inhibit he meabolic of phenitoin? A. Isoniazide

58. OAT with poor bioavailability? E. Streptomycin

Untuk no 59-63 :
A 10 year old girl with history childhood asthma complained cough, dyspnea, and wheezing. Her sympyoms
became severe that her parent brought her to the emergency room. Physical examination revealed
diaphoresis, dyspnea, tachycardia, and tachypnea. Her respiratory rate was 40x/min, pulse rate was 112
beat per minute, and blood pressure was 130/60 mmHg.

a. Inhaled albuterol

b. Inhaled cromolyn
c. Oral or IV metilprednisone

d. Inhaled ipratomium
e. IV propanolol

59. What is the most effectively drug to reverse bronchoconstriction? A. Inhaled albuterol (SABA)

60. The drug most likely to provide sustained resolution of patient’s syndrome? C.
Oral/IV metilprednisone
61. Yang kontraindikasi buat pasien diatas? E. IV propanolol

62. Yang menyebabkan side effect muscle tremor, tachycardia dan cough? A. albuterol

63. Factor that should be consider to calculate the actual energy need for COPD patient ...
a. Stress level add to total energy required
b. BEE
c. Normal physical activity
d. Lupa
e. Balance protein requirement

64. Decreased food intake is common in each person with COPD but actually energy needs may be
increase or decreased. It is crucial to remember that energy balance.

A. Normal stress level add to total energy need


B. BEE
C. Normal physical activity
D. Give more simple carbohydrates composition
E. Balance Ratio Protein

65. The situation appears related to pulmonary complications such as the degree of airflow obstruction,
thus increasing the energy needs resulting from the increased work of breathing, gas difussing capacity,
CO2 retention, respiratory inflammation and biochemical mediators such as hormones and cytokines affect
energy expenditure.

The incorrect condition about metabolic disorders in pulmonary disease is:

a. Hypercatabolic

b. Hyperglycemia
c. Increase lipolysis & ketogenesis
d. Depletion of vitamin & mineral
e. Decreasing of free radicals

66. Lifestyle chnge with healthy food intake decrease risk factor exerbation COPD

These healtjy food supplement or antioxidant can improve health and decrease risk factor exacerbation
of COPD:

a. Vitamin A and ferum


b. Potassium and Biotin
c. Calcium and Vitamin E

d. Natrium and Riboflavin


e. Vitamin C and magnesium

67. Protein requirement in patient with COPD depend on degree of hypercatablic. Sufficient protein
is necessary lung and muscle strength as well as to promote immune function is …

a. <0.8 g/kg BW
b. 0.8-1.2 g/kg BW
c. 1.3-1.5 g/kg BW
d. 1.5-2 g/kg BW

e. >2 g/kg BW

68. These are dilated cavities in the frontal, maxillary, ethmoid and sphenoid bones around the nose and
eyes. Mucus produced here drains into the nasal fossa through small openings protected by the conchae.
Which of the following epithelial tissue is most likely lined the mucous layer of these dilated cavities?

a. Stratified squamous non keratinized


b. Stratified squamous keratinized

c. Pseudostratified columnar
d. Stratified columnar

e. Stratified cuboidal

69. Upper part dari pharynx adalah broad single cavity overlying the soft palate it is continuously
anteriorly with nasal fossae and inferiorly with oral part pharynx. The wall supported by bone
and skeletal muscle. Epithelial tisuue?

a. Stratified squamous non keratinized


b. Strat squamous keratinized
c. Respiratory epithelium
d. Strat columnar epithelium

e. Strat cuboidal

70. This cavity in temporal bone and has connection with auditory meatus. It is closed by thympanic
membrane and communicate with nasopharyng through Eustachian tube. Epithelial tissue?
(pilihan jawaban sama dengan no 69) simple cuuboid

a. Stratified squamous non keratinized


b. Strat squamous keratinized
c. Respiratory epithelium
d. Strat columnar epithelium

e. Strat cuboidal

71. This serous membrane has 2 layers, 1 covering the lung and the other covering internal wall of the
thoracic cavity. The narrow cavity lies between the 2 layers of serous membrane contain only a firm of
lubricating fluid. Wotf epithelial tissue is most likely lined the inner layer of these sac?

a. simple squamous
b. simple cuboidal

c. simple columnar

d. pseudostratified columnar
e. stratified columnar

72. These small air sacs form the primary structural and functional unit of the respiratory system, because
their wall permit exchange of CO2 for O2 between the air in their lumina and blood in adjacent capillaries.
Which of the following epithelial tissue is the most likely lined the inner layer of these sacs?

a. Simple squamous

b. Simple cuboidal
c. Simple columnar
d. Pseudostratified columnar
e. Transitional

73. Polio paralysis intercostals muscle. Wotf is the appropriate value?


a. Increase IRV

b. increase ERV
c. Total lung capacity not affected
d. decrease IRV
e. vital capacity las affected

74. A 45 year present with fever and cough. She has had no past medical problems and was well until about
o
3 days ago. Physical examination is remarkable for a temperature of 39 C and the presence of diffuse rales
on chest examination. Except for an elevated white count with a left-shifted differentia, her blood has resting
hypoxemia and requires hospital admission.

Which of the following is the most appropriate cough mechanism in this patient?

a. 2,5 liter air expired


b. Increase abdominal pressure
c. Increase the pressure of the alveoli

d. Afferent nerve impulse through the Vagus nerve


e. Automatic sequence triggered by fever

75. 22 m.o boy chief complaint: shortness of breath, inspection: chest show retraction of the chest wall.
Wotf event during expiration?

a. lung expands
b. diaphragm stop contraction

c. initiates by neural impulse


d. intrapleural pressure more subatmosphere

e. increaseof transpulmanary pressure

76. Mr.P, an 18 years old motorcyclist involved in a road traffic accident. He complained of pain over his

right upper chest, and breathlessness, which increased progressively. Doctor in charge at E.R has diagnoses
that patient has pneumothorax.

Which of the following statement is a correct about pleural pressure?

a. It always eventually equalizes itself with the atmpspheric pressure


b. It is negative relative to the alveolar pressure
c. It is a measure of the elastic forces in the lungs

d. It is also called the recoil pressure


e. It is the difference between the alveolar and intrapleural pressure

77. 70 y.o man, cc: shortness of breath & yellowish sputum history DM +, PE: RR 30, T 38 celcius, Bp

130/80, ronchi +/-, wheezing -/-, blood glucose 276 mg/dl, chest x-ray: infiltrate +, no history of
hospitalization. wotf describes most accurately about pecculiarities found in case above?

a. alveolar macrophage effectively combated of infection process


b. the smooth muscle on the wall constrict many smaller airways
c. the obstruction will cause air entrapment in the alveoli

d. bronchial obstruction will decrease airway resistance


e. the pathological process increases the lung difusing capacity

78. Once you were on duty in emergency room a 5 years old girl patient was taken by her parents with
chief complaint stridor since 2 days ago. Her parents also complained about sore throat, fever, difficulty on
swallowing and breathing. There was no cough and she was unable to control her own saliva and began to
drool

Result from the physical examination are:

The child sat upright in a bent-forward position. The jaw was open and drooling was frequently present. She
presented a hot potato voice, suprasternal retraction and inspiratory stridor.

Radiologic findings:

Form soft tissue radiograph : “thumb sign” appreance. Chest X-ray : within normal limit.

Related to the case above, which of the following describes most accurately about functions of the
respiratory passageways?

a. All the passageways have cartilage to keep them form collapsing


b. All of the passageways are surrounded by smooth muscles
c. In obstructive disease all of the passageways are constricted
d. The greatest amount of resistance to airflow occurs in alveoli
e. In disease conditions, the smaller bronchioles determine resistance

79. O2 saturation correlates with PO2 and O2 delivery to tissue. Factor influencing?
intinya ini yang kurva antara saturasi dan PO2. Curve is shift to the right if:

a. pH turun
b. 2,3 BPG naik
c. temperatur naik

d. CO2 naik

80. The lung function test in this patient revealed a decrease in MEV. Wotf will be the most
probably found in his lung function test?

a. FEV1 normal
b. FVC nomal
c. FEV1/FVC increase
d. FEV1 increase
e. FEV1 decrease
81. A 39 years old male underwent tracheostomy after sever head injury. For this, air thorugh the tube does
not anymore pass through the nose, so that it is not favorable in term of normal respiration. Because of
that, which of the following describes most accurately about very impotant functions of the nose.

a. Its warming effect is endowed by filtration process in the nose


b. It serves as air conditioning function of upper respiratory tract
c. Its filtration process involves turbulent precipitation by the hairs

d. The entrapped particles will be transported to the trachea


e. The nose can still filtrate the particles smaller than 1 micrometer

82. WOTF can cause increase gas exchange ?


a. Polycythemia vera
b. Physical exercise
c. hypertention

d. Emphysema
e. Increase surface area of respiratory membrane
83. Which is TRUE about respiratory regulation :

a. Dorsal respiratory center causes expiration


b. Ventral respiratory center causes inspiration (ventral = ekspirasi)

c. Dorsal respiratory center causes inspiration


d. Increase pCO2 detected by peripheral chemoreceptor
e. Increase pO2 detected by central chemoreceptor

84. What is the most powerful stimulus for breathing in healthy person ?
a. Increase O2
b. Increase CO2

c. Increase pH
d. Decrease pH

85. Hasil BGA dari seorang pasien pneumothorax usia 25 tahun :

HCO3 = 32,1(tinggi)

BE +2,8

SaO2 = 96,4%

pO2 = 97,4

pCO2 = 71,4 (tinggi)

ventilation process disturbed. WOTF breathing work is mostly possibly involved ?

a. Compliance work is increased


b. Tissue resistance work is increased
c. Airway resistance work is increased

d. Tissue resistance work is decreased


e. Airway resistance work is decreased

86. A 22 month old boy brought to the emergency room with chief complaint shortness of breath,

inspection of the chest showed retraction of the chest wall.

Which of the following muscles pulls the rib cage downward during expiration?

a. External intercostals
b. Abdominal recti

c. Sternocleidomastoideus
d. Anterior serrate
e. Sacleni

87. 70 y.o man, cc: shortness of breath & yellowish sputum history DM +, PE: RR 30, T 38 celcius, Bp

130/80, ronchi +/-, wheezing -/-, blood glucose 276 mg/dl, chest x-ray: infiltrate +, no history
of hospitalization. wotf describes most accurately about pecculiarities found in case above?

a. Alveoli are filtered with due to increased hydrostatic pressure


b. Total surface area of respiratory membrane decreased
c. Ventilation and perfusion increased
d. CO2 pada darah turun
e. Physiologic dead space decreased
88. What disease caused by absent or insufficient surfactant in baby?
a. Respiratory distress syndrome
b. Elastin membrane disease
c. Pneumothorax
d. Congenital lung cysts
e. Lung hypoplasia

89. A male newborn infant was choking and coughing suddenly after his first feeding. These was an
excessice amount of mucus secretion and saliva in the infant’s mouth and the infant experienced
considerable difficulty in breathing. The pediatrician was unable to pass a catheter through the esophagus
into the stomach. Which of the following is the embryological basis that causes that this anomaly?

a. Failure of recanalization of the larynx


b. Incomplete of recanalization of the larynx
c. Narrowing and obstruction the trachea

d. Abnormal partitioning of the trachea and esophagus by the tracheoesophageal septum


e. Larynx and upper trachea fail to separate completely from the esophagus during the first trimester
90. The sphlancnic mesoderm in respiration system will develop which component ?

a. Bone
b. Epithelial tissue

c. Striated smooth muscle


d. Gland
e. Connective tissue

91. A newborn infant was choking and coughing after 1st breastfeeding. There was excessive mucus
secretion and saliva in infant mouth and infant considerably difficult to breath. Pediatricians is
unable to pass catheter through the esophagus. Yg terjadi?
a. Laryngeal atresia
b. Laryngotracheoesophageal atresia
c. Atresia esophagus & trachea esophageal fistula
d. Laryngeal web
e. Tracheal atresia

92. Bayi,RDS.ttg surfactan?


a. Meningkatkan surface tension

b. Mulai diproduksi minggu ke-18


c. Diproduksi alveolar cells tipe 1
d. Minggu 24-26 sufficient amount
e. Sufficient amount: saccular stage

93. Which of the following statements is TRUE about secondary pulmonary tuberculosis lesion?

a. It is Gohn comnplex, consists of parenchymal subpleural lesion and enlarge caseous lymph nodes
b. Cavitary fibrous tuberculosis
c. Milliary tuberculosis
d. Tuberculosis bronchopneumonia
e. Pulmonary tuberculosis represent reactivation of an old (subclinical infection)

94. Newborn, chief complaint choking & coughing suddenly after breastfeeding, mucus secretion dan
saliva banyak ditemukan di mulut, dan dia susah nafas. Catheter gak bisa dimasukkin lewat
esophagus menuju lambung. Penyebab anomalinya apa?

a. Failure of recanalization of larynx


b. Incomplete recanalization of larynx

c. Narrowing n obstruction in trachea


d. Abnormal partition of trachea n esophagus by tracheoesophageal septum
e. Larynx n upper trachea failed to saperate completely from esophagus during first semester

95. Coughing & chocking. Dokter tidak bisa memasukkan Nasogastric tube ke gastric. Kelainan

embriologi ini sering disertai oleh kelainan berupa :

a. Achondroplasia
b. Amelia
c. Anencephali

d. Cranial defect
e. Renal failure

96. Which of the following is the most likely rehabilitation of medicine for COPD patients in acute stage?

a. Recondition
b. Upper arm exercise
c. Chest expantion exercise
d. Control breathing
e. Postural drainage

97. Approximately one week after major surgey, a patient experience some right-sided chest pain
and cough up blood. What is the mmost likely diagnosis?

a. Pulmonary embolism.
b. Atelectasis.

c. Adult respiratory disteress syndrome.


d. Right subdiaphragmatic abcess.

e. Pneumonia
98. Pengendara motor kecelakaan, RR 18x/min, BP sistolik 80 mmhg, vein neck distention,
xray menunjukkan tdk ada aliran udara ke right pulmo. Management?

a. Full primary survey


b. Right intercostal chesttube
c. Right needle thoracocentesis
d. Sedation with mida..(lupa nama obatnya)

e. Urgent craniotomy

99. 25 yo cewek accident, gcs 8, bp 90/60, rr 30, upper airway clear, jvp not distended, trache deviate to
right, breath sound absent over left chest, percussion at left chest dull. right chest normal. Diagnosis:
massive hemothorax

100. 12 y.o girl. Moderate persistent asthma, appropriate drugs


a. Terbutalline syrup
b. Inhaled glucocorticoid
c. Inhaled beta-2 agonist
d. Citrilline oral

e. Oral Theophylline

101. Girl 9 yo asthma since 6 year old. Last year her asthma exacerbate once a week, night symptoms less
than once aa week. Between exacerbation she has normal lung function. Medication:

a. Rapid acting inhaled b2 agonist


b. low dose inhaled corticosteroid
c. medium dose inhaled

d. high dose inhaled

e. medium dose inhaled + oral glucocorticoid

102. Barking cough, stridor, awakened at night, fever :


a. viral laryngitis
b. spasmodic croup
c. epiglotitis

d. viral tb

103. A 5 YO male child presented to the ER department with a history of breathlessness since 5 hours. This
symptoms associated with cold and cough which has been occuring for a week and particularly present
at night and morning times. He had been treated by GP with antibiotics and cold and cold remedies
without any improvement. During the last one year the symptoms appeared recurrently. On PE she
appeared alert, dyspneic, afebrile, chest intercostal and subcostal retraction was noted. Prolong
expiration was found. No expiratory wheeze nor crackles were heard. No abnormality was found on
other examination. Which of the following diagnosis is most likely to this patient?

a. Moderate acute asthma attack


b. mild acute asthma attack
c. Persistent asthma
d. Frequent episodic asthma

e. Acute asthma attack

104. Brutono, 3 tahun, sudah mengidap asma sejak umur 8 bulan, tahun lalu asma ny kambuh 2x sebulan,
Night symptoms lbh dr 1 kali seminggu, menggangu tidur dan aktifitas.

obat controller apa yg diberikan?

a. LABA
b. Low dose ICS

c. Low to medium ICS + LABA


d. High dose ICS + LABA

e. High dose ICS + LABA + phenyltaline

105. The initial approach is considered to any children of all ages without symptoms illness who are
known to have had a recent TB infection?

a. INH 5mg/kg once daily until 3 months

b. 5mg 6months
c. 10mg 3months
d. 10mg 6months
e. 15mg 3 months

106. Soal ttg deep neck infection, di mana kena submandibular, submental, sublingual.
Diagnosisnya? ANS : Ludwig angina

107. What's the initial step in mananging this condition?


a. Tracheostomy
b. Broad spectrum antibiotic
c. Incision and drainage
d. Surgery
e. Palliative

108. 30 yo woman, yellowish nasal secretion from both nostril since 5 month. Nasal blockage, secretion
comes down to oropharyngeal area. Rhinoscopy : secretion comes from ostium maxillary sinus, mucosal
edema, post nasal drip. Diagnostic tools to know every aspect of disease?
a. Posterior rhinoscopy and CT scan
b. Transillumination and sinus paranasal radiology projection
c. CT scan and transillumination

d. Nasal endoscopy and CT scan

e. Water's and Coldwell position radiologic


109. –

110. Pria 70 thn melakukan pemeriksaan spirometri. Hasilnya : FEV1/FVC =52%,


FEV1 = 35% predicted value. Diagnosisnya?

a. Copd stage 0
b. Copd stage I

c. Copd stage II
d. Copd stage III

e. Copd stage IV

111. Seorang pria, setelah kecelakaan, mengalami nyeri, worsening breathing, deviasi trachea, penurunan
vesicular breath sound. Diagnosis?

ANS : Tension pneumothorax

112. SABRINA MUNGGARANI


113. Result of air trapping in asthma :
a. airway obstruction during inhalation
b. increased lung compliance caused by inflammation

c. collapse bronchioles during exhalation


d. shunt ventilation upper lung field
e. increased mismatch ventilation

114. Pasien BTA (+) udah di treatment 1 bulan, pilihan regiment apa?
115. 18 y.o man complaints : cough with sputum for 5 weeks, low grade fever, night sweats. FAB (+), lesion

at upper lung. Past history : 2 years ago undergo TB treatment for 6 month, feel better and cured.
Category for treatment?

a. new case

b. relapse
c. after default
d. treatment failure
e. transfer in
116. ARVIANA PUTRI
117. Wanita 27 tahun tiba-tiba pleuritic pain in left chest, sulit bernapas
a. mycoplasma pneumonia

b. spontaneous pneumothorax
c. pulmonary embolism

d. acute pericarditis
e. pleurodynia
118. A 66 year old man complains of breathlessness for 3 weeks. On examination, he has an area of

dullness to percussion at the left base with markedly decrease tactile fremitus. Breath sound on the l
eft area are distant at the base, but there is an area of bronchial breathing in the mid posterior chest.
What are these physical signs most compatible with?

a. pericardial effusion
b. consolidation of the left lower lobe
c. consolidation of the left lingual

d. left sided pleural effusion


e. atelectasis of the lower lobe

119. SILMINA KUSMAEHADI

120. 65 y.o man come with cough, SOB, whitish sputum for 5 years. 15 years smoking. X-ray:
hyperinflation, deflate diaphragm, no infiltrate. FEV1/FVC : 45% FEV1: 60%. Sign will not found?
a. Crackles
b. prolong expiration
c. wheezing
d. –

e. pursed lip breathing

121. Laki laki 65 tahun datang dengan acute chest pain, diaphoresis dan near syncope, T: 37, 6 C.
distended JVP, pulmonic second sound dan edema ekstremitas. Penyebab hipoksia ? Impaired arterial
diffusion of O2….. shunting …..

122. Tanda-tanda gejala mirip Pneuonia, Namun tidak ada infiltrasi dan no opacity pada X-ray.
Penyakit apa ?

123. Acis

124. 20 tahun wanita dengan keluhan SOB, palpitasi, ketakutan merasa akan mati tanpa ada sensasi lain.
Terjadi 3-4x/minggu. Diagnosisnya ?

A. MI
B. COPD
C. Asthma
D. Panic attack
E. Agrophobia

125. A 40 year old woman presents with complaints of being able to leave her house since one month ago,
she constantly worries that she will not able to get help if she has attack. In fact, she has had numerous
unprovoked episodes of intense fer, associated with shortness of breath, chest pain, diaphoresis and
dizziness that lasted 5 minutes. Which of the following is the mosy likely diagnosis?

A. generalized anxiety disorder


B. social phobia

c. Specific phobia

d. Panic disorder with agoraphobia

e. panic disoreder without agoraphobia

For questions number 126 to 128, refer to scenario below:

A 27 year old woman came to our clinic with 10 days history of nasal discharge. The discharge is yellow-greenish,
thick, blood tinged also accompanied by decreased of the smelling sensation, nasal blockage, fever, and
headache. Three days ago she felt pain and fullness on the left ear with mild hearing lost.

126. she suffered the disease because, she had the predisposing factor is:
a. Allergy

b. virus infection

c. congenital defect
d. cold waaether
e. immunonompremise

127. in this case there is decreased of smelling sensitive cause of ?


A. infacted olfactory
B. bleedy nose
C. increase intranasal pressure

D. increase nasal discharge


E. increase intrasinus pressure

128. At this case sinus x ray we found :


a. Mass image
b. darkness
c. clear
d. cloudiness

e. no abnormality

For questions number 129 to 131, refer to scenario below:


A 15 month old boy brought by his mother with a chief complaint of respiratory distress since the last 6 hours.
Four days ago he had a common cold. His mother also complains of an audible hoarseness, inspiratory stridor,
and barking cough.

129. The etiologi of the disease is:


a. bacterial infection
b. viral infection
c. fungi infection
d. parasite infection
e, pseudoviral infection

130. The inspection of the chest we found?


A. Suprasternal retraction
B. decreased of the respiratory rate
C. Stridor

D. Barel chest
E. Bulging

131. In this case, the microorganism attacks:


A. Laryngeal vestibule
B. Laryngeal muscle
C. Laryngeal nerve
D. Blood vessels
E. Laryngeal cartilage

For questions number 132 to 133, refer to scenario below:

A 30 y.o. Man came to outpatient clinic RSHS, he came with complaint: chronic nasal congestion.
Rhinoscopy shows: gelatinous grayish white tissue with smooth and shiny surface, fill in the right and
left nasal cavity. The biopsy was performed and the microscopic appearance showed the picture below

132. What is the most appropiateTerminology for the nasal mass in this patient?
A. Chronic Rhinitis

B. Nasal polyp
C. Inverted papilloma
D. Angiofibrosis
E. Nasal carcinoma

133. According to pathogenesis, which inflammatory cell are seen microscopically on above picture(nasal)?
A. Limfosit
B. Plasma cell
C. Neutrofil
D. Basofil
E. Eosinofil

For questions number 134 to 135, refer to scenario below:

A 23 year old male came to the hospital because of epitaxis since 2 days ago, he also complained nasal
blocked since 1 month ago. Rhinoscopy and nasopharyngoscopy revelaed a reddish ulcerative mass in
the choane and nasopharynx posterior. Biopsy of the nasal and nasopharynx was performed, macroscopic
and microscopic appearance of the mass was shown as picture below:
134. what is the most appropriate terminology describing the disease in the nasal and nasopharynx of above
patient?

A. Polyp
B. Angiofibroma
C. Papiloma
D. Carcinoma
E. Chronic inflammation

135. Cause pathogenesis?


A. Pollen
B. House mites
C. Ebv
D. HPV
E. HIV

For questions number 136-137, refer to scenario below:

A 45 y.o male come to out patient clinic because of dyspnea, fever, and productive cough since 7 days ago.
He said thar sputum was yellow green sometimes with blood streak. PE showed fever, takipnea, tachycardia
and crackles in his left chest. Macroscopic and microscopic appearance of his lung were shown as picture
below:
136. what is the disease that occur in above patient?

A. pneumonia b.
pulmonary tb c.
emphysema d.
lung Ca

e. COPD

137. What structure are pointed by the white arrows in above case?
a. extravasated erythrocytes

b. alveoli destruction c.
datia langhans cells d.
caseous necrosis e.
tumor cells

138. Mrs. S is a 24 year old college student. She has a history of Crohns disease and is complaining of a
four daya history of bloody-watery diarrhea. A blood gas is obtained to assess her acid/base balance.
What is your interpretation?

a. partially compensated metabolic acidosis due to excessive bicarbonate loss from her diarrhea
b. uncompensated metabolic acidosis due to excessive bicarbonate loss from her diarrhea c.
uncompensated respiratory acidosis due to bicarbonate retention from her diarrhea

d. partially uncompensated respiratory acidosis due to bicarbonate retention from her diarrhea
e. uncompensated metabolic acidosis due to bicarbonate retention from her diarrhea

139. Mrs. L is a thin, elderly looking 61 y.o COPD patient she has an arterial blood gas done as part of her
routine care in pulmonary clinic. The results are as follows:

What is your interpretation for BGA in this patient ?

a. Fully compensated respiratory acidosis with hypoxemia


b. Fully compensated metabolic acidosis without hypoxemia
c. Fully compensated metabolic acidosis with hypoxemia
d. Partially compensated respiratory acidosis with hypoxemia

e. Partially compensated metabolic acidosis with hypoxemia

140. Ms. St is a17 y.o with intractable vomiting and dehydration. She has some electrolyte abnormalities, so
a BGA is obtained to assess her acid/base balance

What is your interpretation and treatment ?

a. Uncompensated respiratory alkalosis. Treatment consist of fluids, anti emetic and management of her
electrolyte disorder

b. Partially compensated metabolic alkalosis. Treatment consist of anti emetics and management of her
electrolyte disorder
c. Partially uncompensated respiratory alkalosis. Treatment consist of bicarbonate, anti emetics
and electrolyte disorders

d. Uncompensated metabolic alkalosis. Treatment consist of bicarbonate and anti emetics

141. Mr. F is a 60 y.o with pneumonia He is admitted with dypnea, fever, and chills. His blood gas is below:

What is your interpretation and what are treatment goals for Mr. F ?

a. Uncompensated metabolic acidosis with hypoxemia, improve both ventilation and oxygenation

b. Partially metabolicacidosis with hypoxemia, improve oxygenation only


c. Uncompensated respiratory acidosis without hypoxemia, improve ventilation only
d. Partially compensated respiratory acidosis with hypoxemia, improve both ventilation and oxygenation
e. Uncompensated respiratory acidosis with hypoxemia, improve both ventilation and oxygenation

142. mrs J is a patient with chronic COPD being admitted for surgery, her admission labworkreveals in arterial
blood gas with the following values:
What is your interpretation ?

a. Fully compensated respiratory acidosis

b. Fully compensated respiratory alkalosis


c. Fully compensated metabolic acidosis
d. Partially compensated metabolic alkalosis
e. Partially compensated respiratory acidosis

143. Yang required most agressive ventilation:


A. 33 tahun woman, breathing 18 breaths/mnt, pulse 76, wheezing and shortness of breath
B. 7 tahun cewek breathing 16 breaths/mnt, pulse 76, sleepy
C. 6 bulan, breathing 16 breaths/mnt, pulse 76, lethargic
D. 65 tahun COPD, breathing 18 breaths/mnt, pulse 76, wheezing, has just ran out oxygen tank

144. A 5-year-old child is brought into the emergency department with drooling, strident cough, and
lethargy. Epiglotitis is suspected. The priority intervention for this child is:

A. Take vital signs


B. Secure child's airway
C. Take arterial blood for BGA
D. Obtain throat culture

145. Anak rumahnya kebakaran, dalam 24 jam gejala apa yang mesti kita waspadai:
A. Infection
B. -aduh lupa-
C. PTSD
D. Pneumothorax

146. Yang paling tepat dikasih PPV (positive pressure ventilation)?


A. 18 years old, shortness of breath, rash on chest

B. 66 years old, shortness of breath, swollen ankle


C. 80 years old, diminished lung sound, smoker >20 years, RR= 24 bpm
D. 6 years old, wheezing bilateral, constantly cries

147. Anak 2 tahun kekurangan oksigen karena S. pneumoniae dan membutuhkan endotracheal tube. Indikasi
manajemen ini berhasil?

A. bilateral breath sound terdengar

B. SaO2 80%
C. ujung endotracheal tube mencapai mainstem bronkus kiri
D. ujung edotracheal tube mencapai mainstem bronkus kanan

148. Prolonged nausea, vomitting, diare. BGA?


A. Metabolic acidosis

B. Metabolic alkalosis
C. Respiratory acidosis
D. Respiratory alkalosis
149. Standard precautions include following:
A. covers body fluid containing visible blood
B. specified for patient known specific pathogen
C. prevent transmission droplet nuclei
D. hand washing as personal hygiene
E. hand-to-hand contact prevention

150. As standard precautions do not cover patients infected with H5N1 influenza virus. It should be
completed by

A. droplet precautions
B. hand hygiene campaign

C. personal protective equipment


D. antimicrobial soap

E. environmental control

151. Anoxic condition which is important in forensic medicine is : (151)


A. Anemic hypoxic
B. Metabolite hystotoxic hypoxia
C. Stagnant hypoxia
D. Hypoxic hypoxia

E. Extra hystotoxic hypoxia

152. Riani Wulandari


153. Thomas Adhi Nugroho Chaidir
154. Endi Pramudya Laksana

155. 20 y.o male complaint persistent nasal discharge, discharge was yellowish and thick. Past history nasal
itchy and nasal obstruction when cold weather. Water film shows cloudy at maxillary sinus. What is the
most likely diagnosis?

A. Tumor of right maxillary sinus


B. Right hematosinus
C. Chronic sinusitis
D. Acute sinusitis
E. Mucocele

156. On posteroanterior chest film, there is a cavity in the apical part of upper lobe, the wall of cavity is thick
and surrounded by patchy consolidation. (156)

A. Lung abscess
B. Pulmonary cyst
C. Mycectoma in cavity
D. Cavitating tumor

E. The cavity at pulmo TB

157. The segmental homogenous lung opacification with air bronchogram is (157)
A. Atelectasis

B. Pleural effusion
C. Pneumonia
D. Schwarte
E. Bronchopneumonia

158. Cerita untuk 157 dan 158. Dyspneu, cough, high fever. Anteroposterior X-ray homogenous lung
opacification with air bronchogram in lateral segment. Pertanyaan: this disease begin as localized
infection of: (158)
A. Terminal air space

B. Terminal bronchioles
C. Respiratory bronchioles
D. Acinus
E. Interstitial

159. A girl, usia 2 tahun menderita dyspneu, fever, dan cough sejak 7 hari lalu. Di X-ray ada homogenous lung
opacities dengan air bronchogram di lateral middle lung. Lateral lung itu di sebelah mana? (159)

A. Upper left lobe


B. Upper right lobe

C. Middle of right lung


D. Lower lobe of left lung
E. Lower lobe of right lung

160. What is the basic standard radiograph for any patient presenting with cough >3 weeks? (160)
A. Posteroanterior chest film
B. Oblique chest film
C. Left lateral decubitus
D. Apical lordotic

E. Anteroposterior chest

161. Azhar – lupa nyatet


162. factor that can interfere BGA in heavy smoker
A. false high pco2 because high pco2 value
B. false low ph level bcoz of low ph level
C. false low o2 bcoz of low o2 value
D. false high o2 saturation
E. false high base excess

163. a 2 months old girl suffered from cough for more than 2 weeks. The patient mother become
concerned with the child turned blue after series of coughing spells that ended with vomiting. She
bought her to pediatrician. The patient had not yet any vaccination . the doctor diagnosis pertussis.
What is characteristic property of etiology of this disease?

A. grow on Mac concay agar


B. gram – bacteria

C. obligate anaerob bacteria

D. non fastidious bacteria


E. specimen culture is sputum

164. Yuni
165. the principal reservoir for antigenic shift variant to be:
A. people in isolated communities such as jungle

B. animal : pigs, horses fowl


C. soil : tropics
D. sewage
E. humid environment

166. 54 yo fever, chill and productive cough of minimal sputum. He stay at local hotel, several more person
same complain possible cause

A. S. pneumonia
B. Cryptococcus
C. RSV
D. legionella pneumonia

E. P. Jirovecii

167. wanita 61 tagun yang merupakan seorang tuna wisma ditemukan tergeletak dijalanan pada saat
hujan. Suhunya 40 C. productive cough. Malnutrisi. Dokter menduga ia TBC. Maka dilakukan
pemeriksaan sputum. Prosedur yang sebenarnya adalah…

A. KOH methylene blue staining


B. gram staining
C. ZN staining
D. nessier staining

E. scafhefer fultok… staining

168. lab menggunakan mac conkey dihasilkan mucoid gram -, lactose fermentation, indol -, apa itu?
A. mycoplasma pneumonia
B. klebsoella pneumonia
C. streptococcus pneumonia
D. legionella pneumophilla
E. Chlamydia pneumonia

169. ada pasien fever, diketahui etiologinya gram +, alfa hemolytic, sensitive optochin?
B. S. pneumonia

Respiratory System Perfecten

f. A 70 yo nursing home patient refused the influenza vaccine and subsequently developed
influenza. She died of acute pneumonia 1 week after contracting the “flu”. The bacteria
that cause of acute post influenza pneumonia is gram – rods and doesnt grow on blood
agar nor Mac Conkey agar. What is the possibly cause of this disease?

Legionella (BCYEAgar), negatif

Listeria X

S. aureus

Klebsiella negatif, bisa Mc Concey
Eschericia coli X

11. An infant, seen in ER, presents with fever and persistent cough. PE and CXR showed
Pneumonia. Which of the following is most likely the cause of this infection?

Rotavirus

Adenovirus

Coxsackievirus

RSV (respiratory synchitial virus)

Rhinovirus

14. Coronaviruses are recognized by club-shaped surface projections that are 20 nm long and
resemble solar coronas. These viruses be a major agent of the common cold, especially in
older children and adults are characterized by :

a. The virion is known to contain 8 fragment of RNA

b. The virion is positive sense single stranded RNA ? susah di kultur , irregular, envelop di luar
ada 3 glycoprotein S (spike),E (envelop),M (membran), HE . respiratory infection SARS,
common cold

c. Grow well in the usual cultured cell


lines d. Non enveloped virus
e. Agglutinate human red blood cells ?

15. Which virus is the leading cause of croup syndrome, bronchiolitis, and pneumonia in
young children that contain RNA in nucleocapsid encased within an envelope, and, when
infecting mammalian cells in culture, will hemabsorbreb blood cells?

A. Group B coxsackievirus
B.Rotavirus
C.Parainfluenza virus
D.adenovirus E.rhinovirus

16. Rhinovirus is a major cause of the common cold that primarily transmitted by contact of
contaminated hands, fingers, or fomites with the conjunctiva or nasal epithelium. Which is

the characteristic of this virus?


A. Member of paramyxoviridae parainfluenza, RSV B.

Member of Orthomyxoviridae influenza

C. Has a positive sense single stranded RNA corona, picorna (echovirus, cosaxkie, rhinovirus)


D. Has a negative sense single stranded RNA A,B, Bunya viridae E.
It is an enveloped virus

6.Excluding influenza virus, which one of the following viruses is a common cause of acute

respiratory disease?

A.cytomegalovirus

B.rotavirus

C.varicella-zoster virus

D.adenovirus d ouble stranded DNA

E.papillomavirus

2 This virus is a single-stranded RNA orthomyxovirus. In the developed countries vaccination


is necessary because of antigenic drift and shift.

A. Measles virus
B. Influenza
virus C. RSV

D. Parainfluenza virus
E. Adenovirus

3 A 62 yo male who had influenza a week earlier reported to the emergency room with a

38,5oC fever and shaking chills, a productive cough with a yellowish sputum, and chest pain.
CBC revealed 30.000/mm3 WBC, predominantly PMN. The result of sputum culture revealed
H.influenza. The virulence of H.influenza, the most likely target would be : polisakarida kapsul/
poliribitol fosfat

A. Exotoxin liberator
B. Endotoxin assembly
C. Flagella synthesis D.
Capsule formation

E. IgA protease synthesis

Virulence, at least in the case of bacteremia and meningitis, is directly related to capsule formation. Virtually all of these infections are caused

by the type b serotype, and its capsular polysaccharide, containing ribose, ribitol and phosphate, is the proven determinant of virulence. The
capsule material is antiphagocytic, and it is ineffective in inducing the alternative complement pathway, so that the bacterium can invade the

blood or cerebrospinal fluid without attracting phagocytes or provoking an inflammatory response and complement-mediated bacteriolysis.
9. What media that should be used to inoculate sputum specimen from that patient above

For this reason, anticapsular antibody, which promotes both phagocytosis and lysis of bacteria, is the main factor in immune defense against

H. influenzae infections.
to grow H. Influenza?
The polyribosyl ribitol phosphate (PRP) capsule is the most important virulence factor because it renders type b H. influenzae resistant to

A. Blood agar phagocytosis by polymorphonuclear leukocytes in the absence of specific anticapsular antibody, and it reduces the bacterum's susceptibility

to the bactericidal effect of serum. However, susceptibility to the bactericidal effect of serum depends on the presence of antibodies to a

B. Lowenstein jensen media number of other antigenic sites, including the lipooligosaccharide and outer membrane proteins designated as P1 and P2

tb
C. Mac Conkey agar
All virulent strains produce neuraminidase and an IgA protease, but the role of these extracellular enzymes in invasion is un proven.

gram - and enteric bacili

D. Chocolate agar

E. Loeffler media
diphteri bacili

16. A 21 yo college student complained of malaise, low-grade fever, and harsh cough, but not
of muscle aches and pains. An xray revealed a diffuse interstitial pneumonia in the left lobes
of the lung. The WBC count was normal. The student has been ill for a week. The bacteria have
a characteristic that doesn’t have cell wall. Based on the information given, what is the
etiologic agent that causes the disease?

Orang hebat adalah orang yang Melakukan Hal biasa secara luar biasa
A. Mycoplasma pneumonia paling kecil paling simple, butuh kolesterol, patogen mucus
membran

B. Chlamydia pneumonia

C. Staph aureus

D. Legionella pneumophila

E. Streptococcus pneumonia

2 A 25 yo man known to have AIDS developed a gradual onset of malaise and anorexia. An
acid fast bacilli from sputum is identified microscopically. M. tuberculosis can be found in the
sputum of patients with tuberculosis. After digestion of the sputum, isolation is best
accomplished using

A. Sheep blood agar


B. Loffler's media C.
Thayer-martin

D. Thiosulfate citrate bile salts sucrose medium

E. Löwensteinjensen medium, ogawa, middle brook media

3 What is theimportant characteristic of cell wall structure of acid fast bacilli?

A. Has a high content of lipopolysaccharide

B. Outer membrane contain lipotechoic acid

C. Has no peptidoglycan, so that, bacteria resistent to antibiotic

D. The presence of mycolic acid and lipoarabinomanan

E. Has polysaccaride capsule as attribute of pathogenicity

17. The structure of M. tuberculosis that potentiate the toxicity and promote intracellular
survival by inhibit the phagosome-lysosome fusion in the macrophage is

A. Mycolic acid
B. Wax D

C. Sulfatide, struktur patogen yg lain : cord factor trehalose mycolate (inhibit neutrofil
migration&granuloma formation)
D. Lipopolysacaride
E. Lipoteichoic acid

18. A 25 yo male with previous rhinnorhea, pharyngitis cough, visit the doctor with a 39* fever
which appears abruptly after a sudden shaking chill episode. He has a chest pain and
productive cough. X-ray shows siffuse lobar consolidation of the right lung. Sample from
sputum was cultured, and the bacteria isolated was Streptococcus pneumonia.

What is the positive test to identify this bacterium from colony on blood
agar? A. Beta hemolysis on blood agar (pyogene)

B. Sensitive to optochin

C. Require of X & V factor (H.influenza)

D. Resistent to novobiocin (S.saprophyticus)


E. Catalase positive (staphylococcus)

19. Which of the following statement about respiratory tract mycosis is correct?

A. A group of superficial mycosis deep

B. Only caused by opportunistic fungi patogenic

C. The true pathogenic fungi is is dimorphic fungi yeast, hifa
D. Cannot be diagnosed by laboratory examination

E. Also called dermatophytosis

2 A 37 yo man presence to the office for the evaluation of cough and fever. He has had these
symptoms for about a week. His pulmonary examination is notable for some taint expiratory
wheezing and crackles in the left upper lung field. A culture from respiratory

specimen grows on saburoud agar represent a rough walled of round macroconidia and


microconidia. Conidia Aseksual fungal spore
A. Histoplasmacapsulatum


B. Coccidioidesimitis


C. Aspergilusfumigatus punya hifa in rare condition D.

Cryptococcus neformans - pseudohifa


E. Paracoccidioidesimitis

3 A 65 yo patient with diabetes and difficulty in breathing was asked by pulmoologist to go


to microbiology laboratory for microbiological examination of sputum. The sputum specimen
was thick and bloody. The colony yield heavy growth of a lactose positive, non motile, gram
negative rodwith large capsule.m

Which of the following bacteria with those characteristic is to be the cause of the pulmonary

problem


A. Enterobacterpneumoniae LRT, UTI, endocariditis, CNS
B. Klebsiellapneumoniae

C. Mycoplasma pneumoniae


D. Clamydophiapneumoniae kecil kayak virus, small obligate E.
Legionella pneumophila -> gram (- )rod

4 An 11 yo girl came home from school because she had a high fever and complained of
difficulty of swallowing any food. Her parents noted that several children from her school had
reported sore throats recently. Throat swab was taken for culture, the bacteria was gram
positive cocci in chain, yields clear, sharp beta hemolysis on blood agar, sensitive to
bacitracin.
Of the organisms list below, which one is the bacteria that cause the
disease? A. Staphylococcus aureus

B. Staphylococcus saprophyticus
C. Streptococcus pyogenes

D. Streptococcus pneumoniae
E. Streptococcus viridans

5 Gas exchange between pulmonary airways and pulmonary blood happen during ?

A. Inhalation, because the alveoli retract during exhalation

B. Inhalation, because at this time oxygen level of the airways is still high

C. Inhalation, because at this time the alveolar surface area is greatly expanding

Orang hebat adalah orang yang Melakukan Hal biasa secara luar biasa
D. Inhalation, because at this time the alveolar air pressure reacheas its peak

E. Exhalation and inhalation, because at this time the alveoli do not appreaciably change in
size

24. What is the effect of 2,3 DPG to the affinity between oxygen and
hemoglobin? A. Increase it, because oxygen is needed to synthesize 2,3 DPG

B. 2,3 DPG has no effect on the affinity between oxygen and hemoglobin C.
Decrease it, because 2,3 DPG replace O2 at the hemoglobin molecule

D. Decrease it, because 2,3 DPG stimulates the changing of hemoglobin shape

Purified hemoglobin binds much more tightly to the

E. Decrease it, because 2,3 DPG stabilize hemoglobin in the T (deoxyhemoglobin)oxygen,formmaking it less useful in oxygen transport. The
difference in characteristics is due to the presence of

2,3-Bisphosphoglycerate(2,3-BPG) in human blood, which

acts as an allosteric effector. An allosteric effector binds in


21. Whi ch of the fa ctors can increase the a ffi nity between O 2 and hemoglobi n?

one site and affects binding in another. 2,3-BPG binds to a

pocket in the T-state of hemoglobin and is released as it

forms the R-state. The presence of 2,3-BPG means that

A. increase of CO2 tension more oxygen must be bound to the hemoglobin before the

transition to the R-form is possible.

B. increase of body temperature

F. increase of hydrogen ion level

G. increase of erythrocyte glycolysis


H. increase of O2 atmosphere pressure, DPG, pH, pCO2

30. If the PO2 of blood that enter the lung is 40 mmHg and PO2 of the alveoli air is 104
mmHg, then PO2 of blood that leave the lung must be:

A. more than 104 mmHg because O2 move from the higher pressure to the lower
pressure

B. 104 mmHg, because the blood uptake of O2 is aiffusion process

C. between 40 to 104 mmHg, because not all blood vessel of the lung is used during
breathing

D. Between 40 to 104 mmHg, because blood from pulmonary veins is diluted by blood
from bronchial veins

E. Between 40 to 104 mmHg, because the blood passes the lung only in a short time so
there is not enough time to saturate it completely

Choose one of the most appropriate answers from these options below :

c. Respiratory acidosis ga bisa eliminate CO2 di paru

d. Metabolic acidosis ginjal ga bisa eliminate

e. Respiratory alkalosis

f. Metabolic alkalosis diabetic, hypercholeremic, lactic acid
g. Normal blood P H

2 Mostly can be happened in the presence of excess CO2 in inspired airA

3 Happened in healthy person that participate a 10 kilometers marathon B


kompensasinya yg D

25. Hyperventilation due to panic syndromeC

kebanyakan ngeluarin co2 --> kurang --> ph naik

26. Decreased alveolar ventilation in emphysema due to COPD A


retensi co2 krna sulit ekspirasi

27. Which of the following statements related about bronchioles?


a are a major site of gas exchange

b contain no smooth muscle in their walls therefore significantly reduce air way
resistance

c contribute to more than 50% of resistance to flow in their lower respiratory tract

d contain of fibroblast which secrete highly proteinaceous fluid

e differ from bronchi which contain cartilage in their wall

33. Which of the following statements related to mucosal lymphoid tissue in the respiratory
tract?

32. non capsulated lymphoid tissue encapsulated
33. mostly β lymphocytes

34. activated lymphocytes will not home bachy to respiratory mucosa

35. within the upper respiratory tract make up waldeyer’s ring

36. it is a major site of monocytes activation in the airways

34. Which of the following statements related to emphysema?


32. diffusion of O2 is decreased due to destruction of the lung parenchyma

33. over 10% of cases have ≤2 antithrypsin deficiency

34. surface for gas exchange is increased

35. reducing airway resistance

36. patient develop of febroticf lung

35. A newborn infant was observed choking and continues coughing. There was and excessive
amount of mucus secretion and saliva in the infant mouth and the infant experienced
considerable difficulty in breathing.

Which one of the congenital anomaly was occur?

32. Olygohydramnion of the lung development

33. Tracheal Stenosis

34. Tracheooesophageal Fistula

35. Oesophageal agenesis

36. Tracheal difeticulum

36. which one the embryological most common locations of this defect?

32. Superior part esophagus and trachea

33. Inferior part oesophagus near tracheal difurcation

34. In the middle part oesopgush and tracheal

35. Upper part of trachea


36. Upper part of oesophagus

37. premature infant developed rapid shallow respiration shortly after died was diagnose
respiratory distress syndrome

Which one usually cause RDS?

32. Incapable type 1 alveolar producing hyaline protein

33. Incapable type 2 alveolar producing surfactant

34. Suggested prolonged extra uterine asphixhya

35. Occur in premature infant undle

36. Incapable the lung function

Orang hebat adalah orang yang Melakukan Hal biasa secara luar biasa
2 the role of the human sinuses is very significant in daily life. Which od the following
statement is the correct embryological development of the sinuses

1 Paranasal sinuses develop during late fetal life

2 The maxillary sinuses grow rapidly and fully developed before all the permanent
teeth have erupted

3 The two most posterior ethmoidal cells grow into the frontal bone, forming a frontal
sinus on each side

4 Paranasal sinuses form from outgrowth of the walls of the nasal placode

5 The maxillary sinuses are present at birth sama ethmoid

3 the exchange of gases between alveolar capillary blood and alveolar cavity is a process

of:

1 Paracellular diffusion

2 Simple diffusion

3 Active transport

4 Facilitated diffusion

5 Antiport transport system

4 thesplanchic mesoderm in the respiratory system will develop which of the following
component

1 Bone

2 Epithelium tissue

3 Striated muscle tissue

4 Glands

5 Connective tissue

5 which of the following type of epithelium cells lining most of the respiratory portion of
the respiratory system?

1 Ciliated psedostratified columnar conducting
2 Simple columnar

3 Simple cuboidal

4 Stratified squamous

5 Simple squamous

6 within the lamina proprialia a number of cartilages. The large cartilages are hyaline, and
the smaller cartilages are elastic. Which of the following is most likely tube?
1 Larynx

2 Trachea

3 Primary bronchi

4 Secondary bronchi

5 Bronchioles

7 the first part of respiratory portion of respiratory system:

1 Bronchi

2 Bronchioles

3 Terminal bronchioles

4 Respiratory bronchioles
40. Alveolar ducts

2 thisintralobular airways have neither cartilage nor glands in their mucosa. In the larger
part the epithelium is ciliated psedostratified columnar. Which of the following is most likely
tube?

A Bronchi

B Bronchioles

C Alveolar duct

D Terminal bronchioles

E Respiratory bronchioles

3 the smaller portion of this respiratory track is lined by cuboidal epithelium and contains
Clara cells. Which of the following is most likely tube?

A Respiratory bronchioles

B Terminal bronchiole

C Alveolar duct

D Alveolar sac

E Alveoli

4 These tonsils are located in the lateral walls of the oral part of the pharynx. Which of the
following epithelial cells that covered external surface of these tonsils?

a. stratified squamous epithelium non


keratinized b. stratified columnar epithelium

c. stratified cuboidal epithelium


d. simple columnar epithelium
e. respiratory epithelium

For question number 42 and 43, refer to the scenario below :

A 17 years old man was brought to the emergency department due to sports injury. He was
guarding the goalpost when suddenly the ball was kicked right in his face’s direction and he
couldn’t avoid it. His nose was bleeding profusely,(posterior) and bent. After the doctor
performed maneuver to his nose, the shaped return to normal.

44. Which of the following structure most likely cause the difference in his nose
shape? a. the septal cartilage

b. the alar cartilage


c. the cribiform cartilage
d. the vomer cartilage e.
the nasal bone

45. Which of the following artery most likely cause the bleeding?

a. opthalamic artery

b. internal carotid artery

c. superior ethmoidal artery

d. sphenopalatine artery

e. facial artery

Orang hebat adalah orang yang Melakukan Hal biasa secara luar biasa
?44. A 2 year old girl brought to the emergency department by her parents. They said that
their daughter had been playing with beads, and one of the beads was stuck inside her nose.
They had tried to take it out, but was unsuccessful. They asked you for assistance. Where is
the most possible location of the logged bead? ?????


a. between nasal septum and the conchae common nassal
meatus b. between the middle and inferior conchae c. in the
sphenoethmoidal recess

d. passed the choanae

e. in the vestibule

46. A 20 year old man has been diagnosed with sinusitis and asks her physician why there is
nasal discharge during the night but not during the day.

Which of the following is the most suitable explanation?

A. Disruption of drainage due to mastication

B. Location of the ostia within the nasal passage

C. The sinus most likely affected is the maxillary sinus

D. Diurnal mucus production increases at night

E. Location of the ostia within the eusthacian tube

47. Which of the sentences below is TRUE regarding the sphenoid sinus?

a. it is located in the body of sphenoid, directly inferior sellaturcica

b. it is located on the greater wing of sphenoid. The sinuses separated by the sellaturcica(harsnya

bony septum) frntal sinus

c. it is drainage located in the middle meatus, in the groove of semilunar hiatus maxillary
sinus


d. it has multiple cells, collectively called the sphenoidal bullae ga punya bulae, yg punya
bulae itu ethmoidal

e. it is supplied by the sphenoidal artery ethmoidal a.

53. A 20 year old male come to you complaining of purulent nasal discharge for 2 weeks. He
stated that the discharge is foul smelling, and most particularly disrupting during bed time.
Upon transillumination examination (cahaya di taro di mulut, nanti shining di frontal dan
maxillary), you found the glow decreased inn both sides. The sentence below is TRUE
regarding the affected sinus above: frontal sama maxillary
A. The frontal sinus. (medial aspect of the eyebrow) Located in the frontal bone,
innervated by branches of CN V1

B. The maxillary sinus. It’s base forms the inferior part of the lateral wall of nasal cavity

C. The ethmoid sinus. It’s anterior cells drains into the middle nasal meatus via the
semilunar hiatus.

D. The sphenoid sinus. It’sostia located higher that the antrum, making drainage only
possible in certain position.

E. The frontal sinus. Formed during childhood. Supplied by branches from opthalamic
artery.
F. A 30 years old female had complaints of uncomfortable feeling in the ear and decrease

hearing right after her plane landed.

 
What structure is responsible to relieve the complaint above? Pull tensor, push levator,
adanya di nasopharynx

The tensor velipalatini muscle that pulls the opening of pharyngotimpanic tube in
middle ear

The levatorvelipalatini muscle that pulls the opening of pharyngotimpanic tube in


middle ear

The tensor velipalatini muscle that pulls the opening of pharyngotimpanic tube in
nasopharynx

The levatorvelipalatini muscle that pulls the opening of pharyngotimpanic tube in


nasopharynx

The tensor tympanic muscle that pulls the opening of pharyngotimpanic tube in
middle ear

G. A 15 years old boy status post tonsillectomy for recurrent tonsillitis complicated by
increased intraoperative bleeding and temporary loss of taste sensation from the posterior
one third of the tounge.

Why was there a temporary loss of taste sensation?

Compression of grater palatine nerve

Compression of pterygopalatine nerve

Compression of glossopharyngeal nerve (CN IX)

Compression of medial pterygoid (CN V3)

Compression of vagus nerve (CN X)

H. An 8 year old girl was brought to the clinic because of the decrease hearing. Her parents
said that for the last 2 weeks she’s been breathing through her mouth. This is her third time
visiting doctor for the same complain for the year.

The ENT doctor told suggested to performs adenoidectomy and tonsillectomy.

Which structure should be closely monitored in order to prevent complication? Large


external palatine vein

Ascending pharyngeal artery

Ascending palatine artery

Descending palatine artery

Superior laryngeal artery


Tonsilar branch of facial artery (tonsillectomy), katanyakalau adenoidectomy itu
yang a (di blue box moore)

f. A group of medical student was observing respiratory tract on a dissected cadaver. Upon
observing the neck, they found a structure anterior to the esophagus on the level of C3-C6
vertebrae.

Which one of the following is the characteristic of thyroid cartilage?



59. Shaped like a signet ring with it’s band facing anterior side crycoid


60. The largest cartilage. Paired on either halves of the posterior part of laryngeal inlet
thyroid

Orang hebat adalah orang yang Melakukan Hal biasa secara luar biasa
f. Located between the bases of the arytenoid cartilages and the superolateral surfaces of
the lamina of the cricoid cartilage

g. The inferior two thirds of its two plate-like laminae fuse anteriorly in the in the median
plane to form the laryngeal prominence

h. A heart-shaped cartilage cartilage covered with mucous membrane. Situated posterior to



the root of the tongue epiglotis

52. From the sentence below, which one is true regarding the larynx?

65. Infraglottic cavity lies inferior to the laryngeal inlet, superior to the vocal apparatus

66. Laryngeal vestibule is another name for laryngeal ventricle. Located between the
vestibular folds and the vocal folds

67. The movement of vocal apparatus is a result of intrinsic laryngeal muscle contractions

68. Supplied by the laryngeal artery that arises directly from external carotid artery

69. Innervated only by superior laryngeal nerve. A branch of vagus nerve (CN X)

F. A 25-year-old woman complaining of losing her voice after she had surgery that removed
her thyroid glands.

What structure is most likely affected in the surgery and cause the above symptoms?

66. The vagus nerve

67. The recurrent laryngeal nerve

The inferior vagal ganglion

The superior laryngeal nerve

The glossopharyngealnerve

f. A 44-year-old man was brought to the emergency department complaining that he


accidentally ‘ínhaled’ peanut. Upon radiological examination of the chest, the peanut was
found slightly upward on the right side of the chest.

Why foreign bodies would most likely lodged there?

a. Beacaue the caliber of the right main bronchus is larger than the trachea

Because the right main bronchus is relatively steeper while the left one has more
horizontal course

Because the right main bronchus has 3 secondary branches


Because teh left main bronchus is compressed by the passing aorta

Because the carina was very sensitive and usually initiate cough reflex

f. A 37-year-old man brought to the ER because of rapid and breathing difficulty. His
family brought him to the hospital directly after he fell from 5 m pole in the independence
day game. The doctor examined him, and found his trachea was deviated.

Which is the most appropriate sentence regarding the windpipe?

68. Extends from the base of the pharynx into the posterior mediastinum

69. Has full ring cartilages to keep its lumen open

70. Innervated by the branch of CN XI

71. Started at level C6 vertebrae to T4-T5 IV disc


69. Lies posterior to the esophagus

f. A 17-year-old male was brought to the emergency department because of severe


breathlessness. Moments before his symptoms occurs, he had fallen from the tree after
attempting to rescue his lost cat. After underwent primary survey and needle thoracostomy
through physical examination was performed. He was diagnosed with tension
pneumothorax and fractured 3rd-4th ribs.

70. Intercostal arteries is less likely to be source of bleeding compared to branches of


pulmonary artery

71. Pain is more likely caused by the tear in the visceral pleura, carried by the branches
of vagus nerve

72. The tear in the pleura caused air to enter the potential pleural cavity, making it’s
pressure fall below atmospheric pressure, thus making breathing impossible

73. Fractured ribs most likely punctured the visceral pleura that lined the inner aspect of
thoracic cavity

74. Needle thoracostomy should be done rightly above the rib to avoid the nerve and
blood supply that runs inferiorly to the ribs

2 Which of these sentences is correct?

73. The lungs has 3 lobes on the right and 2 lobes on the left. Each of the lobes divided
by the horizontal fissure

74. The lung located in the thoracic cavity. The heart carved the right lung into having
cardiac notch

75. The lungs are innervated by branches of vagusvaguskomponennerveparasimpatik(hrsnya sympatik), thus



making it highly sensitive to pain (dari visceral afferent fiber nociceptive + sympatik)
pain dari visceral afferent

f. The lung has 3 borders and 3 surfaces. It’s costal surface is concave and indented
with ribs shape, while it’s mediastinal and diaphragma surface is convex

g. The lung attach to its root, at the hilum. The structure that passes through are
pulmonary artery & veins, the main bronchi, bronchial veins, and lymphatics

75. A 65-year-old women come to you with complains of difficulty of breathing. The
complain has been going on for a month and getting worse by the night when she’s lying
down. You suspected that she has pulmonary edema.

In order to confirm your hypotheses, you need to listen to her breath sound. Where should
you place your stethoscope to hear the loudest for the breath sound anomaly?

4 On posterior chest, between the vertebrae and the scapula, at the level of T5

5 On posterior chest, at the tip border of scapula at the level of T10


ant vs post lebih baik mendengar dari post
f. On anterior chest, between the clavicle and the ridge of trapezius

g. On anterior chest at the level of 2nd ICS on the midclavicular line

h. On anterior left chest at the level of 4th ICS, right above the heart

77. You are medical intern at a public hospital. You were following your supervisor as he
make rounds at hospital wards. Your supervisor is performing physical examination on the
chest of a 60-year-old man with COPD. He was palpating the notch below the neck, in

Orang hebat adalah orang yang Melakukan Hal biasa secara luar biasa
the middle of upper thorax, and then proceed 5 cm below (sternal angle). He said he found
bony prominence and them move slightly lateral to either side. What would most likely to
be found?

c. Another bony prominence, a mark of xyphoid process

d. A bulk of mass from overused external intercostal muscle

e. A groove, widening 2nd intercostal space

f. A round prominence marked by darker skin appearance, the nipples

g. Another bony prominence, mark the costo-sternal joint

f. On cadaver dissection, you found intersecting muscle in the anterior part of the
thorax. The muscle attached from lower end of one rib, to the upper part of the ribs below
it.

What is the characteristic of external intercostal muscle?

79. Most actively involved in expiration


inspiration

b. They elevate the sternum during inspiration


rib

They run inferolaterally towards the sides of the thorax

Together with the internal intercostal occupy the intercostal space, with external
intercostal lying deeper than the internal

They occupy the intercostal spaces from the tubercles of the ribs posteriorly to the
costochondral junctions anteriorly

83. The relaxed expiration doesn’t depend on muscle work BECAUSE Main action on
expiration occurs from elastic recoils of the lung and relaxation of diaphragm.

What is the correct relationship between the two sentences above?

a. Both sentences are correct with causal relationship

b. Both sentences are correct but no causal relationship

c. The first sentence is correct while the second is false

d. The first sentence is false while the second is correct

e. Both sentence are false

84. In a 65 years old woman with difficulty breathing, the total lung capacity and functional
residual capacity (ERV+residual) and functional residual capacity (ERV+residual volume)
are lower than normal and FEV1/FVC slightly higher than normal. Whats the most probably
mechanism that happen on that condition?
resrictive lugn disease

a. Decreased pulmonary blood flow

b. Decreased strength of the chest wall muscle

c. Increased airways resistance : soalnya air trapping, jadiresidunyameningkat,


d. Increased lung elastic recoil

e. Increased chest wall elastic recoil

63. A patient has reduced total lung capacity (IRV+ERV+tidal+residual) and increased residual
volume. Functional residual capacity (ERV+residual) is normal. Whats the most probably
mechanism that happen on that condition? Inspiratory capacity (IRV+tidal) nya turun ????
ini juga resrictive --> karena fibrosis harusnya decrease lung recoil jd compliancenya turun.

a. Decreased pulmonary blood flow



b. Increased airways resistance semakin susah masuk dan keluar (lbh ke keluar)
ini obstruktif

c. Decreased strength of the chest wall muscle of respiration


d. Increased chest wall elastic recoil e. Increased lung elastic
recoil

64. A chest x-ray of patient with left-sided heart failure indicates pulmonary edema. Whats
the most probably additional examination that would reveal in this patient?


a. Decreased pulmonary artery pressure increase b.
gas exchange berkurang
Decreased pulmonary lymph flow

c. Increased pulmonary venous pressure


bisa jadi pulmonary hypertension


d. Normal arterial oxygen partial pressure turun e. Normal

vital capacity turun
kapasitas menurun krn ada fluid

Obstruktif :contohnyaasma, tapimasuknyamasihbisamasihbagus,


Cumakeluarkeluarnyasusah

Restriktif :gabisamengembang, jadigabisamasukgabisakeluar. Fibrosis gtu, myasthenia


gravis, gabisangembang.

86. A newborn infant with a deficiency of pulmonary surfactant probably will need to utilize
the muscle to have an adequate respiration. Which of the following muscle to accomplish
that condition?

a. Diaphragm, internal intercostal, scalene, sternomastoids

b. Diaphragm, internal intercostal, scalene, sternomastoids, rectus abdominis

c. Diaphragm, internal intercostal, external intercostal, sternomastoids

d. Diaphragm, external intercostal, scalene, rectus abdominis



e. Diaphragm, external intercostal, scalene, sternomastoids otot2 inspirasi, supaya
alveoli ga kolaps hrs ada yg nahan tekanan pleural fluid dari arah luar

87. In a patient with difficulty breathing, total lung capacity and functional residual capacity
are greater than normal, and forced vital capacity (FVC) and FEV1/FVC are lower than

normal. Whats the most probably mechanism that happen on that condition? obstruktif
a. Decreased lung compliance
resrictive

b. Decreased strength of the chest wall muscle

c. Increased chest wall compliance

d. Increased airways resistence


e. Decreased chest wall compliance

67. A 68 yo woman with pulmonary fibrosis, who presents with a complaint of increase
dyspnea while performing activities of daily living is referred for pulmonary function
testing. Which is the following laboratory values is consistent when her
diagnosis?Restriktifyaniiihhh, ekpansi alveolar turun, work of breathing naik, lung
volume capacity turun, ventilation dan oksigenation turun. Klo obstruksi yg rusak di

FEV1 aja, FVC nya normal. Klo restriktif FEV1 turun yg FVC lebih turun rationya bisa
lebih dari normal

a. Decreased diffusing capacity of the lung



b. Increased residual volume obstruksi, air trapping

Orang hebat adalah orang yang Melakukan Hal biasa secara luar biasa

b Decreased FEV1/FVC increase

c Increased lung compliance decrease

d Increased airways resistence corrected for lung volume obstruksi

87. A 125-lb, 40 yo woman with a history of nasal polyps and aspirin sensitivity since
childhood presents to the ED (emergency departemen) with status asthmaticus and
hypercapnic respiratory failure. She requires immediate intubation and is placed on
mechanical ventilator on a FiO2 of 40% a control rate of 15 breaths/min and tidal volume
of 500 mL. How much is her approximate alveolar ventilation? Jmlh volume yg mengalami
exchange = RRxvolume yg exchange (70% udara yg masuk)

15 x 350 = 5250

f. 375 mL/min Tidal volume - dead space = volume gas yg exchange

500-150=350 mL
g. 3500 mL/min

h. 5250 mL/min

i. 5625 mL/min

j. 7500 mL/min

90. A 68-year old male with chronic obstructive pulmonary disease entered the Emergency
Department complaining of shortness of breath. His respirations were 35 per minute
and labored. He had a productive cough and rales were heard over lung fields. The
patient had a rather ashen complexion and his nail beds gave clear evidence of

cyanosis. An arterial blood sample was obtained and chest x-ray was ordered. The patient
was then placed on an O2 mask delivering 40% O2. One half hour later, the patient found
unresponsive. His complexion had changed to flushed pink with no trace

of cyanosis. His respiraton were quiet at rate of 6 per minute and a tidal volume of 300
mL (500 mL). Repeat arterial blood gases showed that his arterial PCO2 had increased
from 55 to 70 mmHg.??????????????????
respiratory asidosis

What is the most probably mechanism that happen on that patient?

Alveolar hypoventilation

Hypoxic pulmonary vasoconstriction

Increased firing of carotid body chemoreceptors

Elimination of the hypercapnic drive

Oxygen toxicity

94. oxygen-carrying capacity of whole blood is: ada hub sama Hb (O2 yg diffuse sama yg
diiket Hb)

a. department on the alveolar PO2


b. the amount of O2 dissolved in the blood

c. the sum of the dissolved O2 plus the amount bound to hemoglobin

d. the sum of the dissolved O2 plus the amount of O2 bound to hemoglobin under saturating
conditions soalnya kaitan dengan po2 dan sat o2. Jadi harus tau seberapa jenuh hb sama
oksigen

e. Limited by O2 diffusion
f. the lung function test of a patient show a markedly reduced FEV1 sec
andFRC=reservefunctionalexpirasi+residual = 2,4 residual capacity of 4.2 L. Which is the following statement that
the most likely cause of the reduced FEV1sec?

a. weak respirator muscle


b. small diameter Airways obstruksi
c. pulmonary congestion

d. pulmonary fibrosis

e. dynamic compression of Airways

g. Mr.Smith complains of short of breath and dfficulty with moderate exercise. Pulmonary
function test indicate a reduced FRC, and his FEV1 sec was 2.6 L (78%). His Force vital capacity
was 3.1 L (70%). What is the most likely cause of Mr.Smith problem?

a. weak respirator muscle

b. small diameter Airways


2,6 / 3.1=83%

c. dynamic compression of Airways



d. pulmonary fibrosis gas exchange tergaanggu
e. pulmonary congestion

96. a 36 y.o woman undergoes chemotherapy with bleomycin for an ovarriann germ cell
cancer. She develops mild pulmonary fibrosis to the chemotherapy. Which agent’s diffusion
across the alveoli-pulmonary capillary Barrier would be most likely to be afffected by her
disease?Krnkoefisiendifusi O2 jauhlebihrendah, terus moa obatnyamembutuhkanoksigen,
terusada fibrosis lagi, jadimakinparah. O2 dan CO2 terganggu, liat koefisien difusi,

a.CO

b.CO2

C.N2O

D.O2

97. the 25 y.o woman poisoned with carbon monoxide. The carbon monoxide can bind with
hemoglobin and affected the oxygen binding capacity. What is the condition that decreased
PO2 in the arteial nblood of this patient?

a. dependent on alveolar PO2


b. dependen on the amount of CO2 bound to hemoglobin

c. increased from normal because of dissociated ooxygen from hemoglobin

d. reduced from normal because of the CO bound to hemoglobin

karena afinitas nya ke Hb lebih tinggi dr O2

2 which factor that can inhibit O2 binding to hb in the pulmonary capillary? Afinitas
menurun (pH rendah, pCO2 tinggi, konsentrasi H+ tinggi, temperature tinggi, 2,3 DPG tinggi)
a. CO2 dissociation from hb

b. diffusion of CO2 from pulmonary cappilary to alveolar


c. reduction of bicarbonate wit H+ shift to the left, afinitas naik d
shift to more acidic pH that it found invenous blood

3 a 47 week gestation infant is develop respiratory. What is the most probably mechanism
that will occurs with his First diaphhragmatic respiration?

Orang hebat adalah orang yang Melakukan Hal biasa secara luar biasa
a. paO2 increase
and saturation increase too

b. pulmonary vaskular resistance is increase


decrease, yg sistemik naik, makanya lung bisa inflasi


c. pulmonary cappilary hydrostatic pressure increased decreased d. syystematic

vascular resistance decrease increase

e. all of the fetal vascular channel functional Close

98. a 24 y.o present with sigi of hypoxia (hypoxic (pO2 dan saturasi menurun), anemic
(saturasi menurun), stagnant (sirkulasi menurun, pO2 dan saturasi normal),histotoxic
(masalah jaringan)). Arterial blood gases reveal that the PaO2 is normal but the arterial O2
saturation is reduced (anemic hypoxia). Which is the most likely cause of this patient
condition?

a. anemia

b. a low V/Q ratio

c. carbon monoxide
poisoning d. hypoventilation

e. righ to left Shunt

????78. a 70 y.o man admitted with chief complaint shortness of breath, cough, and yellowish
sputum. History DM +, physical examination: RR 30x/min, T:38.0 C, BP: 130/80 mmHg, ronchi
+/-, wheezing -/-, blood glucose 276 mg/dL, chest x-ray: infiltrate + (TB) . There is no history
of hospitalization previously. Which of the following describes most accurately about
physiological peculiarities found in the case above? ??????

a. the alveoli are filled with fluid due to increased hydrostatic


pressure b. the total surface area of respirator membran is decreased

c. the content of carbon dioxide in the blood will be decreasedkarena tb membuat scar
tissue
d. the physiologic dead space also become decrease

99. a result of ABG analysis of 25 y.o pneumothorax patient: pH:6.971, pCO2:71.4 mmHg,
pO2: 97.4 mmHg, HCO3:32.1 mEq, BE: +2.8, O2 saturation 96.4%. these findings suggest
homeostasis in preserving gas exchange to fulfill metabolism demand. Related to the case
above, in the normal condition on the other hand, the negative pressure in the pleura space
is allowed through which of the following factor?

a. strong and continuous alveolar epithelium

b. pumping of fluid from pleura space by lympathics


c. collection of large amounts of fluida in the pleura
space d. hydrostatic pressures at the capillary

e. osmotic Force at the alveolar membran of the lungs

pH: negative log of the hydrogen ion concentration

2 Blood pH in Adult : 7.35 – 7.45

3 Blood pH in Child : 7.36 – 7.44

4 PCO2: 35 – 45 mmHg
102. HCO3: 22 – 28 mEq /L

103. PO2 : > 80 mmHg

104. once you were on duty in emergency rol a 5 y.o girl patient was taken by her
prents with chief complaint stridor since 2 days ago. Her parents also complained about sore
throat, fever, difficulty on swallowing and breathing. Three was no cough and she was unable
to control her own saliva and began to drool.

Result from the physical examination are:

The child sat upright in a bent-forward position. The jaw was open and drooling was frequently
present. She presented a hot potato voice, suprasternal retraction and inspirator stridor.

Radiological findings:

From sofa tissue neck radiograph: “thumbepiglottitis sign” appearance, chest x ray: within normal
limit. Related to thhe case above, which of the following decribes most accurately about the
functions of the respirator passageways?

a.all passageways have cartilage to keep them from collapsing b.


all of the passageways are surrounded by smooth muscles

c. in obstructive diseases all of the passageways are constricted


While the assumptions of the Hagen–Poiseuille equation are not strictly true of

d. the greatest amount of resistance to airflow occurs in alveoli bronchioles


the respiratorytractitserves to show that, because of the fourth power,

relatively small changes in the radius of the airways causes large changes in
airway resistance.

e. in disease condition, the smaller bronchiles determine resistanceAnindividual small airway has much greater resistance than a large airway,
however there are many more small airways than large ones. Therefore,

resistance is greatest at the bronchi of intermediate size, in between the fourth

and eighth bifurcation

105. Related to the case above (80), WOTF describes most accurately about the cough
reflex? a. afferent nerve is mediated through trigeminal nerve (trigeminal.glossopharyngeal,
superior laryngeal vagus)

efferent : vagus, recurrent laryngeal nerve, corticospinal tract, perihperal nerve

b. the effect is the opening of glottis, followed by closing


closing dulu baru opening...

c. initially air is rapidly inspired


deep inhalation

d. both thoracal and abdominal muscles


relax e. the bronchi and trachea are dilated
For questions number 82 – 83:
A 36yo man came to clinic to control hisReflexpulmonaryStimulation TB. He felt better, but still
uncomfortable with the red sweat. He also failedCoughtoreceptors:get driving license because he could not
extrathoracic: nose, oropharynx, larynx, upper trachea

pass the Ishihara blind test. He felt strange because he always passed this kind of exam

intrathoracic: rapidly adapting irritant receptors in epithelium of lower trachea and large central bronchi

other locations: tympanic membrane, diaphragm, stomach

Afferent nerves: trigeminal, glossopharyngeal,th superior laryngeal, st vagus

before. He is a public transport driver. He is currently in the 4 week of 1 category TB

Efferent nerves: recurrent laryngeal nerves, vagus, corticospinal tract and peripheral nerves

Reflex begins with deep inspiration, followed by glottic closure, diaphragmatic relaxation, and thoracic and

therapy. abdominal expiratory muscle contraction. Posterior wall of airway invaginates and causes shearing of mucus.

Positive pleural pressure generated up to 100-300 mm Hg; peak flows of 12 L/sec

106. WOTF is the most appropriate regiment for this


patient? a. 2HRZE
b. 2(HRZE)S yg kategori 1 = 2HRZE / 4 H3R3

111. 2HRZ/4(HR)3

112. 2(HRZE)S/HRZE

Orang hebat adalah orang yang Melakukan Hal biasa secara luar biasa
e. 2(HRZE)S/HRZE/4(HR)3

kategori 2

112. WOTF should be avoided in this


patient? a. Isoniazid

b. Rifampicin c.
Ethambutol d.
Pyrazinamide e.
Streptomycin

113. A 27yo woman came to primary health care with hemoptoe. She was diagnosed
with lung TB a few
months before her
pregnancy. She said she
had 1 month of
continuation phaseThe preferred initial treatment regimen is INH, rifampin (RIF), and ethambutol (EMB) daily for 2 months, followed by INH and RIF daily, or twice
weekly for 7 months (for a total of 9 months of treatment).
Streptomycin should not be used because it has been shown to have harmful effects on the fetus.
therapy using R and HPyrazinamide (PZA) is not recommended to be used because its effect on the fetus is unknown.v
before stopping the
treatment because she was afraid it might harm her fetus. One week ago, she delivered a
healthy baby.

WOTF is the most appropriate regiment for her initial phase?

RHZ

4RH

5(HR)3E3

d. 2RHZES

e. 2(HRZE)S/(HRZE)

85. A 48yo man was diagnosed with acute pharyngitis and was given erythromycin, ibuprofen,
and bromhexim by another doctor two days ago. Yet, he complains of diarrhea and vomiting
that he has been suffering since yesterday. His medication has already been used up.

WOTF is/ the most appropriate thing to deal with his complaint?

a. Change ibuprofen

b. Change bromhexim

c. Change erythromycin

d. Give antacid
e. Continue the medication wit additional drug for his complaint

86. A 38yo man was presented to the ER due to shortness of breath. He has been feeling
that symptom since this afternoon after he took propranolol for his palpitation. He got the
drug from another doctor in a clinic this morning. You give him theophylline 300 mg 2x1. If
the patient has a liver dysfunction, why should you change the treatment? a. Toxicity
potency

b. Reduction of efficacy Co-administration of theophylline with propranolol decreases theophylline clearance by 33% to
52%.

c. Increasing of clearance

d. Progression of liver disease

e. Shortened duration of action

For number 87 to 89:

A 60yo man with history of smoking for > 20 years (2 packs a day) complained of cough,
dyspnea, and wheezing. His symptoms have become severe that his family brought him to
the ER. PE revealed diaphoresis, dyspnea, tachycardia, and tachypnea. RR 30/min, PR 112
bpm, BP 130/60 mmHg.

For each statement below, choose the most appropriate drug from the following list:

c. inhaled cromolyn antihistamin

d. inhaled salbutamol B2 agonist bronchodilator

e. oral/IV methylprednisolone

f. inhaled ipraptropium bromide antimuscarinic

g. IV propanolol non selective B blocker, kurangi HR

2 the most appropriate drug as a rapid bronchodilator (B)

3 the most likely drug to provide sustained resolution of the inflammatory symptoms (C

4 a contraindicated drug for this patient (E)


copd kontraindikasi dg betablocker , kr efeknya bisa bronkokonstriksi

walaupun akan mengurangi HR dan BP, propanolol non selektive soalnya

For number 90 – 94:

a. INH
sintesis mikolic acid

b. Rifampicin
sintesis dna

c. PZA
d. Ethambutol

sintesis cell wall arabinose [?]

121. This drug interacts with the β-subunit bacterial DNA-dependent RNA
polymerase and thereby inhibit synthesis of RNA (B)

For question 90-94

A 36 yo woman came to your clinic complained that he had prolong cough, night swear, body
weakness, loss of appetite and mild fever since two month ago. After taking history, physical
examination and preliminary investigation, he is diagnosed with pulmonary TB. He is then
placed on a four drug regiment consisting of INH, rifampisin, PZA, and ethambutol. For each
of statement below choose the most appropriate drug from the following list:

C. INH

D. Rifampicin

E. PZA
Ethambutol
129. The drug interacts with the beta subunit bacterial DNA dependent RNA
polymerase and therapy inhibits RNA synthesis (B)

130. The drug reacts with pyridoxine which can cause deficiency of this vitaminA (A)

131. The drug inhibit the metabolism of phenytoin (A)

132. The drug may caused red-green color blindness (D)

133. The drug crosses inflamed meninges (C)

PYRAZINAMIDE:

It crosses inflamed meninges and is an essential part of the treatment of tuberculous meningitis.

It is metabolised by the liver and the metabolic products are excreted by the kidneys.

Orang hebat adalah orang yang MelakukanPyrazinamideisonlyHalused inbiasacombinationsecarawithotherluardrugs biasasuchasisoniazid and rifampicin in the
treatment of Mycobacterium tuberculosis. It is never used on its own. It has no other indicated medical uses.
130. A 30 yo man complained dyspnea and fever. His symptoms became severe that
his family brought him to the emergency room. Physical examination revealed respiratory rate
was 32x/min, pulse rate was 112 beats per minute, temperature 40C, and blood pressure was
130/80 mmHg, with crackles in the inferior lobe of left pulmo. Others were within normal
limit, Hb 12 g/dl, WBC 27.000/mm3. Chest x-ray showed inferior left lob infiltrate. He had
history of penicillin and cephalosphorin allergy. The doctor which of the following drug of
choice for this patient?

A. Oral cefuroxime
B. Oral azithromycin

C. IV azithromycin (macrolyde)karenadiIGDkasih
IV D. IV beta lactam + oral azithromycin E.
IV beta lactam + IV azithromycin b. Doxycycline, 100 mg orally twice a day.

For previously healthy patients who have not taken antibiotics within the past 3 months:

a. A macrolide (clarithromycin, 500 mg orally twice a day; or azithromycin, 500 mg orally as a first dose and then 250 mg

96. A 4 yo boy present to emergencyorallyroomdailyforwith4days, orstridor,500mgorallycoughdailyfor3 anddays),orslightly hard to breath.

Physical examination revealed respiratory rate was 42x/min, pulse rate was 102 bpm,
Inpatient management not requiring intensive care

temperature 18C and suprasternal retraction1.Arespiratory.fluoroquinoloneOtherswas within normal limit. Hb 12 g/dl,


a. See above for oral therapy.

b. For intravenous therapy, moxifloxacin, 400 mg daily; levofloxacin, 750 mg daily; ciprofloxacin, 400 mg every 8–12 hours

WBC 7000/mm3.Chest x-ray withinor normal limit. He had history of penicillin and
2. A macrolide plus a beta-lactam
cephalosphorin allergy. Which of the following drug the most appropriate for this patient?

a. See above for oral therapy.

b. For intravenous therapy, ampicillin, 1–2 g every 4–6 hours; cefotaxime, 1–2 g every 4–12 hours; ceftriaxone, 1–2 g every

12–24 hours

A. Erythromycin 1x/day
di aafp anak2 sudah boleh pake azithromycin, dan jadi pil utama obat pneumoni

B. Erythromycin 2x/day
ini dosisnya bener cuma jarang pake eryth kr adr nyaa intolerate.....

C. Azithromycin 3x/day (pada orang dewasa)


dosisnya 1x sehari

D. Clarithromycin 1x/day

E. Clarithromycin 3x/day

2x sehari

132. A 15 yo girls came to the primary health care with problem difficulty in breathing.
The problem occurred after she cleaned her room this morning. In physical examination,
found wheexing during expiration. What do you think the etiology for her problem?
A. Funggus
B. Dust mite
C. Viral

D. Bacteria
E. Microflora

133. The statement which is wrong related with the etiology is:

A. Survive in all climate but mostly in humid condition

B. Well survive in carpet

C. Prefer outdoor environment

D. Consume minute particle in organic matter

E. Can be found in high altitude

99. One way to eradicate is:

A. Maintaining humidity above 50%

B. Exposing them to temperature above 30C


C. Using abate powder

D. Regularly cleaning and washing

E. Sweep the carpet

134. A 60 yo man has a 5 month history of progressive weakness and a weight los of 13 kg
along with intermittent fever, chills and a chronic cough production of yellow sputum. Culture
of the sputum is positive for Mycobacteriom tuberculosis. Media for M. tuberculosis is:

A. Mac Conkey

B. Horse Blood Agar


C. Loeffler Agar

D. Lowenstein
Jensen E. Kligler Agar

135. Staining for M Tuberculosis?

A. Gram staining

B. Neisser staining

C. Ziehl Nelson staining

D. Giemsa staining

E. Negative staining

136. Criteria for reporting the level of M Tuberculosis according to IUALTD in staining:

1(+) if find >10 AFB/100 HPF

3(+) if find >10 AFB/HPF

1(+) if find 1-10 AFB/100 HPF

(-) if find only 9 AFB/100 HPF

2(+) if find >10 AFB/HPF

137. 3 y.o develops Haemophilus influenza. Culture of nasopharynx swab specimen in blood

agar with X and V factor. What is the morphology of this strain?


haemophilus influenza

A. Gram (+) cocci

B. Gram (-) bacilli


coccibacili

C. Diplococci gram (+)


D. Diplococci gram (-)

E. Can’t stain by gram staining

104. For growth, H. influenza in blood agar with X and V factor. V factor contains:

A. Folic acid

B. Nicotinamide adenine dinucleotidefactorV :factor X

C. Bacitrasin

D. Heme= factor V
factor X

E. Nicotinic acid

Orang hebat adalah orang yang Melakukan Hal biasa secara luar biasa
136. 2 y.o boy lives with his grandpa that has a chronic bloody cough since the last 4 months.
The boy has poor weight gain, loss appetite, prolonged fever, and moderate malnutrition.
Wotf diagnostic examination is likely to result in the correct diagnosis of this boy?

A. Bronchoscopy

B. Tuberculin skin test and chest X ray

C. Tuberculin skin test only

D. Chest X ray only

E. ESR

137. 16 month old girl with the 3 months history of weight loss, recurrent low grade fever
with the tuberculin skin test 15 mm, and the chest X ray shows an enlargement of the hilar
lymph node, the nutritional status was moderate malnutrition. The most likely diagnosis of
this patient is:

A. Lymphadenitis TB

B. Latent TB infection

C. Pulmonary TB

D. Miliary TB hematogenous primary TB
E. Pneumonic type TB
What Are False-Positive Reactions?
Some persons may react to the TST even though they are not infected with M.
tuberculosis. The causes of these false-positive reactions may include, but are
not limited to, the following:

 Infection with nontuberculosis mycobacteria



 Previous BCG vaccination

 Incorrect method of TST administration

 Incorrect interpretation of reaction

 Incorrect bottle of antigen used

What Are False-Negative Reactions?

Some persons may not react to the TST even though they are infected with M.
tuberculosis. The reasons for these false-negative reactions may include, but are
not limited to, the following:

 Cutaneous anergy (anergy is the inability to react to skin tests


because of a weakened immune system)

 Recent TB infection (within 8-10 weeks of exposure)

 Very old TB infection (many years)

 Very young age (less than 6 months old)

 Recent live-virus vaccination (e.g., measles and smallpox)

 Overwhelming TB disease

 Some viral illnesses (e.g., measles and chicken pox) 

 Incorrect method of TST administration

 Incorrect interpretation of reaction

http://www.cdc.gov/tb/publications/factsheets/testing/skintesting.htm

2 13 month old boy came to pediatric clinic with chief complaint of a poor weight gain and
a 3 weeks cough. Physical findings are moderate malnutrition, enlargement of the neck lymph
node (confluent), looks tired. Tuberculin skin test was reactive with diameter of induration 17
mm, the chest X ray showed an infiltrate in bilateral perihilar. The appropriate

treatment of this patient is:

OAT anak: 2HRZ/4HR

A. INH prophylaxis

141. INH, rifampicin for 6 months

142. INH, rifampin for 2 months

143. INH, rifampicin, pyrazinamide for 6 months

144. INH, rifampicin, pyrazinamide for the first 2 months and INH, rifampicin for the

following 4 months kategori anak 2HRZ/4HR

Sistem skor gejala dan pemeriksaan penunjang TB (pedoman TB hal 17-


18)

Orang hebat adalah orang yang Melakukan Hal biasa secara luar biasa
Catatan :

2 Diagnosis dengan sistem skoring ditegakkan oleh dokter.

3 Batuk dimasukkan dalam skor setelah disingkirkan penyebab batuk


kronik lainnya seperti Asma, Sinusitis, dan lain-lain.

4 Jika dijumpai skrofuloderma (TB pada kelenjar dan kulit), pasien


dapat langsung didiagnosis tuberkulosis.

5 Berat badan dinilai saat pasien datang (moment opname).-->


lampirkan tabel berat badan.

6 Foto toraks bukan alat diagnostik utama pada TB anak

7 Semua anak dengan reaksi cepat BCG (reaksi lokal timbul < 7 hari

setelah penyuntikan) harus dievaluasi dengan sistem skoring TB anak.

142. Anak didiagnosis TB jika jumlah skor > 6, (skor maksimal 14)
143. Pasien usia balita yang mendapat skor 5, dirujuk ke RS untuk
evaluasi lebih lanjut.

Setelah dokter melakukan anamnesis, pemeriksaan fisik, dan pemeriksaan


penunjang, maka dilakukan pembobotan dengan sistem skor. Pasien dengan
jumlah skor yang lebih atau sama dengan 6 (>6), harus ditatalaksana sebagai
pasien TB dan mendapat OAT (obat anti tuberkulosis). Bila skor kurang dari 6
tetapi secara klinis kecurigaan kearah TB kuat maka perlu dilakukan
pemeriksaan diagnostik lainnya sesuai indikasi, seperti bilasan lambung,
patologi anatomi, pungsi lumbal, pungsi pleura, foto tulang dan sendi,
funduskopi, CT-Scan, dan lain lainnya.

2 3 y.o boy without any clinical manifestations of TB (0), chest X ray examination within

normal limit (0), tuberculin skin test was reactive. His father has been diagnosed as
pulmonary TB with positive AFB in sputum. The appropriate prophylaxis management of this
child is:

143. INH and rifampicin for 2 months

144. INH and rifampicin for 3 months

145. INH for 6 months

146. INH for 9 months

147. INH for 12 months

Pengobatan Pencegahan (Profilaksis) Tuberkulosis untuk Anak Pada


semua anak, terutama balita yang tinggal serumah atau kontak erat
dengan penderita TB dengan BTA positif, perlu dilakukan pemeriksaan
menggunakan sistem skoring. Bila hasil evaluasi dengan sistem skoring
didapat skor < 5, kepada anak tersebut diberikan Isoniazid (INH) dengan
dosis 5-10 mg/kg BB/hari selama 6 bulan. Bila anak tersebut belum
pernah mendapat imunisasi BCG, imunisasi BCG dilakukan setelah
pengobatan pencegahan selesai.

2 The following answer is the component of scoring system for pediatric TB in Indonesia’s
national TB program (NTP):

a. Fever more than 1 week (2 wk)

b. Cough more than 2 weeks (3 wk)

c. (+) reaction of tuberculin

d. (+) contact with other child diagnosed TB (keluarga)

e. Palpable neck lymph node of 0.5 cm (1 cm)

7 month old baby was brought to pediatric emergency with the chief complaint of difficulty in
breathing since the last 3 days. The physical findings are tachypnea, chest indrawing, crackles,
fever. Chest X ray showed a patchy infiltrate in the bilateral hemithorax.

151. the most likely diagnosis in this patient is:

a Lobar pneumonia (uniformly consolidation)

b Bronchopneumonia (patches of consolidation)

c Bronchiolitis

d Bronchitis

e Laryngotracheobronchitis
152. In case above, what is the initial management????????? Very severe :
clorampenicol a. Oral amphycillin

b. oral macrolide
c. IV gentamycin

d. IV amphycillin

IV amoxicillin bkn ampicillin

e. IV ampicillin

2 An 11 y.o. gorl came to pediatric ER RSHS with a chief complaint of difficulty breathing
and cough since the last 3 hours. Physical exam found wheezing, patient prefer in sitting
position RR was 45/mnt. The most appropriate diagnosis of this patient is :

Orang hebat adalah orang yang Melakukan Hal biasa secara luar biasa
RR, PR, wheezing

a. Mild asthma exacerbation : walking, can lie down

b. Moderate asthma exacerbation : talking, infant = difficulty breathing, prefer sitting

c. severe asthma exacerbation : hunchbreathing

d. impending respi failure

e. respi failure
161. initial management of the above case
: a. inhaled corticosteroid

b. inhaled ephinephrine


c. inhaled SABA reliever
d. inhaled LABA

e. inhaled anticholinergic
For question number 114 to 115, refer to scenario below :

A 4-y.o. boy were taken by his parents to the emergency room with stidor since 2 days ago
which becoming worse in time. The chief complaint was accompanied by sore throat, fever,
that increasing gradually, difficulty of swallowing and breathing. On PE : fully alert, BP 95/60
mmHg, PR 110bpm, RR 42x/mnt, T 39.5 degree celcius. The child upright in a bent-forward
position. The jaw was open and drooling was frequently present. He has a hot potatoe voice,
supasternal retraction and inspiratory stridor. Lab test shows leukocytosis and from soft tissue
neck. Radiograph seen “thumb sign” appearance

In radiology, the thumbprint sign, or Initiate antibiotics to provide empiric coverage of the most likely bacterial pathogens in the context of the
clinical setting. Treatment should continue for 7-10 days, in general.
thumbprinting, is a radiologic sign
found on a lateral C-spine radiograph that suggests the diagnosis of epiglottitis. The
sign is caused by a thickened free edge of the epiglottis, which causes it to appear
more radiopaque than normal, resembling the distal thumb.

2 WOTF structure that inspiratory stridor most likely would be expected to lie with
lesion?

a. adenoid area

b. lower one-third of the trachea


c. right main stem bronchus
d. nasal area

e. glottic area : biphasic daninspratory stridor (sebenernya inspiratory stridor di sub glotic)

3 Medical management for this condition is

a. not frequently effective

b. directed toward more resistant bacteria

c. usually targeted toward spesific bacteria and broad spectrum coverage is not
warranted d. universally effective

e. best provided with IV therapy

For questions number 116 to 117, refer to scenario below:

170. 50-y.o. man complaints dypnea, when he walks in arush. Complaints


accompanied with cough and whitish sputum for the last 5 years. He is a heavy smoker, with
a history of

Orang hebat adalah orang yang Melakukan Hal biasa secara luar biasa
smoking about 30 pack/year cigarettes since he was 17 y.o. PE found CVS within normal
limits. His body weight is 55 kg and height is 175 cm.

2 WOTF is the most likely condition? a.


he needs more energy due to smoking

b. he needs more energy due to increased work of breathing c.


he needs more energy due to preserving fat mass

d. he needs more energy due to preserving visceral fat mass

e. he needs more energy due to preserving subcutaneous fat mass

3 what is teh most appropriate nutrition recommended??????????

a. additional folic acid above Recommended Dietary Allowance (RDA) is necessary

b. additional vitamin C above Recommended Dietary Allowance (RDA) is necessary

c. Additional vitamin A above Recommended Dietary Allowance (RDA) is necessary antioxidant??? d.


Additional vitamin B above Recommended Dietary Allowance (RDA) is necessary e. Additional
vitamin B12 above Recommended Dietary Allowance (RDA) is necessary

For number 118 to 119, refer to the scenario below :

A 35 y.o. came to your clinic complaining cough and dyspnea. He is known as a heavy smoker.
During PE BP 120/80 mmHg, PR 120x/min, RR 30x/min, T 38.2 degree celcius, chest exam :
barrel chest. BGA : pH 7.28, PaCO2 60, SaO2 90%

pH: negative log of the hydrogen ion concentration

• Blood pH in Adult : 7.35 – 7.45

• Blood pH in Child : 7.36 – 7.44

• PCO2: 35 – 45 mmHg

• HCO3: 22 – 28 mEq /L

• PO2 : > 80 mmHg


118.WOTF energy intake is the most appropriate ?

a. Increasing energy needs with 65% calories from carbohydrate and 20% calories from fat b.
Increasing energy needs with 60% calories from carbohydrate and 25% calories from fat c.
Increasing energy needs with 50% calories from carbohydrate and 35% calories from fat d.
Increasing energy needs with 65% calories from carbohydrate and 35% calories from fat e.
Increasing energy needs with 70% calories from carbohydrate and 30% calories from fat

119. Which of the most appropriate statement due to substrate metabolism


use? a. Respiratory Quotient (RQ) for carbo is 0.7, protein 1, and fat 0,8

b. Respiratory Quotient (RQ) for carbo is 1, protein 0.8, and fat 0.7 c.
Respiratory Quotient (RQ) for carbo is 0.7, protein 0.8, and fat 1 d.
Respiratory Quotient (RQ) for carbo is 0.8, protein 0.7, and fat 1
e. Respiratory Quotient (RQ) for carbo is 0.7, protein 1, and fat 0,7

The respiratory quotient (or RQ or respiratory coefficient), is a dimensionless number used


in calculations of basal
metabolic rate (BMR) when estimated from carbon dioxide production.
calorimetry. It is
Such measurements, like measurements of oxygen uptake, are forms of indirect
measured using Ganong's Respirometer.

It can be used in the alveolar gas equation.

The respiratory quotient (RQ) is calculated from the ratio:

RQ = CO2 eliminated / O2 consumed

where the term "eliminated" refers to carbon dioxide (CO2) removed ("eliminated") from the body.

In this calculation, the CO2 and O2 must be given in the same units, and in quantities proportional to the
number of molecules. Acceptable inputs would be either moles, or else volumes of gas at standard
temperature and pressure (time units may be included, but they cancel out since they must be the
same in numerator and denominator).

Many metabolized substances are compounds containing only the elements carbon, hydrogen,
and oxygen. Examples include fatty

acids, glycerol, carbohydrates, deamination products, andethanol. For complete

oxidation of such compounds, the chemical equation is

CxHyOz + (x + y/4 - z/2) O2 ---> x CO2 + (y/2) H2O

and thus metabolism of this compound gives an RQ of x/(x + y/4 - z/2).

The range of respiratory coefficients for organisms in metabolic balance usually ranges from 1.0
(representing the value expected for pure carbohydrate oxidation) to ~0.7 (the value expected for
pure fat oxidation). See BMR for a discussion of how these numbers are derived. A mixed diet of fat
and carbohydrate results in an average value between these numbers. An RQ may rise above 1.0 for
an organism burning carbohydrate to produce or "lay down" fat (for example, a bear preparing for
hibernation).

RQ value corresponds to a caloric value for each liter (L) of CO 2 produced. If O2 consumption numbers
are available, they are usually used directly, since they are more direct and reliable estimates of energy
production.

RQ as measured includes a contribution from the energy produced from protein. However, due to the
complexity of the various ways in which different amino acids can be metabolized, no singleRQ can
be assigned to the oxidation of protein in the diet.
Orang hebat adalah orang yang Melakukan Hal biasa secara luar biasa
120. WOTF statement is the most appropriate

a. Respiratory Quotient (RQ) is the ratio of the volume of O2 expired to the volume of CO2
inspired

b. Respiratory Quotient (RQ) is the ratio of the volume of O2 inspired to the volume of CO2
expired

c. Respiratory Quotient (RQ) is the ratio of the volume of CO2 inspired to the volume O2
expired

d. Respiratory Quotient (RQ) is the ratio of the volume of CO2 expired to the volume of O2
inspired

e. Respiratory Quotient (RQ) is the ratio of the volume of CO2

121. Which food intake method to avoid aspiration????????

a. Rest before eat meals, oral route, large portion of nutrient-dense food, proper sitting
position
b. Deep nutrient food, proper sitting position

c. Rest before eat meals, parenteral route, small portion of nutrient-dense food, proper
sitting position

d. Deep breathing during eat, oral route, small portion of nutrient-dense food, proper sitting
position

e. Rest before eat meals, oral route, small portion of nutrient-dense food, proper sitting
position

122. Which of the following statement that most accurate nutrient intake???????

A. Caloric needs ranging from 94% to 164% of predicted range and protein needs
ranging from 1,2 to 1,7 g/kg of body weight

B. Caloric needs ranging from 80% to 90% of predicted range and protein needs
ranging from 0,6 to 0,8 g/kg of body weight

C. Caloric needs ranging from 80% to 100% of predicted range and protein needs
ranging from 0,8 to 1 g/kg of body weight
D. Caloric needs ranging from 60% to 90% of predicted range and protein needs
ranging from 0,5 to 0,8 g/kg of body weight

E. Caloric needs ranging from 90% to 110% of predicted range and protein needs
ranging from 1,6 to 0,8 g/kg of body weight

123. A 3 yo boy arrived in an emergency department with a 3-day history of left ear pain and
a 5-day of common cold, fever. He has been seen by his primary care physician the day before
and started on amoxicillin (40 mg/kg/day) divided three times a day 1 tea spoon. The patient
had a history of recurrent otitis media requiring four to five courses of antibiotics per year
since birth. Physical examination revealed alert boy with temperature of 39° C. Otoscopy
shows tympanic membrane hyperemic on the left side.

Which of the following is the most common organism causing acute otitis media?

A. Haemophillus influenza

B. Moraxella catarrhalis

C. Streptococcus pneumoniae

D. Staphylococcus aureus

E. Klebsiellapneumoniae

Otitis media is inflammation of the middle ear, and may present as either
acute otitis media (AOM) or otitis media with effusion (OME). AOM
exhibits rapid-onset middle ear effusion and signs and symptoms of
middle ear inflammation, including fever, otalgia, otorrhoea, or
irritability,1 whereas OME is middle ear effusion in the absence of
symptoms of acute infection.

 Common viruses that cause upper respiratory tract infection are


frequently associated with AOM and new-onset OME. These include
respiratory syncytial virus, rhinovirus, adenovirus, parainfluenza
and coronavirus.

 Predominant bacteria that cause otitis media are Streptococcus


pneumoniae, Moraxella catarrhalis, and non-typeable Haemophilus
influenzae.

124. Which of the following is middle ear


contain? A. Auriculotemporalis nerve

B. Glossopharyngeal nerve

C. Semicircularcanal telinga dalam

D. The corda tympani The chorda tympani is a nerve that originates from the taste
buds in the front of the tongue, runs through the middle ear, and carries taste messages
to the brain. It joins the facial nerve (cranial nerve VII) inside the facial canal, at the level
where the facial nerve exits the skull via the stylomastoid foramen.

E. Utriculus The utricle, or utriculus (Latin: utriculus, diminutive of utur, meaning "leather bag"), along with

the saccule is one of the two otolith organs located in the vertebrate inner ear. The utricle and the saccule are parts of the
balancing apparatus (membranous labyrinth) located within the vestibule of the bony labyrinth (small oval chamber). These

Orang hebat adalah orang yang Melakukan Hal biasa secara luar biasa
use small stones and a viscous fluid to stimulate hair cells to detect motion and orientation. The utricle detects linear
accelerations and head tilts in the horizontal plane.

telinga luar terdiri dari daun telinga, lubang telinga, dan saluran telinga luar. Telinga
luar meliputidaun telinga atau pinna, Liang telinga atau meatus auditorius eksternus,
dan gendang telinga atau membran timpani.

125. Plexus kiesselbachkecuali:



A. Sphenopalatine maxillary artery
B. Inferior labial

C. Greater palatine maxillary a.

D. Superior labial facial a.


E. Anterior ethmoid ophtalmic a. & posterior ethmoidal a.

126. Kasus rinosinusitis, what are the appropriate and latest diagnostic tools to see every
aspects of this disease? Mucopurulence, edema, septal deflection, polyps

A. Poste rhinoscopy and ct scan

B. Translumination and sinus paranasal radiologic


projection C. Ct scan and transillumination
D. Nasal endoscopy and ct scan

E. Water's and caldwell position radiology

127. What is the latest surgery for this disease? Sinus surgery, (sebelumnya dikasih antibiotik
4-6 minggu, nasal steroid, systemic steroid therapy)

A. Caldwell lucoperation : chronic maxillary sinusitis (19 century)

B. Endoscopic sinus surgery

C. Anthrostomy washed out

D. Ethmoidectomy intra anthral

E. Turbinectomy reduction

128. Swelling in sublingual, submandibular, and submental, elevated tongue, and inspiratory
stridor. Diagnosis:
A. Peritonsilar abscess

B. Retropharyngeal abscess

C. Parapharyngeal abscess

D. Ludwig angina

E. Danger space infection

129. What is the initial step in this condition


? A. Tracheostomy

B. Broad Spectrum Antibiotic :soalnya kata cipongdiamasihbengkak doing, belomada pus


dll C. Incicion and drainage

D. Surgery
E. Palliative

130. A 25 years old woman with chief complain bloody sputum. Two years ago has been
treated bFAILU. What category of this patient?

A. Relapse

B. Treatment failure

C. Treatment after interuption (after default)


default = putus obat

D. New case

E. Chronic case

c. Klasifikasi berdasarkan riwayat pengobatan sebelumnya

Klasifikasi berdasarkan riwayat pengobatan sebelumnya disebut sebagai

tipe pasien, yaitu:

1) Kasus baru

Adalah pasien yang belum pernah diobati dengan OAT atau sudah

pernah menelan OAT kurang dari satu bulan (4 minggu). Pemeriksaan

BTA bisa positif atau negatif

2) Kasus yang sebelumnya diobati

Kasus kambuh (Relaps)


Adalah pasien tuberkulosis yang sebelumnya pernah mendapat

pengobatan tuberkulosis dan telah dinyatakan sembuh atau

pengobatan lengkap, didiagnosis kembali dengan BTA positif


(apusan atau kultur).

Kasus setelah putus berobat (Default )


Adalah pasien yang telah berobat dan putus berobat 2 bulan atau

lebih dengan BTA positif.

Kasus setelah gagal (Failure)


Adalah pasien yang hasil pemeriksaan dahaknya tetap positif atau

kembali menjadi positif pada bulan kelima atau lebih selama

pengobatan.

3). Kasus Pindahan (Transfer In)

Adalah pasien yang dipindahkan keregister lain untuk melanjutkan

pengobatannya.

4). Kasus lain:

Adalah semua kasus yang tidak memenuhi ketentuan diatas, seperti yang

i. tidak diketahui riwayat pengobatan sebelumnya,

ii. pernah diobati tetapi tidak diketahui hasil pengobatannya,

iii. kembali diobati dengan BTA negative.

131. A 62 years old man come to your clinic because he is concern about increasing shortness
of breath. He complaint about cough and increased sputum production for the last 5 years.
He smoked since of 15 years old and he stopped smoking since 2 years ago. His

chest xray showed hyperinflation, flattened diaphragm, no infiltrate was found.


Spirometricresult showed FEV1/VC 60% and FEV1 45% predicted, ECG tracing was normal.
The PE which would not be found :

A. Crackles

B. Prolonged expiratory time

C. Wheezing during auscultation

D. Hypersonor on percussion

Orang hebat adalah orang yang Melakukan Hal biasa secara luar biasa
E. Purse-lip breathing

The characteristic symptoms of COPD are cough,

sputum production, and dyspnea upon exertion. Chronic

cough and sputum production often precede the

development of airflow limitation by many years, although

not all individuals with cough and sputum production go

on to develop COPD. This pattern offers a unique

opportunity to identify those at risk for COPD and

intervene when the disease is not yet a health problem.

A major objective of GOLD is to increase awareness

among health care providers and the general public of

the significance of these symptoms.

Stage 0: At Risk - Characterized by chronic cough and

sputum production. Lung function, as measured by

spirometry, is still normal.

Stage I: Mild COPD - Characterized by mild airflow

limitation (FEV1/FVC < 70% but FEV1 > 80% predicted)

and usually, but not always, by chronic cough and sputum

production. At this stage, the individual may not even be

aware that his or her lung function is abnormal. This

underscores the importance of health care providers

doing spirometry in all smokers so that their lung function

can be observed and recorded over time.

Stage II: Moderate COPD - Characterized by worsening

airflow limitation (50% < FEV1 < 80% predicted), and

usually the progression of symptoms with shortness of

breath typically developing on exertion. This is the stage

at which patients typically seek medical attention

because of dyspnea or an exacerbation of their disease.

Stage III: Severe COPD - Characterized by further

worsening of airflow limitation (30% ≤ FEV1 < 50% predicted),

increased shortness of breath, and repeated

exacerbations which have an impact on patients’ quality

of life.

Stage IV: Very Severe COPD - Characterized by severe

airflow limitation (FEV1 < 30% predicted) or the presence

of chronic respiratory failure. Respiratory failure is

defined as an arterial partial pressure of O2 (PaO2) less

than 8.0 kPa (60 mmHg) with or without arterial partial

pressure of CO2 (PaCO2) greater than 6.7 kPa (50 mm

Hg) while breathing air at sea level. Respiratory failure

may also lead to effects on the heart such as cor

pulmonale (right heart failure). Clinical signs of cor


pulmonale include elevation of the jugular venous pressure

and pitting ankle edema. Patients may have Stage IV:

Very Severe COPD even if the FEV1 is > 30% predicted,

whenever these complications are present. At this stage,

quality of life is very appreciably impaired and exacerbations

may be life threatening.

132. A 20 years old man come to your clinic with chief complaint of cough and sputum more
than 5 weeks. The complaints were accompanied by chest wall pain, low grade of fever and
night sweating. He had taken antibiotic, but showed no improvement. Two specimens o
sputum of FAB show positive. One year ago he had history of TB treatment for 2 months, and
discontinued this drug because of felt better and cured. Chest x-ray showed lesion in upper
parts of right lung. Which of the following is the Gold standard to assess respiratory failure ?

A. BGA

B. Chest x ray C.
Spirometry D. CT
scan thorax

E. Pulmonary angiography
133. A 65 years old male with COPD stage III (TNF-gamma) and a woman 23 year old with
asthma bronchial moderate persistent (TNF-alpha). Both were taken inhalation corticosterois.
While the women gain improvement from this therapy the man is not. Possible explanation
for this is :

A. Gender difference between


them B. Diference in age

C. Difference in spirometri result


D. Difference in risk factors

E. Difference in inflammatory cell and mediators

134. Exercise which is recommended in asthma is :

A. Deep breathing exercise

B. Chest expansion breathing exercise

C. Strengthening diaphragmatic exercise

D. Strengthening muscles of extremities

E. Aerobic exercise

activities that involve short, intermittent periods of exertion such as volleyball, gymnastics, baseball and wrestling
are generally well tolerated by people with symptoms of asthma.

Activities that involve long periods of exertion, such as soccer, distance running, basketball, and field hockey, may be less
well tolerated. Also less well tolerated are cold weather sports such as ice hockey, cross-country skiing, and ice-skating.
However, many people with asthma are able to fully participate in these activities.

Swimming, which is a strong endurance sport, is generally well tolerated by many people with asthma because it is usually
performed in a warm, moist air environment. It is also an excellent activity for maintaining physical fitness.

Other beneficial activities for people with asthma include both outdoor and indoor biking, aerobics, walking, or running on a
treadmill.

What Should I Do to Control My Asthma When I Exercise?

Always use your pre-exercise asthma inhalers (inhaled bronchodilators)


stage 3
before beginning exercise.

 Perform warm-up exercises and maintain an appropriate cool-down period after exercise.

 If the weather is cold, exercise indoors or wear a mask or scarf over your nose and mouth.

If you have allergic asthma, avoid exercising outdoors when pollen counts are high and when there is high air pollution.

Restrict exercise when you have a viral infection, like a cold.

 Exercise at a level that is appropriate for you.



Maintaining an active lifestyle is important for both physical and mental health. Remember: asthma is not a reason to avoid
exercise. With proper diagnosis and the most effective treatment, you should be able to enjoy the benefits of an exercise
program without experiencing asthma symptoms.

135. A 65 year old man comes to your clinic because he is concern about increasing shortness
of breath. He complains about cough and increases sputum production for the last

5 years. He started to smoke at age 15 year old. His chest xray shows hyperinflation, no infiltrate
founded. Spirometry result shown FEV1/FVC 60% and FEV1 45% predicted, ECG is

normal.

Which of the following treatment is NOT recommended for stage of this patient’s disease?

A. Rehabilitation

B. Bronchodilator

C. ICS

Orang hebat adalah orang yang Melakukan Hal biasa secara luar biasa
D. Long term O2

E. Influenza Vaaccination

136. A man, 67 years old, complained cough with whitish sputum since about 2 years ago.
His spirometer examination showed FEV1/FVC ratio was 65% and FEV1 is 82% from
stage 2

predicted value. Beside avoidance from risk factor and influenza vaccination, other
medication for him is :

a. Inhaled regular long acting β2 agonist

b. Inhaled corticosteroid

c. Rehabilitation

d. Regular oral methylxanthyn

e. Inhaled short acting β2 agonist as needed

137. A male patient is suffering pulmonary tuberculosis. He is now in the second month of
intensive phase of the treatment. Three days ago he began to develop vision disturbance.
Which one of these drug is the most likely caused?

a. Rifampicin

b. Pyrazinamide

c. Ethambutol

d. Isoniazide

e. Combination ofisoniazide and rifampicin

138.Perry, an 18 years old previously healthy medical student come to you with difficulty in
breathing. He began to feel this symptom 3 days ago. He also complained high fever,
cough with yellow thick sputum. On physical examination chest movement was
asymmetric, right hemithorax : tactile fremitus was increased and dullness on percussion
and crackles was heard.

The diagnosis of this medical student :

a. Community acquired pneumonia

b. Acute exacerbation of asthma exacerbation

c. Empyema

d. Acute exacerbation of copd

e. Health care associated pneumonia


139. Mr. Sarpini, a 47-years old farmer complained cough for almost 2 months. Acid Fast
Bacilis was found in his sputum smear. Two years ago he had been treated for lung
tuberculosis and his doctorsdeclaired that he was cured.

The current anti tuberculosis regiment for Mr. Sarpiniis :

a. Streptomycin, INH, rifampicin, ethambutol

b. Rifampicin, INH, ethambutol, pyrazinamide, streptomycin

c. Rifampicin, INH, pyrazinamide

d. Rifampicin, streptomycin, ethambutol, pyrazinamide

e. INH, pyrazinamide, ethambutol, ciprofloxacin


B. P
140. Mr. Prakash Padukone is a 65-year old man, come to you with shortness of breath and a
cough with productive and thick sputum. On chest examination ypu found hypersonor 0
and wheeze. Spirometri after bronchodilator showed FEV1/FVC was 56% and FEV1 was 2
42% of predicted value. b
e
Your diagnosis will be : l
o
w
a. COPD stage II 6
5
b. COPD stage III m
m
c. Moderate persistent Asthma bronchial H
g
d. Severe persistent asthma bronchial o
r
e. Mild asthma bronchial S
a
t
141. Ms. Elsa is a 17 yo high school students came to you due to shortness of breath. She b
usually feels her SOB between midnight to early morning and she feels this about 3 times a e
week and sometimes accompanied with cough. Sometimes she was absent from school l
because of her symptoms. o
w
On chest examination you only heatd wheeze and others were within normal limits. The 8
most likely diagnosis is : 8
%
w
A. COPD stage I it
h
B. COPD stage II o
r
C. Mild persistent asthma bronchial w
it
D. Severe persistent asthma bronchialgelala malam <2x sebulan h
gejala malam daily
o
E. Moderate persistent asthma bronchial u
t
gejala malam > 1x seminggu h
y
p
142. A 67 yo complained cough with whitish sputum since about 2 years ago. His spirometer e
examination showed FEV1/FVC ratio was 65% and FEV1 is 82% fromm predicted value. Beside r
avoidance from ris factors and influenza vaccination, other medication for him is : c
a
A. Inhaled regular long acting β2 agonist p
n
B. Inhaled corticosteroid i
a
C. Rehabilitation C. P
a
D. Regular oral methlxanthym 0
2
E. Inhaled short acting β2 agonist is needed
b
e
l
143. A 72 yo, known for years as a COPD patient. Recently his pulmonologist recommend
o
him to add long term oxygen therapy in the treatment. w
6
The indications for long term oxygen therapy are :
0
A. Pa02 below 55 mmHg or Sat below 88% with or without hypercapnia m
mHg without evidence of pulmonary hypertension
D. Pa02 below 60 mmHg without evidence of polycythemia
E. Pa02 below 65 mmHg without evidence of pulmonary hypertension

144. Mr. X had been diagnosed as pulmonary TB patient since 4 month ago. And he received
rifampicin, ethambutol, INH and pyrazinamide. 2 days ago he complained that he had vision
disturbance. Which drug is the most likely cause?

A. IMA (mungkinmaksudnya INH)

B. Rifampicin

gejala malam = persisten


sedang

Orang hebat adalah orang yang Melakukan Hal biasa secara luar biasa
C. Ethambutol

D. Pyrazinamide

E. Combination of above drugs

145. Anoxic condition which is important in Forencic Medicine is : mayat kaku

A. Anemic hypoxia

B. Metabolic hystotoxic anoxia

C. Stagnant hypoxia

D. Hypoxic hypoxia
gak ada O2

E. Extra cellular hystotoxic hypoxia

146. Mechanical asphyxia s : sirkulasi


aliran ke jaringan terganggu

A. Stagnant hypoxia

B. Hystotoxic hypoxia

C. Anemic hypoxia

D. Substrate hystotoxic hypoxia

E. Hypoxic hypoxa

147. In cases of sea water drowning it is found :

A. Hemodilution

B. Na Plasm increase

C. Hemolysis

D. K Plasm increase air tawar

E. Ventricular fibrillation

148. In internal examination of drowning cases it is found :

A. Cutis Anserina

B. Cadaveric Spasm

C. Washerwoman hands : tenggelam udah lama, early sign of immersion, few hours in
cold water

D. Tandieu Spot : ptechie di pleura, epicardium, thymus

E. Pseudo Foam

 
149. The common cause of death in fresh water drowning is : cairan masuk ke paru2 infiltrasi

menekan jantung. Asphyxia + electrolyte problems vagal inhibition ?????????
A. Asphyxia

B. Ventricular defibrillaton

C. Laryngeal spasm

D. Vagal inhibition

E. Cardiac Arrest

150. A ..infant of 4 months old is found unexpectedly dead in her cot one morning. A …
examination is performed. The pathologist report increased thickness and extension of
pulmonary artery muscle. (First respon asfiksia, increase in number and size)

What does this finding indicate?

A. Cardiomyopathy is likely
B. Death was caused by asphyxiation

C. Death was due to epilepsy

D. The child would have developed hypertension in adult life

E. There was chronic hypoxia before death . i

151. On a frontal chest film, there is the lung apex retracts toward the hilum, the
sharp white line of visceral pleura is visible, separated from the chest wall by a
radiolucent pleural space, which is devo----lung markings:

A. Pneumothorax
B. Giant bullae

C. Pulmonary emphysema
D. Giant emphysema

E. Pleural effusion

152. A 20yo male, complained of persistent nasal discharge, this nasal discharge
was yellowish and thick. Past history, he had nasal itchy and nasal obstruction whe
the weather was cold. On waters film there is clouding in right maxillary sinus.

Which of the following is the most likely diagnosis?


A. Tumour of right maxillary sinus

B. Right hematiosinus
C. Acute sinusitis

D. Chronic sinusitis
E. Mucocele

153. On posteroanterior chest film, there is a cavity in apical parts of the left upper
lobe, the wall of the cavity is thick and is surrounded by patchy consolidation. WOTF is
best feature has been described?

A. Lung abcess

B. Pulmonary cyst

C. Mycetoma in a cavity
D. Cavitating tumor

E. The cavity of pulmonary tuberculosis

Question number 154-155


x-year-old girl admitted to pediatric clinic with a chief complaint of difficulty of
breathing since x-hours agp. This complaint was accompanied by cough and high fever.
In anteroposterior chest x-ray, there is homogenous lung opacification with air
bronchogram in the lateral segment of the lung.

154. The segmental homogenous lung opacification with air bronchogram


is: A. An atelectasis

B. A pleural effusion
C.A pneumonia D.A
schwarte

E. A bronchopneumonia

Orang hebat adalah orang yang Melakukan Hal biasa secara luar biasa
155. This disease begins as a localized infection of:
A. Terminal air spaces

B. Terminal bronchioles

C. Respiratory bronchioles

 
D. Acinus respiratory duct sac
E. Interstitial

Source : slide radiologi

156. A 10yo girl chief complaint dyspnea sice 7 days ago. This complaint was
accompanied by cough and high fever. In Anteroposterior chest xray there is
homogenous lung opacification with air bronchogram in the lateral segm]ent of te lung.
The lateral segment of the lung is located in?

A. Upper lobe of the right lung


B. Upper lobe of the left lung
C. Middle lobe of the right lung
D. Lower lobe of the left lung
E. Lower lobe of the right lung

For question number 157-158

A 25yo female comes with cough more than 3 weeks and accompanied by the
production of purulent sputum, night sweat, weight loss, anorexia, general malaise,
and weakness.

157. What is the basic standard radiograph for any patient presenting with a cough
more than 3 weeks?

A. A posteroanterior chest
film B. An oblique chest film

C. A left lateral decubitus


film D. An apical lordotic film

E. An anteroposterior chest film

158. What is the best radiograph for showing the presence of a small pleural effusion?
A. A lateral edcubitus film
B. A posteroanterior chest
film C. A lateral chest film

D. Aanteroposterior chest
film E. An apical lordotic film

159. In the posteroanterior chest Xray,there is only calcification in the apex of the lung. It
is no clear if there are patchy consolidation in the apex of both lungs,because the clavicle
and the ribs overlaps with the apex of both lungs. What is the best radiograph for showing
the precence of minimal patchy consolidation in the apex of the lung?

A. An oblique chest film


B. A left lateral decubitus

C.An apical lordotic film

D. An anteroposterior chest film

E. A lateral chest film

160. Mrs. X 28th 10 day history of nasal discharge. Discharge is yellow-greenish, thick, blood
tinged, also accompanied with decrease of smelling sensation,nasalblockage,fever and
headache. Three days ago she felt pain and fullness on the left ear with mild increase of
hearing. PE result discharge from the nose and bulging of tympanic membrane. WOTF is the
best pathogenesis of discharge in this patient?

A. The ostium obstruction causes the vasodilatation and cilliary and mucus
glands dysfunction

B. Enlargement of concha part in the nose

C. Eustachian tube disfunction caused by secondary infection

D. Viral infection cause vasoconstriction and destruct the mucous gland directly

E. The upper respiratory tract infection resulting in Eustachian tube dysfunction


and improved bacterial adherence to the URT mucosa

161. Which of the following is the best pathogenesis of bulging tympanic membrane in this
patient?

a. The ostium obstruction causes the vasodilatation and ciliary and mucous
gland dysfunction

b. Enlargement of concha part in the nose

c. Altered regulation of middle ear pressure and formation of negative pressure in


middle ear
d. Viral infection causes vasoconstriction and destruct the mucous gland directly

e. The upper respiratory tract infection resulting in Eustachian tube dysfunction


and improved bacterial adherence to the upper respiratory tract mucosa

162. A 9 year old boy come to your private practice with breathlessness as a chief complaint.
The symptom was accompanied by wheezing since last 1 hour. These symptoms had
occured about two hours after helping his mother cleaned the bedroom. He had his first
asthma attack when he was six years within the last two months he also experienced 1-2
times night cough.
His father had been diagnoed as having asthma and the symptoms had relieved since he
had taken controller medication. Which of the following is the best pathogenesis in this
patient? a. TH2 cells secrete cytokines that promote allergic inflammation and stimulate B
cells to produce Ige and other antibodies

b. Virus induced inflammation of the respiratory mucosa lowers the threshold of the
subepithelial vagal receptors to irritants

c. Inhibiting the cyclooxigenase pathway of arachidonic acid metabolism without


affecting the lipooxygenase route, thus lipping the balance toward elaboration of the
bronchoconstorleukotrienes

d. A marked increase in goblet cells of small airways—small bronchi and bronchioles—


leading to excessive mucus production that contributes to airway obstruction

e. The destuctive effect of high protease activity in subjects with low antiprotease activity
and also the oxidant-antioxidant imbalance

For questions number 163-163 refer to scenario below

Orang hebat adalah orang yang Melakukan Hal biasa secara luar biasa
A 65 year old man was brought to the Emergency department of HasanSadikin Hospital with
the chief complaint of dyspnea which was worsening since two days before.

Since three days before the patient had flu that made his dyspnea and cough worsened
with thick and greenish sputum, fever, and also by noisy breathing. PE : Chest : barrel
shaped chest, lung hypersonor and extrimities : Clubbing fingers was noted; neither edema
nor cyanosis was found

163. Which of the following is the best pathogenesis in this patient?

a. TH2 cells secrete cytokines that promote allergic inflammation and stimulate B cells
to produce Ige and other antibodies

b. Virus induced inflammation of the respiratory mucosa lowers the threshold of the
subepithelial vagal receptors to irritants

c. Inhibiting the cyclooxigenase pathway of arachidonic acid metabolism without


affecting the lipooxygenase route, thus lipping the balance toward elaboration of the
bronchoconstorleukotrienes

d. A marked increase in goblet cells of small airways—small bronchi and bronchioles—


leading to excessive mucus production that contributes to airway obstruction

e. The destuctive effect of high protease activity in subjects with low antiprotease activity
and also the oxidant-antioxidant imbalance

164. Which of the following is the best for microscopic finding in that patient?
a. Large alveoli separated by thin septa with only focal centraacinal fibrosis

b. Chronic inflammation of the airways (predominantly lymphocytes) and enlargemenr


of the mucus-secreting glands of the trachea and bronchi

c. Occlusion of bronchi and bronchioles are sufficiently dilated that they can be
followed almost to the pleural surface

d. The bronchi and bronchioles are sufficiently dilated that they


can e. Sub-basement membrane fibrosis

165. Which of the following is the best pathogenesis in this patient? TB

a. TH2 cells secrete cytokines that promote allergic inflammation and stimulate B cells
to produce Ige and other antibodies

b. Virus induced inflammation of the respiratory mucosa lowers the threshold of


the subepithelial vagal receptors to irritants

c. Role of blockage of phagolysosom, interferon gamma, and delayed hypersensitivity


d. A marked increase in goblet cells of small airways—small bronchi and bronchioles—
leading to excessive mucus production that contributes to airway obstruction

e. The destructive effect of high protease activity in subjects with low antiprotease
activity and also the oxidant-antioxidant imbalance
166. Which of the following is the best of microscopic finding in this
patient? a. Diffuse infiltration of polymono nuclear cell and bloody tissue
b. Congestion of cells with exudat and caseous necrosis

c. Granulomatous inflammation contain of datialanghans and central necrosis


d. Diffuse infiltration of mononuclear cell and epitheloid cell

e. Consolidation and hepatization of parenchymal tissue of the lung

167. A-10 month-old-boy admitted to Pediatric Emergency Roon with difficulty of


breathing and suffers from fever since the last 2 days as his chief complaint. This complaint
had been preceded by common cold since 4 days ago. Laboratory results :
Hb 11.4 g/dl , Hematocrit 37%, WBC 21.700/mm3 , Thrombocyte 210.000/mm3
Differential count -/2/4/68/25/1.
Blood smear shows toxic granule in neutrophil
Chest X-Ray showed bilateral infiltrate
Which of the following is the best pathogenesis for this patient.

a. Consolidation of acute fibrinosupurrative inflammation in patchy infiltrate.


bronchopneumonia

b. TH2 cells secrete cytokines that promote allergic inflammation and stimulate B cells
to produce Ige and other antibodies

c. Virus induced inflammation of the respiratory mucosa lowers the threshold of


the subepithelial vagal receptors to irritants

d. Role of blockage of phagolysosom, interferon gamma, and delayed hypersensitivity


e. A marked increase in goblet cells of small airways—small bronchi and bronchioles—
leading to excessive mucus production that contributes to airway obstruction

For questions number 168-169, refer to scenario below :

A year old male came to the outpatient clinic HasanSadikin General Hospital because of
chronic nasal congestion. Rhinoscopy revealed gelatinous grayish-white tissue with
smooth and shiny surface fill in the right and left nasal cavity. The biopsy was performed
and the microscopic appereance showed the picture below.
168. What is the most appropriate terminology of the nasal mass in this patient?
a. Chronic rhinitis


b. Nasal polyp karena chronic rinitis c.
Inverted papilloma

d. Angiofibroma

e. Nasopharyngeal carcinoma

169. According to the pathogenesis of the disease, which inflammatory cells are most
commonly seen microscopically in above nasal mass?

a. Lymphocytes
b. Plasma cells
c. neutophils d.
Basophils e.
Eosinophils

For questions number 170-171

A 23 year old male came to the hospital because of epistaxis since 2 days ago. He also
complained nasal thickened since 1 month ago. Rhinoscopy and nasopharyngoscopy

Orang hebat adalah orang yang Melakukan Hal biasa secara luar biasa
revealed a reddish ulcerative mass in the choanae and nasopharynx posterior. Biopsy of
the nasal and nasophaynx was performed, macroscopic, and microscopic appereance of
the mass was shown as below picture.

170. what is the appropriate terminology describing the disease in the nasal and
nasopharynx of above patient?

a. chronic
inflammation b. palyp

c. papilloma

d. angiofibroma

e. carcinoma (heredity, age, EBV)

171. which of the following agent that is closelu related to the pathogenesis of above
disease?

a. polen
b. Hause mites

c. Epstein Barr virus

d. Human papilloma virus

e. Human immunodeficiency virus


172. what is the disease that occurs in above
patient? a. pneumonia

b. Pulmonary TB

c. Emphysema

d. Lung carcinoma

e. COPD

173. What structures are pointed by the white arrows in above


case? a. Extravasated erythrocytes

b. Alveoli destruction
c. DatiaLanghans cells
d. Caseous necrosis e.
Tumour cells

174-175

A 45yo male come to out patient clinic because of dyspne, fever and productive cough since
7 days ago. He said that his sputum was yellow green sometimes with blood streak. PE showed
fever, takipnea, tachycardia, and crackles in his left chest. Macroscopic and microscopic
appearance of his lung were shown as below picture.

Orang hebat adalah orang yang Melakukan Hal biasa secara luar biasa
Tim Akademik ANANTARA 2015
Divisi SL (Soal Latihan)
Fakultas Kedokteran Universitas Padjadjaran

174. What is the disease that occurs in above


patient? a. pneumonia

b. Pulmonary TB
c. Emphysema d.
Lung carcinoma e.
COPD

175. What structures are pointed by the white arrows in above


case? a. Extravasated erythrocytes

b. Alveoli destruction
c. DatiaLanghans cells
d. Caseous necrosis e.
Tumour cells

MDE RESPIRATORY SYSTEM BATCH 2008


1. Comparing to the oxygen partial pressure in alveoli. Which one is true about pulmonary veins oxygen partial
pressure
a. Lower because it contains blood from right chamber of heart
b. The same because its transport is simple diffusion
c. higher because its transport by active transport
d. higher because its large and thin surface
e.higher because its transport is so fast

2. Faktor yang meningkatkan afinitas oksigen terhadap Hb? (Rendy S.) shunt to the left
e. blood O2 level increased

3. Which of the following mediates predominantly the respiratory regulation of central nervous system
mechanism?
a. The level of blood ph
b. The level of O2
c. The level of blood CO2
d. The level of blood 2,3 bpg
e. The body temperature

4. Can lung act in regulating acid base balance?


a. No, because the main function of lung is to exchange gases
b. Yes, because the lung has the ability to excrete and preserve H +
c. yes, because the lung has the ability to excrete and preserve H2O
d. Yes, because the lung has the ability to excrete n preserve CO2
e. yes, because the lung has the ability to prevent us from inhaling acid

5. What is approximate amount of inspiratory reserve volume in normal man at rest?


a. 2200 ml d. 500 ml
b. 1500 ml e. 1200 ml
c. 3100 ml
Tim Akademik ANANTARA 2015
Divisi SL (Soal Latihan)
Fakultas Kedokteran Universitas Padjadjaran

6. Wotf is responsible for the movement from the alveoli into the blood in the pulmo capiller?
a. active transport
b. filtration
c. second active transport
d. facilitated diffusion
e. passive transport

7. True "airway resistance"


a. > jika paru2 diremove, inflate ketika dikasih saline
b. > asma
c. > paraplegik pasien
d. Not effect on work of breath
e. Berkontribusi pada 80% work of breath

8. Which of the following relax the bronchial smooth muscle?


a. Leukotriene d. cool air
b. Vip e. Sulfur dioxide
c. Acetylcholine

9. The most form of CO2 transported in blood?


a. Dissolve in plasma
b. In corbamino compound formed from plasma protein
c. In carbamino compound formed from hb
d. bound to Cl
e. HCO3-
10. Greatest effect on ability to transport O2 is
a. Capacity blood to dissolve O2
b. Jumlah hb di darah
c. Ph plasma
d. CO2 content of RBC
e. Temperature blood

76. At inspiratory, event yang terjadi ?


31. Intraabdominal pressure turun
32. Venous return increase
33. HR turun
34. Intrapleural pressure turun
35. Muscle abdominalis contracts

77. Air in the lung subdivided into


A. 3 vol 4 capacities
B. 5 vol 5 cap
C. 4 vol 4 cap
D. 4 vol 5 cap
E. 5 vol 6 cap

78. IV lactic acid dapat meningkatkan pernafasan akibat rangsangan ke reseptor di..?
h. Medulla oblongata
i. Carotid body
Tim Akademik ANANTARA 2015
Divisi SL (Soal Latihan)
Fakultas Kedokteran Universitas Padjadjaran

j. Lung parenchyme
k. Aortic
l. Trachea dan large bronchi

79. In which the following conditions is CO2 retention most likely occur ?
12. Climbing high mountain
13. Ventilatory failure
14. CO
15. Lung failure
16. Hysterical hyperventilation

80. Child 2 y.o.datang ke THT clini dgn kelujan hearing loss sejak 2 minggu yang lalu. Tanpa otalgia, otorrhea
ataupun febrile. Otosopy menunjukkan right ear normal, left ear, tympanic membran intact namun
terlihat adanya “buble sign” di tympanic cavity.
70. External otitis
71. Acute otitis media
72. Chronic otitis media
73. Serous otitis media
74. Malignant external otitis

81. Pasien dengan buble sign yang sudah ditreatment dengan obat tp tidak berhasil. Apa yg dilakukan
kemudian,,
h. Myringotomy
i. Myringoplasty
j. Mastoidectomy
k. Tympanoplasty
l. Hearing aid

17. in the past two weeks, an 8 y.o boy suffered from blockage of the nose with purulent rhinorrhea, facial pain,
and sometimes epistaxis. from rhinoscopy anterior there was hyperemic nasal mucosa, with mucopurulent
discharge. no mases or anatomical pathologies were found. blood clot was found in anterior nasal septum.
oropharyngeal exam reveal post nasal drip.
medical management for this condition is:
a. not frequently effective
b. directed toward more resistant bacteria
c. usually targeted toward specific bacteria and broad spectrum
d. universally effective
e. best provided with IV therapy

g. What muscle actively open the eustachian tube ?


4 M. Tensor veli palatine
5 M. Levator veli palatine
6 M. Salphyngopheryngeus
7 M.lateral pterygoid
8 M. Medial pterygoid

h. Drainage nasolacrimal duct..


46. Common meatus
Tim Akademik ANANTARA 2015
Divisi SL (Soal Latihan)
Fakultas Kedokteran Universitas Padjadjaran

47. Superior meatus


48. Middle meatus
49. Inferior meatus

i. Kiesselbach plexuse, little area tersusun dari superior labial artery dan..
g. Pharyngeal artery
h. Anterior ethmoid
i. Posterior ethmoid
j. Descending palatine
k. Sphenopalatine

21. 21 y.o male pain and difficulty in swallowing since 2 days ago. Pain mnyebar ke left ear,can only eat liquid.
Kesulitan dalam mmbuka mulut,enlarged le ft tnsil,deviated uvula to right. Diagnosis?
A. Ludwig's angina
b. Parapharyngeal abcess
c. Tonsillar hypertrophicans
d. Peritonsillar abcess
e. Retropharyngeal abcess

22. the strap muscles of neck are enveloped in:


a. deep layer of deep cervical fascia
b. middle layer of deep cervical fascia
c. superficial layer of deep cervical fascia
d. superficial cervical fascia
e. carotid sheath

23. The lateral pharyngeal space divided into 2 compartment by?


A. Cranial nerve 2
B. Carotid artery
C. Deep larynx of deep cervical fascia
D. Styloid process
E. Jugular artery

24. The most common place of infection of lateral pharyngeal space is from the :
A. Soft palate d. Neck
B. Mastoid e. Floor of the mouth
C. Tonsil

25. 12 yrs old male has dirty gray membrane on tonsils pillars and uvula. Removal attempts caused bleeding.
There is positive marked clinically lymphadenopathy. The fluorescent study is positive. What is the
diagnosis?
A. Vincent angina
B.
C.
D. Diphteri
E. Infectious mononucleosis

26. 5 years old, pneumonia, alergi penicillin & cephalosporin. Obat yg cocok?
A. Azithromycin
B. Amoxicillin + clavulanate, krn tahan thdp beta-lactamase
C. Cefadroxil
Tim Akademik ANANTARA 2015
Divisi SL (Soal Latihan)
Fakultas Kedokteran Universitas Padjadjaran

D. Erythromycin, krn efektif melawan gram positif dan immunologic characteristic berbeda dgn beta-lactam
E.
(duh maaf ya yg ini ga lngkap, ga bs nyatet duduknya depan dosen >.< )

For question 27-28:


A male patient is given antibiotic on enteric coated form. Because of his difficulty at swallowing drugs, it is
divided into small pieces. He had a good compliance in taking the drugs. A few days after, there is no improve
on clinical features. He said that he got additional symptoms including nausea & diarrhea.

27. What is the most possible cause of treatment failure in this case?
A. Microorganism is resistant to antibiotic consumed
B. The patient also have allergy agains this antibiotic
C. The antibiotic was inactivated cause by drug
D. The antibiotic was innapropriate against the organism
E. The antibiotic's absorption is interfered by gastric acid

28. The most appropriate management for the abdominal symptoms?


A. Substitute the antibiotic used
B. Add antiemetic and anti diarrhea
C. Continue the antibiotic used but do not crush the drugs
D. Stop the antibiotic use and consider additio of antibiotic for GIT
E. Take the antibiotic after meal

Soal no. 29-30


Seorang ibu baru melahirkan bayi, dia telah mengkonsumsi OAT sejak lama tapi kemudian dihentikan 1 bln
yang lalu.

29. Appropriate planning for mother?


A. Stop breastfeeding; lanjutkan pengobatan sampai 6 bln
B. Continues breastfeeding; beri pengobatan baru ( 2HREZ/HREZ/5HRE)
C. Stop breastfeeding; beri pengobatan baru
D. Continues breastfeeding; beri pengobatan 5HRE
E. Stop breastfeeding; beri pengobatan 5HRE

30. Appropriate planning for baby?


A. Beri full TB treatment
B. Beri INH untuk pencegahan, dan jauhkan dari ibu sampai 2mgg pengobatan
C. Vaksin BCG
D. Vaksin BCG dan pencegahan (INH) after TB role out

31. Gilang

32. A 56 yo man was diagnosed of End State Renal Disease since 2 years ago. He had to have blood
hemodialysis once a week. Because of his 2 times positive sputum smear of MTB, his doctor said that he
have to get R, H, Z, E, S. Fortunately, his liver function is normal. If the doctor does not adjust the regimen
dosage for him because of his past clinical history, which one is the most possible symptom that could
happen?
a. Nephropathy because of streptomycin
b. Hyperuricemia because of isoniazid
c. Peripheral neuropathy because of pyrazinamide accumulation
d. Retrobulbar neuritis because of ethambutol
e. Hepatotoxicity because of Rifampicin
Tim Akademik ANANTARA 2015
Divisi SL (Soal Latihan)
Fakultas Kedokteran Universitas Padjadjaran

33. 45 yo man administrated ART, recently diagnosed TB


a. 2HRZE/4HR, STOP ART
B. 2HRZE/4H3R3, STOP ART
C. 2HREZES/HRZE/4H3R3, CONTINUE ART
D. 2HRZE/4HR, CONTINIE ART
E. 2HRZE/4H3R3,CONTINUE ART

Gambar2 yang gangguan development trachea-esophagus.

34. Bella
35. Which of the following structure is match with the statement “the air can’t pass the gastric and
esophagus”?
36. H-type tracheoesophageal itu yg mana? (pilihannya berupa gambar)
37. isolated esophageal atresia yang mana?

38. Ecky

39. components of the blood air barrier in the lung are derived from wotf?
a. ectoderm only
b. visceral mesoderm only
c. visceral mesokderm and ectoderm
d. ectoderm adn endoderm
e. visceral mesoderm and endoderm

i. The laryngotracheal tube initially is in open communication with the primitive foregut. WOTF structures is
responsible for separating this two structure?
A. Laryngotracheal groove
B. Posterior esophageal folds
C. Laryngotracheal diverticulum
D. Tracheosophageal septum
E. Bronchopulmonary segment

j. Collapse of bronchial caused by failure of bronchial cartilage development are indicator of WOTF
congenital malformation:
A. Congenital bronchial cyst
B. Congenital neonatal emphysema
C. Tracheoesophageal fistula
D. HMD
E. Pulmo hipoplasia

k. 27yo,man,smoking,sudden onset chill fever, temp 28,5'C, cough, left-sided pleurutic chest pain. xray=
focal consolidation in left lower lobe
organisme likely =
a. s.pneumonia
b. h.influenza
c. m.catarrhis
d. m.pneumonia
e. s.aureus
Tim Akademik ANANTARA 2015
Divisi SL (Soal Latihan)
Fakultas Kedokteran Universitas Padjadjaran

l. Which of the following statement is correct about wall layers of the larynx?
6 It has elastic and hyaline cartilage
7 Lined by simple columnar epithelium
8 Has sebaceous gland
9 Not a component of skeletal muscle
10 Has sweat gland

m. WOTF differentiate streptococcus and staphylococcus:


A. Gram reactivation
B. Cell diameter
C. Cell morphology
D. Catalase
E. Acid fast staining

n. Staining untuk C. Diphteri:


A. Gram stain d. Giemsa
B. Ziehl Nielsen e. Neisser
C. Burri gins

o. Intracellular bacterial parasite:


A. S. pyogenes
B. Klebsiella pneumoniae
C. Clostridium perfringens
D. M. Tb
E. S. Pneumoniae

p. Which is true about Legionella pneumoniae:


A. Gram positive rod
B. Intracellular paracite bacteria
C. It produce no toxin
D. It is associated with room air conditioner
E. It can infect tissue human bodies

q. Mycobacterium yang dapat menyebabkan tuberculosis:


Ans: M. Bovis

r. WOTF most likely cultivation for M. Tb:


A. Mac conkey agar
B. Loeffler agar
C. Lowenstein jensen agar
D. Chocholate agar
E. BCYE agar

s. What are the organism that have a heavy encapsulated capsule and frequently cause pneumonia in
debilitated patient?
A. S. pneumoniae
B. Klebsiella pneumoniae
C. M. Tb
Tim Akademik ANANTARA 2015
Divisi SL (Soal Latihan)
Fakultas Kedokteran Universitas Padjadjaran

D. S. aureus
E. Legionella penumoniae

55. in diagnosis of pertusis, bordetella is most likely isolated from?


Blood culture during bacteriemic phase of disease
Joint fluid
Nasopharyngeal swab cultured on bordet-bengou medium
Abscess on skin
Spinal fluid

56. s.pneumonia can escape phagocytic clearance by which mechanism?


Capsulated-mediated inhibition of phagocytosis
Inhibition of phagosome-lysosome fusion
Inhibition of opsonization mediated by protein A
Lysis of phagosome & replication in cytoplasm
Replication in fused phagosome-lysosome

57. handry

58. use to differ streptococcus pneumonia & s.viridans?


bacitracin d. Coagulation
optochin e. Oxidase
catalase

59. which of following situation is caused by increase of pCO2 in arterial blood that causes chemoreceptor
stimulate the respiratory center?
Decrease RR d. hypercapnia
Increase RR e. Hypoxia
Hypocapnia

60. 52 yo, smoker male comes to rshs having diff breathing. pO2 45mmhg. Then his pCO2 become 37 mmhg &
ph 7.4. WOTF most likely condition?
Doesn’t have respi problem
Respiratory acidosis
Hypoxia, unlikely acidosis
Respiratory alkalosis
Hypoxia, alkalosis

61. 17 yo,female, hyperventilating (freq,deep,sighing respiration to sustained,obvious,rapid,deep breathing)


fell unconscious. An attending physician diagnosed her condition as ‘hyperventilation syndrome’ caused by
anxiety & mental stress. He successfully treated her by rebreathing expired CO2 from paper bag & she
completely recovered. What happened to the acid-base balance in her body?
Increase rate of respiration due to hyperventilation cause the lung to remove more O2 from the blood
than normal & will decrease pO2 & increase the blood ph
Plasma O2 is considered an acid & HCO3 its conjugated base
Decrease of acid relative to the level of its conjugated base causes decrease in ph
Respiratory alkalosis
The process of acid-base balance is acidosis

62. 46 yo, male, high urea & creatinin. Acid-base disorder?


Respi acidosis due to CO2 retention
Acidosis cause overproduction acid
Metabolic acidosis cause impaired excretion acid
Tim Akademik ANANTARA 2015
Divisi SL (Soal Latihan)
Fakultas Kedokteran Universitas Padjadjaran

Respiratory alkalosis due to CO2 retention


Metabolic alkalosis due to overproduction base

63. pilihannya : (Puti)


shallow breathing
Kusmaul’s breathing
Deep breathing
Slow breathing

64. 55 yo,man, hard to breath,smoke or 30 years 16 pack/day, rr 30. BGA?


Ph decrease
Sat o2 increase
Ph increase
Po2 increase
Pco2 increase

61. 17 yo girl experienced deep frequent sighing respiration to sustained obvious, rapid, deep breathing, feel
unconscious on floor. Attending physician diagnosed her send caused by her anxiety & mental stress. He
successfully treated her with rebreathing expired CO2 from paper bag in improving her condition by
neutralized the blood pH?
a. Accumulation of CO2 decreased blood pH (Respiratory Acidosis)
b. Accumulation of O2, normalized pO2, decrease blood pH
c. Accumulation HCO3-, normalized pHCO3-, increased pH
d. Accumulation H+, normalized pHCO3-, increased pH
e. Accumulation OH-, normalized pOH-, increased pH

62. Prolonged alveolar hyperventilation can cause?


a. Metabolic acidosis
b. Metabolic alkalosis
c. Respiratory acidosis
d. Respiratory alkalosis
e. No change in blood pH

63. Why is gas exchange at respiratory membrane are efficient?


a. The difference in partial pressure are substansial
b. The gas are lipid soluble
c. The total surface area is large
d. The gas are protein soluble
e. The gas are lipid & protein soluble

64. For any partial pressure of O2 which of the following statement are correlated with condition of 2,3 BPG
increase?
a. The amount of O2 release from Hb decrease
b. The amount of O2 release from Hb increase (shunt to the right)
c. The amount of CO2 release from Hb increase
d. The amount of CO2 carried by Hb increase
e. The amount of O2 and Hb unaffected

65. Mutiara Retno Anjani (lagi sakit, lupa nyari back up nya..maaf ( )
Tim Akademik ANANTARA 2015
Divisi SL (Soal Latihan)
Fakultas Kedokteran Universitas Padjadjaran

66. Throat specimen from 3 y.o. boy with pharyngitis. Throat swabs are mostly used for :
A. Group A Streptococcus
B. N. diphthteria
C. Coli
D. Staphylococcus aureus
E. Pseudomonas aeruginosa

67. Suryo Indah W. (soalnya ketinggalan di jantinangor, nanti nyusul diupload)

68. Eosinofil naik di kasus?


a. Tb d. Malaria
b. Parasit e. Amobeasis
c. Hepatitis

69. An adult woman patient always complains about had difficulty of breathing. The doctor says that she has
asthmatic bronchitis and need to perform laboratory test to evaluate ventilation status of the lung by
blood gas analysis. In her condition of ventilation disturbance cause of respiratory acidosis. The result of
BGA are:
a. Increase pH, increase pCO2, increase HCO3-
b. Decrease pH, decrease pCO2, decrease HCO3-
c. Decrease pH, increase pCO2, HCO3- in normal level
d. Increase pH, decrease pCO2, decrease HCO3-
e. Decrease pH, decrease pCO2, increase HCO3-

70. Characteristic of exudates?


a. Total protein < 3 g/dl
b. WBC count > 1000/mm3
c. Specific gravity < 1,016
d. Cholesterol < 60 mg/dl
e. Rivalta test (+)

71. Diff count OMA dengan?


a. thin blood film/smear
b. bone marrow film
c. thick blood film
d. gram/ziehl neelsen
e. papanicolau

72. 32 yo man, chief complaint: dyspnea, fever, never experience before, no family history. BP 130/80 mmHg,
HR 90X, RR 36X, temp 390. Pneumococcus, hypersensitivity B-lactam. He got antibiotic but few hour
after feel abdominal discomfort. What is the antibiotic? Beri Macrolide !
a. azythromycin
b. cefalodroxyl
c. clarithromycin
d. clindamycin
e. erythromycin

73. 10-15 µm, nucleus bilobed, granules coarse and orange red cytoplasm abundant and pale pink tan. What
cell?
a. basophil
b. eosinophil
c. Neutrophil
Tim Akademik ANANTARA 2015
Divisi SL (Soal Latihan)
Fakultas Kedokteran Universitas Padjadjaran

d. Lymphocyte
e. Monocyte
74. Khairunisa

75. Jumlah eosinofil akan meningkat pada kondisi? alergi, infeksi parasit(Ligai)

76. Factor that interfere BGA in heavy smoker?


a. false hi pCO2 because hi pCO2 valu
b. false low pH level because of low pH level
c. false low O2 because of low O2 value
d. false hi O2 saturated
e. false hi base excess

77. Male. Complain of tachypnea, slight fever. Wheezing and decrease body weight despite of the regular
eating habit. Past history: ever taken TB medication but stopped after he felt better. What’s the medication
he should take?
a. category I
b. category II
c. category III
d. don’t give medication until the culture result come out

78. Seorang pasien 20 yo datang ke klinik dengan keluhan utama chronic cough and sputum. Diikuti dengan ada
chest wall pain, low grade fever, night sweating. Makan antibiotik tapi tidak ada perubahan. Two specimen
AFB +. Setahun yang lalu didiagnosis TB dengan pengobatan berhenti setelah 2 bulan, dan merasa baik.
Dari chest x-ray, ada koloni upper lobe. Tipe kasus?
a. new case
b. relapse
c. treatment failure
d. after default

79. 64 yo, complaint dyspnea especially when walked in rushed for several hundred meters and cough with
whitish sputum for the last 2 years. He is current smoker, smoked for 36 packs/year clove cigarrette (kretek)
since he was 20 yo.
He comes form low socio-economic. His father has allergy. Which one represent the chronic inflammation?
a. Lymphocyte CD4+ dominant
b. eosinophil dominant
c. neutrophil dominant
d. reversible airflow limitation
e. Excellent response to glucocorticoid

80. 65 years old man, increasing shortess of breath. cough and sputum production for the last 5 years. Smoke
since age 15. chest x-ray: hyperinflation, no sign of infiltrate found. spirometry: FEV1/FVC 60%. FEV1: 45%.
Predicted ECG normal. If this patient has pneumothorax, which of the following will be found:
If he has spontaneous pneumothorax, which is the following is the diagnosis?
a. Primary pneumothorax
b. secondary pneumothorax (akibat COPD)
c. tension pneumothorax
d. iatrogenic pneumothorax

81. A 65-years old man with COPD stage III and women23-years old with asthma bronchial moderate
persistent. Both were taken inhalation corticosteroid. While women gains improvement from this therapy,
the man is not. What is possible explanation for this?
g. Difference gender between them
h. Different in age
i. Different in spirometry result
j. Different in risk factor
Tim Akademik ANANTARA 2015
Divisi SL (Soal Latihan)
Fakultas Kedokteran Universitas Padjadjaran

k. Different in inflammatory cell & mediator

82. 22-years old woman, intermittent wheezing in response to exercise presents to emergency room with
shortness of breath. Her attack occurred during aerobic class. She is obvious difficulty breathing & diffuse
wheezes on pulmonary exam. 02 saturation is 95% by pulse oxymetry.
WOTF condition occurs as results of air trapping?
2 Airway obstruction during inhalation & subsequent alveolar collapse
3 Increase lung compliance because inflammation
4 Collapse of congested bronchioles during exhalation
5 Shunting of ventilation of the right lung fields
6 Increase mismatch ventilation

83. 65-years old, dyspnea, spirometry exam FEV/FVC 57%, FEV 30% of the predictive value after
bronchodilator treatment. Which PE may be appear?
76. Pleural friction rub
77. Bronchial breath sound
78. Broncophony
79. Narrow intercostals space
80. Hypersonor in percussion

84. 20-y.o woman, ER, dyspnea, start 1,5 jam because she nervous in the gym class. Suddenly hard breath
with wheezing breath. WOTF exam statement?
36. Bronchial provocation test are essential for diagnosis
37. FEV1 ↑ after bronchodilation
38. TLC is decreased
39. Residual volume maybe increased
40. Skin prick test are usually negative in extrinsic asthma

85. Not influence clinical feature TB?


38. Tropism of specific tissue
39. Malnutrition
40. Immunization BCG
41. Coexisting another disease
42. Genetic

86. 62-y.o, shortness of breathing, cough, increase of sputum for 5 years. Chest X-ray: hyperinflation, no
infiltration. FEV1/FVC 60% & FEV1 45%. ECG normal.
Type of pneumothorax if suspected as DD?
3 Primary pneumothorax
4 Secondary pneumothorax
5 Endogenous pneumothorax
6 Tension pneumothorax
7 Iatrogenic pneumothorax

87. 17-y.o. AFB+, treated with category 1, after 9 days he developed jaundice & vomit. Which drugs cause
that condition?
i. R, H, Z (Rimfam, Isoniazid, Pyrazinamide)
j. R, H, E
k. Z, E, S
l. S, R, E
m. H, S, Z
Tim Akademik ANANTARA 2015
Divisi SL (Soal Latihan)
Fakultas Kedokteran Universitas Padjadjaran

88. 30-y.o man came to clinic with chief complaint cough & sputum > 6 weeks. Complaints accompanied by
decrease body weight in spite of regular eating habits, low grade fever, and night sweating. Which of the
following factor that has no role in management in this patient?
61. Species & virulence of mycobacterium
62. Immunity of host to mycobacterium
63. Adherence of patient to regiment ATD
64. Interaction between host & drug
65. Prevalence of organ involvement

89. What is the most common cause of massive hemoptysis in Indonesia?


25. Bronchiectasis
26. Lung cancer
27. Tuberculosis
28. Necrotizing pneumonia
29. Lung abscess

90. A 70-y.o man admitted with chief complaint of shortness of breath, yellowish sputum. History of DM. PE
shows RR 30x/min, T: 380C, BP: 130/80, rhonchi +/-, wheezing +, blood glucose 276 mg/dL, chest x-ray:
infiltration. No history hospitalization.
If he come with respiratory failure, what is gold standard for diagnosis?
g. Chest x-ray
h. Spirometry
i. BGA
j. Pulse oxymetry
k. Pulse angiogram

91. Diberi Amoxicilyn 500 mg diminum 3x sehari untuk 7 hari setelah makan, tulis resepnya.
c. R/ Amoxicillin tab 500 mg No. XXI S.3.d.d tab I a.c
d. R/ Amoxicillin tab 500 mg No. XXI S.3.d.d tab I p.c

92. Obstructive lung disease


a. Lower energy consumption & hypometabolic state
b. Lower energy consumption & hypermetabolic state
c. Normal energy consumption
d. Higher energy consumption & hypometabolic state
e. Higher energy consumption & hypermetabolic state

93. A 35 y.o is a heavy smoker. He came to your clinic with cough & dyspnea. His BP: 120/80 mmHg, Pulse rate:
120x/m, Respiration rate: 30x/m, and temperature: 38,2o C. Chest examination: from inspection was found
barrel shaped chest. From blood gas analysis:pH: 7.28, PaCO2: 50, PCO2: 60, Sat O2: 90%. Which of the
following statement is the most appropriate due to substrate metabolism use?
a. Respiratory Quotient (RQ) is the ratio of the volume of oxygen expired to the volume of carbon dioxide
inspired
b. Respiratory Quotient (RQ) is the ratio of the volume of oxygen inspired to the volume of carbon dioxide
expired
c. Respiratory Quotient (RQ) is the ratio of the volume of carbon dioxide inspired to the volume of oxygen
expired
d. Respiratory Quotient (RQ) is the ratio of the volume of carbon dioxide expired to the volume of
oxygen inspired
e. Respiratory Quotient (RQ) is the ratio of the volume of carbon dioxide inspired to the volume of carbon
dioxide inspired
Tim Akademik ANANTARA 2015
Divisi SL (Soal Latihan)
Fakultas Kedokteran Universitas Padjadjaran

94. A 50 yo man complains dyspnea, especially when he walks in rush. His complains are accompanied with
cough and whitish sputum for the last five years. He is a heavy smoker, with a history of smoking about 30
pack-year cigarettes since he was 17 year old. During physical examination, it is found that the
cardiovascular is within normal limits. His body weight is 55 kg and his height is 175 cm.
Which of the following is the most likely condition?
32. He needs more energy due to smoking
33. He needs more energy due to the increased work of breathing
34. He needs more energy due to preserving fat mass
35. He needs more energy due to preserving visceral fat mass
36. He needs more energy due to preserving subcutaneous fat mass

95. A 35 yo came to your clinic complaining cough and dyspnea. He is known as a heavy smoker. During
physical examination it is found that his BP: 120/80 mmHg, pulse rate: 120x/m, respiration rate: 30 x/m,
and temperature: 38.2o C. Chest examination: from inspection barrel shaped chest was found. Blood gas
analysis result: pH: 7.28, PaCO2 : 50, PCO2: 60, Sat O2: 90%.
Which of the following energy intake is the most appropriate?
20. Increasing energy needs with 65% calories from carbohydrate and 20% calories from fat
21. Increasing energy needs with 60% calories from carbohydrate and 25% calories from fat
22. Increasing energy needs with 50% calories from carbohydrate and 35% calories from fat
23. Increasing energy needs with 65% calories from carbohydrate and 35% calories from fat
24. Increasing energy needs with 70% calories from carbohydrate and 30% calories from fat

96. Which parasite is most apt to be diagnosed by examination of sputum specimens?


a. Schistosoma mansoni
b. Metagonimus yokogawai
c. Paragonimus westermani
d. Schistosoma japonicum
e. Fasciolopsis buski

97. A 22-month-age boy brought to the emergency room with chief complaint shortness of breath. Inspection
of the chest showed retraction of the chest wall.
Which of the following muscle pulls the rib cage downward during expiration?
4 External intercostals
5 Abdominal recti
6 Sternocleidomastoid
7 Anterior serrate
8 Scaleni

98. A 70 yo man, admitted with chief complaint shortness of breath, cough, and yellowish sputum. History
diabetes mellitus +. Physical examination: respiratory rate 30 x/m, T: 38o C, BP: 130/80 mmHg, ronchi +/-,
wheezing -/-, blood glucose 276 mg/dL, chest X Ray : infiltrate +. There is no history of hospitalization
previously.
Which of the following describes most accurately about physical peculiarities found in case above?
g. The alveoli are filled with fluid due to increased hydrostatic pressure
h. The total surface area of respiratory membrane is decreased
i. The ventilation and perfusion ratio is consequently increased
j. The content of carbon dioxide in the blood will be decreased
k. The physiologic dead space also become decrease
Tim Akademik ANANTARA 2015
Divisi SL (Soal Latihan)
Fakultas Kedokteran Universitas Padjadjaran

99. A result of arterial blood gas analysis of 25 yo pneumothorax patient: pH: 6.971, pCO2: 71.4 mmHg, pO2:
97.4 mmHg, HCO3-: 32.1 mEq (naik), BE: +2.8, O2 saturation 96.4 %. These findings suggest homeostasis in
preserving gas exchange to fulfill metabolism demand.
Related to the case above, in the normal condition on the other hand, the negative pressure in the pleural
space is allowed through which of the following factor?
8 Strong and continous alveolar epithelium
9 Pumping of fluid from the pleural space by lympathics
10 Collection of large amounts of fluid in the pleural space
11 Hydrostatic pressure at the capillary
12 Osmotic forces at the alveolar membrane of the lungs

100. Related to case above, which of the following describes most accurately about the cough reflex?
a. Afferent nerve is mediated through trigeminal nerve (sneez reflex)
b. The effect is the opening of epiglottis followed by closing
c. Initially air is rapidly inspired
d. Both thoracal and abdominal muscles relax
e. The bronchi and trachea are dilated

101. A 55 yo male patient is admitted to the E.R with difficulty in breathing. According to his family, he has
been smoking for 30 years and consumed 16 cigarettes every day. His breathing difficulty has last for 1
year and becoming worst. He never went to health center before. Physical examination result shows
blood pressure 140/90 mmHg, heart rate 100 x/m and respiration rate 30 x/m: swallow breathing.
In term of oxygen saturation, it is strongly related to the pO2 and O2 delivery to the tissue. Which of the
following describes most accurately about the factors influencing the process?
5 Decrease of pH will lead to the shift of the curve to the right
6 Decrease CO2 in the blood will shift the curve to the right
7 Increase temperature will shift the curve to the left
8 Increase BPG will shift the curve to the left
9 Exercise will shift the curve to the left

102. A 70 yo man admitted with chief complain shortness of breath, cough and yellowish sputum. History
diabetes mellitus +. Physical examination: respiratory rate 30 x/m, T: 38oC, BP: 130/80 mmHg, ronchi +/-,
wheezing -/-, blood glucose 276 mg/dl, chest X ray: infiltrate +. There is no history of hospitalization
previously.
Which of the following describes most accurately about physical peculiarities found in the case above?
4 Alveolar macrophage effectively combated the infection process
5 The smooth muscles on the wall constrict many smaller airways
6 The obstruction will cause air entrapment in the alveoli
7 Bronchial obstruction will decrease airway resistance
8 The pathological process increase the lung diffusing capacity

103. A result of arterial blood gas analysis of 25 yo pneomothoraz patient: pH: 6.971, pCO2: 71.4 mmHg, pO2:
97.4 mmHg, HCO3-: 32.1 mEq, BE: +2.8, O2 saturation 96.4%. these findings suggest homeostasis in
preserving gas exchange to fulfill metabolism demand.
Related to the case above, ventilation process is somewhat disturbed. Which of the following breathing
work is mostly possibly involved in that situation?
31. Compliance work in increased
32. Tissue resistance work is increased
33. Airway resistance work is increased
Tim Akademik ANANTARA 2015
Divisi SL (Soal Latihan)
Fakultas Kedokteran Universitas Padjadjaran

34. Tissue resistance work is decreased


35. Airway resistance work is decreased

104. Once you were on duty in emergency room a 5 yo girl patient was taken by her parents with chief
complaint stridor since 2 days ago. Her parents also complained about sore throat, fever, difficulty on
swallowing and breathing. There was no cough and she was unable to control her own saliva and began to
drool.
Result from the physical examination are:
The child sat upright in a bent-forward position. The jaw was open and drooling was frequently present.
She presented a hot potato voice, suprasternal retraction and inspiratory stridor.
Radiological findings:
From soft tissue neck radiograph: “thumb sign” appearance (epiglottitis). Chest X-ray: within normal
limit.
Related to the case above, which of the following describes most accurately about the function of the
respiratory passageaways?
72. All passageways have cartilage to keep them from collapsing
73. All of the passageways are surrounded by smooth muscle
74. In obstructive disease all of the passageways are constricted
75. The greatest amount of resistance to airflow occurs in alveoli
76. In disease conditions, the smaller bronchioles determine resistance

105. WOTF is true about Maximum Expiratory Flow (MEF)?


a. MEF is much greater when the lungs are empty
b. in the enlarged lungs, the bronchi are completely collapse
c. beyond MEF, additional forces will no more increase in flow
d. further expiratory force may dilate the alveoli and the bronchus
e. compressing the chest cage tends to expands the bronchus

106. Decrease of MEF?


a. normal FEV1
b. normal FVC
c. increased FEV1/FVC ratio
d. increased FEV1
e. decreased FEV1

107. most important function of nose?


a. its warming effect is endowed by filtration process of nose
b. air conditioning of URT
c. the filtration procrss involves tubular precipitation by hair
d. the entrapped particles transported to trachea
e. filter the particle smaller than 1 micrometer

108. 45 yo woman presents with fever and cough. She has had no past medical problem and was well until about
3 days ago. PE is remarkable for temperature of 39 o C and the presence of diffuse rales on chest exam.
WBC count is elevated with left-shifted differential. Chest X-ray reveals patchy bilateral infiltrates. She is
unable to produce sputum. She has resting hypoxaemia and required hospital admission. What is the
mechanism of cough happened in this patient?
a. 2,5 L air is expired
b. abdominal pressure increased
c. alveoli pressure increased
d. afferent nerve impulse through the vagus nerve
e. automatic sequence triggered by fever

109. Rehab COPD acute stage?


Tim Akademik ANANTARA 2015
Divisi SL (Soal Latihan)
Fakultas Kedokteran Universitas Padjadjaran

Ans: breathing control

110. what is the characteristic of death by vagal reflex from strangulation case?
a. cyanosis
b. tardieu spot
c. bleeding at neck muscle
d. pulmo edema
e. pseudoform

111. In internal exam of drowning cases, its found?


a. Cutis anserine
b. Cadaveric spasm
c. Washerwoman hands
d. Tardieu spot
e. Pseudo foam

112. In cases of sea water drowning, is found ?


a. Hemodilution
b. Kadar plasma natrium meningkat
c. Kadar plasma kalium mmenurun
d. Hemolisis
e. Ventricular fibrillation

For Question 113-114 :


Brotoseno, 3 yo has been suffered asthma since 8 months old. In the last year his asthma exacerbate
>2x/month. Nocturnal symptom >1x/week. His activity and sleep were disturbed.

113. Classification severity asthma?


a. Mild intermitten
b. Mild persistent
c. Moderate persistent
d. Severe persistent
e. Chronic Asthma

114. Management for severe exacerbation asthma ?


a. Inhaled LABA
b. inhaled SABA
c. inhaled SABA + inhaled ipratropium bromide (IB)
d. inhaled SABA + inhlaled IB + ICS
e. inhaled SABA + inhaled IB + systemic corticosteroid

For Question for 115-118 :


5 y.o male presented to emergency Department with a history of breathlessness since 5 hours ago. This
symptom associated with cough and cold which has been occurring for a week at night and morning time. He
had been treat by GP with antibiotic, cold and cough remedies without any improvement. During the last year
those symptomps appeared recurrently. On PE he appeared alert, dyspneic, afebrile, chest intercostals and
subcostal reatraction were noted, prolonged expiration was noted, no expiratory nor crackles were heard. No
abnormality was found in other examination

115. Which of the following diagnosis is most likely to this patient ?


a. Moderate acute asthma attack
Tim Akademik ANANTARA 2015
Divisi SL (Soal Latihan)
Fakultas Kedokteran Universitas Padjadjaran

b. Mild acute asthma attack


c. Persistent asthma
d. Frequent episodic asthma
e. Acute asthma attack (frequent episodic asthma)

116. Which of the following initial management in the most appropriate for this patient?
a. nebulized salbutamol
b. nebulized ipratropium bromide
c. Pressured metered dose inhaler salbutamol
d. nebulized terbutalline
e. IV aminophyline

117. After giving 2x of one medication chosen above which was shown partial improvement, which of the
following diagnosis is most likely to this patient?
a. Moderate acute astma attack (Frequent Episodic Asthma/FEA)
b. Mild FEA
c. Severe FEA
d. FEA
e. Persistent Asthma

118. (Angga)

119.An 8yo boy comes to your clinic with exacerbation of asthma as his chief complain. What is the most
appropriate drug for him?
A. Terbutaline syrup
B. Inhaled ipraprotium bromide
C. Inhaled glucocorticoid
D. Inhaled salbutamol
E. IV aminophyline

120. 12 y.o girl suffered from moderate persistent asthma. What is the most appropriate drug for her?
a. Terbutalline syrup
b. Inhaled glucocorticoid
c. Inhaled beta-2 agonist
d. Critize oral …. (?)
e. Oral theophylline

121. Yusas

122. Asmi,9yo gitl has been suffered from asthma since 6yo. In the last year her asthma exacerbate once a
while,nocturnal symptom less then once a week. Between episodes,she has normal lung function.
Medication?
A. Rapid acting inhaled beta-1-agonist as needed
B. Daily low dose inhaled corticosteroid
C. Daily medium dose inhaled corticosteroid
D. Daily high dose inhaled corticosteroid
E. Daily medium dose inhaled corticosteroid plus oral glucocorticoid

123. 5mo boy,laryngomalacia,frequently had aspiration when he got milk. Last 3 days--> pneumonia. Most
common etiologic pathogen of this patient age&adolescent?
A. S pneumonia
B. S aureus
Tim Akademik ANANTARA 2015
Divisi SL (Soal Latihan)
Fakultas Kedokteran Universitas Padjadjaran

C. Pseudomonas aeruginosa
D. E coli
E. M pneumonia

124. 22 yo female w/ history of intermittent wheezing in response to exercise present to emergency room w/
shortness of breath. Her attack occured during aerobic class. At this point she is having obvious
difficulty breathing & has diffuse wheeze on pulmonary exam. Oxygen saturation 95% by pulse oxymetry.
WOTF is most effective treatment at this point?
a. IV amynophyline
b. inhaled cromolyn sodium
c. inhaled albuterol (SABA)
d. IV hydrocortisone
e. inhaled beclometasone

125. A boy has admitted to RSHS with dyspnea as a chief complaint. Sub bronchial sound in the upper part of
the right thorax was found. Chest xray: consolidation of 2 upperlobe. What is classification of this
disease?
A. CAP interstitial
B. CAP lobar
C. CAP bronchopneumonia
D. HAP lobar
E. HAP bronchopneumonia

126. 5 mo baby, dtg ke ER.Komplain: fever 10 days. No seizure Cough 2 weeks No BCG vaccine Treatment:
antibiotik not improve X-ray: snow flakes appearance Hepatomegali. Diagnosis?
A. tb meningitis
B. miliary tb
C. sepsis
D. pneumonia
E. acute bronkitis.

127. On a frontal chest film,the apex of the lung retracts toward of visceral pleura is visible,separated from
chest wall by a radiolucent space devoid of lung markings..
A. Pneumothorax
B. Giant bulla
C. Giant emphysema
D. -
E. Pleural effusion

128. Radiography of sphenoidal sinus:


A. Lateral
B. Caldwell
C. Waters
D. Rhese
E. Stanver

129. a 10yo girl dyspnea since 7days ago. AP chest xray: homogenous lung opacification with air bronchograms
in the lateral segment of the lung. The segmental homogenous lung opacification with air bronchograms is
:
A. Atelectasis
B. Pneumonia
C. Bronchopneumonia
Tim Akademik ANANTARA 2015
Divisi SL (Soal Latihan)
Fakultas Kedokteran Universitas Padjadjaran

D. Pleural effusion
E. Schwarte

130. A 20 y.o. Girl , chief complaint: dyspnea since 7 days ago accompanied by cough & fever.
In PA chest x-ray: The left costophrenic angle is blinded by a fairly well defined & concave upper edge
homogeneous opacity.
It is caused by:
A. Atelectasis
B. Pneumonia
C. Bronchopneumoni
D. Pleura effusion
E. Schwarte

131. 25 y.o patient comes to pulmo clinic. The patient is cough > 3 weeks and accompanied by the production
of purulent sputum, night sweats, weight loss, anorexia, general malaise, weakness. X ray to see small
pleural effusion
A. Lateral chest film
B. Oblique chest film
C. lateral decubitus position
D. Apical lordotic
E. Anteroposterior chest

132. Posterior anterior chest xray revealed cavity in apex of left upper lobe, wall cavity is thick and surrounded
by patchy consolidation. What is it?
A. Lung abscess
B. Pulmonary cyst
C. Mycetoma in cavity
D. Cavity tumor
E. Cavity of pulmonary TB

133. 25 y.o patient comes to pulmo clinic. The patient is cough > 3 weeks and accompanied by the production
of purulent sputum, night sweats, weight loss, anorexia, general malaise, weakness. What is the basic
standard radiograph for any patient presenting with cough >3 weeks.
A. Posteroanterior chest film
B. Oblique chest film
C. Left lateral decubitus
D. Apical lordotic
E. Anteroposterior chest

134. Persistent nasal discharge, yellowish, thick, past history: nasal itchy and obstruction in cold weather,
water's film: clouding right maxillary sinus. Diagnosis?
A. Tumor right max sinus,
b. Right hematosinua,
c. Acute sinusitis
d. Chronic sinusitis
e. Mucocele

135. A doctor prescribed an anti tuberculosis drug. What medicine caused red orange urine?
A. Rifampicyn
b. Ethambuthol
c. Pyrazinamide
Tim Akademik ANANTARA 2015
Divisi SL (Soal Latihan)
Fakultas Kedokteran Universitas Padjadjaran

d.Isoniazid
e.streptomycin

136. A patient 30 y.o. Receive anti TB treatment and he has pain in right big toe swollen & low grade fever and
diagnosed gouty arthritis and high uric acid in blood. Treatment used?
A. Isoniazid
B. Rifampin
C. Pyrazinamide
D. Cycloserine
E. Streptomycin

137. 25 years old man could not obtain lisence driver because poor performance in red green color vision
discrimination. No family history color vision. He was taking 4 drugs regimen for tuberculosis. Which anti
TB drug responsible?
a. Rifampin
b. Pyrazinamide
c. Ethambutol
d. Isoniazid
e. Streptomycin

Clinical situation for 138-139


15 y.o girls taken to the clinic by her mom with chief complaint 1 day history of right ear discharge. Discharge
is yellow,thick,odorless, also pain and aural fullness on right ear with febrile. after doctor perform pe and lab,
she was diagnosed acute right otitis media stage suppuration. She receive erythromycin as treatment.
138. Why erithromycin is a drug of choice of this patient? Because it is effective:
a. Against aerobic
b. Against anaerobic
c. Against gram positive
d. Against gram negative
e. Against all bacteria

139. The treatment is change from erythromicin to azytromicin. She has gastritis and consume antacid to
relieve the gastritis. What will happen in this situation ?
A. Antacid reduce bioavailibility of azytromicin
B. Azytromicin alters bioavailibility of antacid
C. Antacid delay absortion & reduce peak serum concentration
D. Azytromicin delay absorption & reduce peak serum concentration
E. Azytromicin delay excretion & reduce peak serum concentration

140. Streptomycin is very important in life threatening condition of tb. What is the side effect of streptomycin
?
gangguan hearing (Ototoxicity)

141. A 20 y/o ,am with asthma present with complaints of cough and chest tightness, ehich have been accuring
daily, requiring him to inhale a Beta receptor agonist. He notes waking up from sleep approximately three times
per weeks with similar symptoms. If he is taking sympathomimetic agent and inhaled corticoste...roids to
improve his asthma control, which of the following new generation of long acting β-selective agonist to aim
of his therapy?
a. Metaproterenol
b. Pirbuterol
c. Albuterol
Tim Akademik ANANTARA 2015
Divisi SL (Soal Latihan)
Fakultas Kedokteran Universitas Padjadjaran

d. Terbutalin
e. Salmeterol

142. 32 y o male came to clinic with chief complaint dyspneu and fever, HT: never suffered same experience
before., no history of the same disease in the family. Hipersensitif thd beta laktam antibiotik, so doctor give
another antibiotik. PEà BP: 130/80, HR: 90bpm, temp : 39, RR : 36x, microbial culture: pneumococcus.
Few hour after taking drugs had abdominal discomfort. Wotf MOST POSSIBLE antibiotic taken??
a. azithromycin
b. cefadroxyl.
c. clarithromicyn
d. clindamycin
e. erythromycin

For question number 143 to 144, refer to scenario below:


A 20 y/o woman came to the doctor with dyspnea and fever. She has been had this condition for the last 2 days
and now its became worsen. She is noted as betalactam hypersensitive patient.
In physical examination found RR 36x/minutes, temperature 38.6oC, epigastric and intercostals retractions;
vesicular breathing sound and crackles was found by auscultation.
Laboratory result showed WBC 18.000/mm. Blood culture: Streptococcus pneumonia

143. What is the most likely appropriate drug?


a. Amoxicyllin
b. Ceftriaxone
c. Azitromicin
d. Netilmicin
e. Vancomycin

144. What is the most appropriate drug for her if she comes from low society?
a. Erythromycin
b. Cefoperazone
c. Azithromicin
d. Vancomycin
e. Spiramycin

145. The presence of fluid in the thoracic cage reduces ventilation capacity.
Thoracocentesis procedure to remove the fluid is best accomplished by inserting needle:
a. Adjacent to the sternum in the second intercostals space
b. Adjacent to the sternum in the fifth intercostals space
c. In the midclavicular line in the fifth intercostals space
d. In the midaxillary line in the seventh intercostals space
e. In the twelfth intercostals space adjacent to vertebral column

146. During thoracocentesis procedure,the physician must be careful to avoid the intercostals neurovascular
bundle, which is particularly susceptible to injury from a fractured rib because it lies:
a. Behind the superior border of the rib
b. Beneath the inferior border of the rib
c. Between external and internal intercostals layers
d. directly behind the midpoint of the rib.
e. halfway between two adjacent ribs

147. Two elderly persons, both were diagnosed as having pleurisy, were comparing their cases. One had left
latera; thoracic pain, whereas other had left neck and shoulder pain. They went to the point of reviewing the
academic qualifications of their respective physicians. Assuming the diagnosis was correct, the explanation for
these differences in pain distribution is that pleura in innervated by:
a. Cervical sphlanchnic and thoracic sphlanchnic nerves
b. Intercostal and phrenic nerves (visceral)
c. Parasympathetic and symphatetic nerves
Tim Akademik ANANTARA 2015
Divisi SL (Soal Latihan)
Fakultas Kedokteran Universitas Padjadjaran

d. Phrenic and vagus nerves


e, Vagus and intercostals nerves

148. purulent drainage from sup. Nasal meatus come from which sinus?
a. ethmoid
b. frontal
c. sphenoid
d. maxilla
e. sphenoethmoid

149. during PE of pt with facial ache, ottolaryngoligist noted drainage into left middle meatus when patien
lying inthe right side. Most likely infection:
a. anterior ethmoid sinus
b. frontal sinus
c. maxillary sinus
d. nasolacrimal duct
e. maxillary sphenoid sinus

150. a 15 y.o boy eating a fish and in advertently has a bone caught in his throat, complaint à significant pain
above the vocal cord. WOTF responsible carrying the sensation for this pain?
a. superior laryngeal nerve
b. recurrent laryngeal nerve
c. spinal accessory nerve
d. hypoglossal nerve
e. maxillary nerve

151. 33 years old women underwent partial thyroidectomy. She is noted to have some hoarseness of voicess 1
month later. What of the allowing is the most likely explanation.
G. Endotracheal tube trauma to vocal cords
H. Injury to cricoid cartilage
I. Injury thyroid cartilage
J. Injury to recurrent laryngeal nerve
K. Injury to arythenoid cartilage

152. A guy admitted to ER. He aspirated peanut. Bronchoscopy perormed. Which side is the peanut probably
located?
17. Right lower lobe
18. Let lower lobe
19. Right upper segmental bronchus
20. Left upper segmental bronchus
21. Acini

153. a 28 years old neuroanatomi graduate student noted pain at the bridge of his nose and had been told that
he had sinus infection. He was speculating about the afferent nerve supply from this area. What o the
following is the most accurate description of sensory nerve innervation?
54. Branches of CN III
55. Branches of CN IV
56. Branches of CN V
57. Branches of CN VII
58. Branches of CN IX

154. a 5 years old complain severe pain for right ear due to acute otitis media. Which of the following nerve
is the most likely to be responsible in carrying sensation pain of timpanic membrane?
68. CN V
Tim Akademik ANANTARA 2015
Divisi SL (Soal Latihan)
Fakultas Kedokteran Universitas Padjadjaran

69. CN VII
70. CN VIII
71. CN IX
72. CN X

155. when a person is standing upright, the gravity pulls the pleural fluid. What is the lowest level of the
fluids drains to during that situation?
75. Costomediastinal recess
76. Costodiaphragmatic recess
77. Obligue fissure
78. Horizontal fissure

156. 24 years old medical student diagnosed having sinusitis and ask his physician why he experienced a nasal
drainage during night but not day. What you explanation?
a. Location ostia within sinus
b. Location ostia within nasal passage
c. Disruption of drainage due to mastoid
d. Diurnal mucus production increase at night
e. Locatin of ostia within eustachian tube

157. 55 years old anemic dan hypotension due to anterior epistaxis . which of major artery?
78. Ethmoidal artery
79. Sphenopalatine artery
80. Superior labial artery
81. Greater palatine artery
82. Alveoar superiar artery

158. 18 years old woman come to clinic with chief complaint anterior epistaxis, on going bleeding on right
nostrill. Which of the following most likely source of bleeding?
8. Anterior nasal septum
9. Posterior nasal septum
10. Anterior turbinate
11. Posterior turbinate
12. Nasal septum

159. these closed cavities, there are in the frontal, maxillary, ethmoid, and sphenoid bones. These cavities
communicate with the nasal cavity. Which of the following epithelial tissue is the most likely lined the
mucous layer of these cavities?
70. Stratified squamous non keratinized epithelium
71. Stratified squamous keratinized epithelium
72. Stratified columnar epithelium
73. Pseudostratified columnar epithelium (epithel sinus)
74. Stratified cuboidal epithelium

160. below the epiglottis, the mucosa o larynx form 2 pairs of fold, the upper pair and the lower pair. Which of
the following epithelial tissues most likely lined the mucous layer of the lower pair fold?
17. Stratified squamous non keratinized epithelium
18. Stratified squamous keratinized epithelium
19. Stratified columnar epithelium
20. Pseudostratified columnar epithelium
21. Stratified cuboidal epithelium
Tim Akademik ANANTARA 2015
Divisi SL (Soal Latihan)
Fakultas Kedokteran Universitas Padjadjaran

161. This single conducting portion of respi system is lined with respi epithelium. In the lamina propria are 16-
20 C shaped ring of hyaline cartilage. WOTF is most likely portion?
a. nasal cavity
b. nasopharynx
c. larynx
d. trachea
e. bronchiole

162. In the larger portions of this conducting respiratory system, the hyaline cartilage rings completely
encircle the lumen. As the diameter decreases, the cartilage rings are replaced with isolated plate. WOTF
is the most likely portion?
A. larynx
B. trachea
C. bronchi
D. bronchioles
E. terminal bronchioles

163. Bagian tsb tidak ada cartilage maupun gland. Ada clara cell. Sel apa yang melapisi? Terminal Bronchiole
a. simple squamous
b. simple cuboidal
c. simple columnar
d. pseudostratified cuboidal
e. pseudostratified columnar

164-165
A 25 y.o. woman got traffic accident while driving his car 2 days ago and had been amputated (right lower
limb) yesterday. Few hours post operation his condition was decreased & finally she died.

164. Which of the following is the most likely cause of death.


a. fat embolism
b. air embolism
c. fluid embolism
d. thromboembolism
e. amnion embolism

165. Post mortal autopsy reported a pulmo embolism. WOTF is the most common source pulmo emboli.
a. Hypogastric vein
b. SVC
c. deep veins of the thigh
d. axillary vein
e. jugular vein

166. 64 y.o. man come to respiratory clinic with difficulty in breathing since yesterday, had chronic history of
asthma bronchiole, from PE: temp 39, Pulse 115 bpm, radiology: barrel chest, leukocyte: 15000. WOTF is
the etiology?
a. diffuse interstitial lung fibrosis
b. portal hypertension
c. lymphoid hyperplasia
d. medial hypertrophy of pulmo artery vessel
e. chronic pneumoconisiosis

167. Most common cause of bacterial pneumonia


a. pneumocystic carinii
b. S aureus
c. E coli
d. S pneumonia
e. pseudomonas aeruginosa
Tim Akademik ANANTARA 2015
Divisi SL (Soal Latihan)
Fakultas Kedokteran Universitas Padjadjaran

168. A 42-year-old black woman presents with shortness of breath. She is a non smoker. A diffuse interstitial
pattern with small parenchymal nodules is seen on chest radiograph, as well as massively enlarged hilar
lymph nodes. Biopsy was performed.
Which of the following is the most likely microscopic feature from transbronchial biopsy of the lung?
a. Loss of alveolar walls with minimal changes in the airways
b. Non caseating granulomata
c. Many neutrophils in alveoli with edema
d. Type II pneumocyte proliferation with chronic interstitial inflammation & intranuclear viral inclusions
e. An increased mucus

169. A 70-year-old male, who was a heavy smoker, complained of blood tinged sputum. A chest X-ray & CT
demonstrated a large hilar mass with retraction of mediastinum. He also complained of headaches. A
work up showed nodules in the brain. A transbronchial biopsy was performed. The biopsy consistence
with malignancy.
Which of the following is the most likely diagnosis?
a. Lymphoma
b. Squamous cell carcinoma
c. Adenocarcinoma
d. Small cell carcinoma
e. Malignant mesothelioma

170. A 54-year-old suffered lung cancer since 6 months ago, came to hospital with chief complaint of low
vision and enlargement of the eye-ball. Physical examination revealed ptosis, miosis, hemianhidrosis.
Which of the following site is the most likely tumor location?
a. Inferior pulmonary lobule
b. Superior pulmonary sulcus lesions
c. Peripheral pulmonary lesions
d. Terminal lobules pulmonary lesions
e. Central lobules pulmonary lesions

171. A 70-year-old suffered lung cancer since 6 months ago, came to hospital with chief complaint of low
vision and enlargement of the eye-ball. Physical examination revealed ptosis, miosis, hemianhidrosis.
Which of the following conditions is the most likely to have?
a. Tuberculosis
b. Bronchogenic pneumonia
c. Bronchiectasis
d. Bronchogenic carcinoma
e. Emphysema

172. Patient who inhales his left lower bridge consisting of two teeth and completely obstruct his right lower
lobe main bronchus. Wise but stubborn, he refuses surgical intervention for 2 weeks. By that time a right
lower lobotomy is necessery.
The LEAST likely pathological finding in light of the history?
a. Atelectasis
b. Emphysema
c. Empyema
d. Bronchiectasis
e. Bronchopneumonia

173. A 45-year-old man develops severe shortness of breath after automobile accident. He is thought to have
adult respiratory distress syndrome. Histo finding most characteristic of the first lesion?
a. Mucous plug
b. Hemosiderin laden macrophage
c. Multinucleated giant cell
d. Abcess
e. Hyalin membrane
Tim Akademik ANANTARA 2015
Divisi SL (Soal Latihan)
Fakultas Kedokteran Universitas Padjadjaran

174. A 45-year-old man smoked 2 packs of cigarettes/day for 20 years. For the past years, he has had a chronic
cough with abundant mucoid expectoration. Several episodes or respiratory tract infection, developed
difficulty of breathing, wheezing, & consumed salbutamol.
Which of the following is the most likely microscopic feature?
a. Alveolis are vary size. Some of them show enlargement & sinusoidal
b. Enlargement bronchiolus terminalis caused by loss of elastin protein
c. Bronchus until alveoli should be normal
d. Air entraped in alveoli
e. – (gak sempet nyatet)

175. A 67-year-old man has had long standing smoker, and for 5 years show the symptom breathless when
exercise, cough > 3 / month with productive sputum. PE: RR 30x/minute, barrel shaped chest, wheeze &
crackles.
Which of the following is the most likely pathological finding?
a. Trachea-brochioles enlarge in internal diameter, inflammatory cell infiltrate the surface epithelium, and
enlarge mucus secreting gland
b. Trachea-brochioles smaller in internal diameter, inflammatory cell infiltrate the surface epithel, and
hyperplasia & hypertrophy mucus gland
c. Destruction of lung parenchyme in COPD caused constriction & destruction of respiratory bronchioles
d. Pulmonary vascular change are characterized by thinning of vessel wall, thickening of the intima
followed by decrease in smooth muscle
e. Increasing collagen content and scar tissue in respiratory tract caused by narrows the lumen and
imbalance of endogenous proteinase & anti-proteinase in the lung
Tim Akademik ANANTARA 2015
Divisi SL (Soal Latihan)
Fakultas Kedokteran Universitas Padjadjaran

Potrebbero piacerti anche